You are on page 1of 185

MC Question

Chapter 18 Gases
1.

(MC)18P1

(80)
A mixture of two gases X and Y is maintained at constant temperature. The relative molecular mass of X is 9 times
that of Y. What is the ratio of the r.m.s. speed of molecules of Y to that of molecules of X?
A. 3

B. 3 2

C. 9

D. 18

E. 81

2.

(81)
The equation of state for mass m of an ideal gas may be written as p V = m r T. With reference to this equation,
(1) the value of r depends upon the particular gas used.
(2) r is independent of m.
(3) if R is the universal gas constant, then the mass of each mole of this gas is R / r.
A. (1) only
B. (3) only
C. (1) & (2)only
D. (2) & (3) only
E. (1), (2) & (3)

3.

(81)
An ideal gas is enclosed in a container at absolute temperature T. The absolute temperature is now raised to 2T,
whilst the volume is kept constant. Which of the following statements is correct ?
(1) The average separation of the molecules is reduced by a factor of 2 .
(2) The average speed of the molecules is doubled.
(3) The average kinetic energy of the molecules is doubled.
A. (1) only
B. (3) only
C. (1)&(2)only
D. (2)&(3)only

4.

E. (1),(2)&(3)

(82)
Mass M of an ideal gas at pressure p occupies volume V. The r.ms. speed of its molecules is
A.

pV / 3M

B.

pV / M

C.

3 pV / M

D.

pVM

E.

3 pVM

5.

(83)
A cylinder containing air is fitted with an air-tight frictionless piston maintained at a constant temperature. The
piston is moved very slowly inwards until the volume of the cylinder has halved, Which of the following
quantities has doubled ?
(1) The average speed of the gas molecules in the cylinder.
(2) The average momentum of the gas molecules in the cylinder.
(3) The average force exerted by the gas molecules on the piston.
A. (1) only
B. (3) only
C. (1) & (2) only
D. (2) & (3) only
E. (l), (2) & (3)

6.

(84)
A fixed mass of gas at s.t.p. occupies a volume of 1 m3. The gas is heated and allowed to expand to a final volume
of 2 m3 with its pressure doubled. The average kinetic energy of the gas molecules is
A. reduced to one quarter of its initial value.
B. halved.
C. unchanged.
D. doubled.
E.
increased four times.

MC Question
Chapter 18 Gases

7.

(MC)18P2

(84)
The graph shows the distribution of speeds (v) of the molecules in a constant mass of gas. Which of the following
statements is/are correct?
N

v
c
(1) The value v = c at which the peak of the curve occurs increases when the temperature rises.
(2) The peak of the curve rises when the temperature rises.
(3) Provided that the temperature does not change, the kinetic energy of each molecule is fixed.
A. (1) only
B. (3) only
C. (1) & (2) only
D. (2) & (3) only
E. (1), (2) & (3)

8.

(85)
According to the kinetic theory of gases, at a given temperature, the molecules of different gases have the same
(1) average speed.
(2) average kinetic energy.
(3) average total energy
A. (1) only
B. (3) only
C. (I) & (2) only
D. (2) & (3) only
E. (1), (2) & (3)

9.

(85)
A vessel of volume 1 u 10-3 m3 contains 0.72 g of an ideal gas at a pressure of 1 u 105 Pa. The r.m.s. velocity of
the gas molecules is
B. 110 m s-1
C. 340 m s-1
D. 650 m s-1
E. 3400 m s-1
A. 20 m s-1

10.

(85)

B
S
P1

P2

T1

T2

A and B are two identical containers connected by a tap S initially closed. A contains an ideal gas at a pressure P1
and a temperature T1. B contains the same gas at a pressure P2 and a temperature T2. The tap S is then opened. If
the temperatures of the containers A and B remain constant at T1 and T2 respectively, the final pressure of the gas
mixture will be
A. (P1 + P2) / 2.
B. (P1 + P2).
C. (P1T1 + P2T2) / (T1 + T2).
D. (P1T2 + P2T1) / (T1 + T2).
E.
(P1T1 + P2T2) u (T1 + T2).
11.

(86)

2N

container X

container Y

Container X holds N molecules of a certain gas at atmospheric pressure. Container Y has the same volume as X
and hold 2N molecules of the same gas. Both containers are at the same temperature. The ratio of the average
number of collisions per second with the walls in X to those in Y is
A. 1 : 2

B. 1 : 2

C. 1 : 1

D. 2 : 1

E. 4 : 1

MC Question
Chapter 18 Gases

(MC)18P3

12.

(86)
At 0C temperature and 1.00 u 105 N m-2 pressure, the density of a gas is 0.179 kg m-3 . The r.m.s. speed of the
gas molecules at 91C will be
A. 231 m s-1.
B. 470 m s-1.
C. 1290 m s-1.
D. 1490 m s-1.
E. 1730 m s-1.

13.

(87)

N (c )

c
The graph shows the distribution of molecular speeds c for a gas at room temperature. N (c) represents the number
of molecules 'N in a small range of speeds c to c + 'c so that 'N = N (c) 'c. As the temperature is increased,
A.
B.
C.
D.
E.

14.

the peak position will move to the right and its height will increase.
the peak position will move to the right and its height will decrease.
the peak position will move to the left and its height will increase.
the peak position will move to the left and its height will decrease.
the peak position and its height will both remain unchanged.

(87)
Two closed vessels X and Y contain equal masses of an ideal gas. X has a greater volume than Y. When the
temperature T changes, which of the following represents the variation of the pressure P of the gas in each vessel
with temperature T?
A.
B.
C.
D.
E.
P

X, Y

X
T/ oC

15.

Y
T/ oC

T/ oC

T/ oC

(88)
An ideal gas is contained in two metal cylinders A and B connected by a tap, which is initially closed. The volume
and pressure of the gas in the cylinders are as follows:
volume / m3
pressure / Pa
5
A
5 u10
11 u10-3
5
4 u10-3
B
2 u10
When the tap connecting the two cylinders is opened, what will be the final pressure in the vessel ? You may
assume that the temperature remains constant.
A. 2.4 u 105 Pa
B. 3.5 u 105 Pa
C. 4.2 u 105 Pa
D. 5.0 u 105 Pa
E.
6.9 u 105 Pa

T/ oC

MC Question
Chapter 18 Gases

(MC)18P4

16.

(88)
For smoke particles undergoing Brownian motion in air, the motion of the smoke particles is mainly caused by
A. air convection currents.
B. the interaction between oxygen and the nitrogen molecules.
C. collisions between air molecules.
D. collisions between smoke particles.
E.
collisions between air molecules and smoke particles.

17.

(88)
An inexpansible vessel contains 1.2 kg of gas at 300 K. What is the mass of gas expelled from the vessel if it is
heated from 300 K to 400 K under constant pressure ?
A. 0.25 kg
B. 0.3 kg
C. 0.6 kg
D. 0.75 kg
E.
0.9 kg

18.

(88)
Which of the following is NOT an assumption in deriving the kinetic theory of gasses ?
A. The volume of the molecules is negligible compared with the volume of the gas.
B. Attractive forces between the molecules are negligible.
C. The duration of a collision is negligible compared with the time between collisions.
D. Collisions with the walls of the container and with other molecules cause no change in the average kinetic
energy of molecules.
E.
The molecules suffer negligible change of momentum on collision with the walls of the container.

19.

(88)
An ideal gas is at temperature T. If the mass of a gas molecule = m, the universal gas constant = R and the
Avogadro constant =NA then the r.m.s. speed of the molecules is
A.

3RT / m

B.

RT /(mN A )

D.

RTN A / m

E.

3RTN A / m

C.

3RT /(mN A )

20.

(89)
Two vessels of equal volume both contain an ideal gas and are connected by a tube of negligible volume. Initially
both vessels are at temperature To and pressure Po. One vessel is maintained at To while the temperature of the
other is raised to T. The new pressure is then given by
B. TPo /T
C. ToPo / T
D. 2TPo / (T+To)
E. Po(T+To)/2To
A. Po

21.

(89)
The internal energy of an ideal gas at temperature T is 3 C T / 2, where C is a constant.
Given that R = the universal gas constant, NA = Avogadro constant, k = Boltzmann constant.
For an ideal gas containing N molecules, the constant C is equal to
A. NAR.
B. NA k.
C. NR / NA.
D. N k/ NA.

22.

E. NR.

(89)
Two different gases A and B are contained in two identical vessels. If the ratio of their molecular masses and
absolute temperatures are respectively 8 : 1 and 2 : 1, the ratio of their r.m.s. molecular speeds will be
A. 1:4
B. 1:2
C. 1:1
D.2:1
E. 4:1

MC Question
Chapter 18 Gases

(MC)18P5

23.

(90)
Container X holds pure hydrogen gas while container Y holds pure oxygen gas. If the hydrogen molecules in X
have the same r.m.s. speed as the oxygen molecules in Y, which of the following conclusions may be drawn ?
A. The gas in X has a higher temperature than Y.
B. The gas in X has a higher pressure than Y.
C. The gas in Y has a higher temperature than X.
D. The gas in Y has a higher pressure than X.
E.
The gases in X and Y are at the same temperature.

24.

(91)
A fixed mass of ideal gas at S.T.P. occupies a volume of 2 m3. The gas is heated and allowed to expand to a final
volume of 4 m3 with its pressure doubled. The root mean square speed of the gas molecules is
A. reduced to one quarter of its original value.
B. halved.
C. unchanged.
D. doubled.
E.
increased four times.

25.

(91)
Which of the following properties of molecules of an ideal gas is/are the same on the moon as on the earth, if the
temperature and volume of the gas are unchanged ?
(1) The average momentum change when a molecule of the gas rebounds from a wall of the container
(2) The average kinetic energy of a molecule of the gas
(3) The weight of a molecule of the gas
A. (1) only
B. (3) only
C. (1) & (2) only
D. (2) & (3) only
E. (1), (2) & (3)

26.

(91)
Which of the following statements about an experiment showing the Brownian motion with smoke particles in air
is/are correct?
(1) The motion is caused by collisions between air molecules and smoke particles.
(2) The experiment makes it possible to see the motion of the air molecules.
(3) The motion is irregular because air is a mixture of gases, and the molecules have different masses.
A. (1) only
B. (3) only
C. (1)&(2)only
D. (2)&(3)only
E. (1),(2)&(3)

27.

(92)
Which of the graphs best represents the distributions of molecular speeds in a gas at 500 K if the dotted curve
represents this distribution for the same gas at 300 K?
A.
B.
C.
fraction of
molecules

fraction of
molecules

speed
0

D. fraction of

(92)
Given :

speed
0

molecules

speed

28.

speed
0

E. fraction of

molecules

fraction of
molecules

speed
0

Avogadro constant = 6 u 1023 mol-1


Boltzmann constant = 1.38 u 10-23 J K-1
Universal gas constant = 8.31 J mol-1 K-1
If 1 mole of an ideal gas is heated under constant pressure from 20oC to 70oC, the total energy of the gas
molecules is increased by
A. 208 J
B. 415 J
C. 581 J
D. 623 J
E. 831 J

MC Question
Chapter 18 Gases

(MC)18P6

29.

(92)
The relative atomic mass of oxygen is 16. What is the ratio of the average speed of oxygen molecules to that of
hydrogen molecules at room temperature?
A. 1/ 16
B. 1/4
C. 1
D. 4
E. 16

30.

(93)
Identical containers A and B contain oxygen (O2) and hydrogen (H2) respectively. Both gases are at room
temperature arid atmospheric pressure. Which of the following statements is/are true?

O2

H2

Container A
Container B
In both containers,
(1) the number of gas molecules is the same;
(2) the r.m.s. speed of gas molecules is the same;
(3) the frequency of collision of gas molecules with the walls of container is the same.
A. (1) only
B. (3) only
C. (1)&(2)only
D. (2)&(3)only

E. (1),(2)&(3)

31.

(93)
An inexpansible vessel contains air at 50C. What percentage of air remains in the vessel if it is heated to 100C
under constant pressure ?
A. 87%
B. 85%
C. 73%
D. 63%
E. 50%

32.

(95)
The r.m.s. speed of the molecules of a certain gas X is 341 m s-1 at 298 K. Find the molar mass of the gas X.
(Given : Universal gas constant R = 8.31 J mol-1 K-1 )
A. 5.4 g
B. 21.8 g
C. 33.8 g
D. 42.6 g
E. 63.9 g

33.

(95)
The pressure of an ideal gas in a container is P. If the number of gas molecules is halved, the volume of the
container is doubled and the temperature is kept constant, the pressure will be
1
1
A.
P
B.
P
C. P
D. 2P
E. 4P
4
2

34.

(97)
Two different ideal gases, A and B, are contained in two identical vessels. If the ratio of their absolute temperature
and the ratio of the root-mean-square speed of the molecules ewe respectively 2 : 1 and 3 : 1, the ratio of their
molecular mass is
A. 2:3
B. 2:9
C. 1:6
D. 9:2
E. 3:2

35.

(97)
When a gas is below its critical temperature, which of the following is/are correct ?
(1) Intermolecular forces are significant.
(2) The gas can be liquefied by applying pressure alone.
(3) Collisions between molecules are not perfectly elastic.
A. (1) only
B. (3) only
C. (1) & (2) only
D. (2) & (3) only

36.

E. (1), (2) & (3)

(98)
The root-mean-square speed of a sample of helium gas molecules, each of mass m, is c. Which of the following
deductions is correct ?
A. The percentage of molecules travelling at speed c is greater than at other speed.
B. Half of the molecules travel at a speed higher than c.
C. All molecules travel randomly with speed c.
D. The average speed of the molecules is c.
1
E.
The average kinetic energy of the molecules is mc 2 .
2

MC Question
Chapter 18 Gases

37.

(MC)18P7

(99)
Two metallic containers X and Y of volume V and 4V respectively are connected by a narrow tube as shown.
Initially the tap S is closed and an ideal gas is contained in X at a pressure of 400 kPa while container Y is
Y
evacuated.
X

S
4V

The tap S is then opened and when equilibrium is finally reached,


A. the gas pressure in X is 100 kPa.
B. there are still gas molecules moving through the tap S.
C. the product of pressure and volume of the gas in X is equal to that in Y.
D. the density of gas molecules in X is greater than that in Y.
E.
the gas molecules in Y on average move faster than those in X.
38.

39.

(99)
Which of the following description about ideal gas is/are correct?
(1) An ideal gas obeys Boyle's law only under high temperature and low pressure.
(2) The molecules of an ideal gas have no size.
(3) The internal energy of an ideal gas consists of kinetic energy only.
A. (1) only
B. (3) only
C. (1) & (2) only
D. (2)&(3) only
(00)
Which of the following statements concerning a real gas is/are correct ?
(1) Collisions between molecules and the wall of a container are not perfectly elastic.
(2) The volume of the molecules cannot be neglected.
(3) Intermolecular forces cannot be neglected.
A. (1) only
B. (2) only
C. (3) only
D. (1)&(2) only

40.

(01)
Which of the following is NOT a basic assumption of the kinetic theory of an ideal gas?
A. All molecules are in random motion.
B. All molecules move with the same speed at a certain temperature.
C. All molecules are point particles that have no physical size.
D. All collisions are perfectly elastic.
E.
All molecules do not exert force on one another except during collisions.

41.

(02)

E. (1),(2)&(3)

E. (2) & (3) only

P
B
A
0

The graph shows the relation between the pressure P and the absolute temperature T of a fixed mass of an ideal
gas, which changes from state A to state B along the path AB. Which of the following statements is/are correct?
(1) The graph shows that P is directly proportional to T.
(2) The volume V of the gas increases.
PV
= constant.
(3) All the points on straight line AB satisfy the relation
T
A. (1) only
B. (3) only
C. (1) and (2) only
D. (2) and (3) only

MC Question
Chapter 18 Gases

(MC)18P8

42.

(02)
What is the order of magnitude of the number of molecules in 1 cm3 of air in an atmospheric pressure of 105 Pa
and at room temperature?
(Given: Universal gas constant = 8.31 J K-1 mol-1
Avogadro constant = 6.02 u 1023 mol-1)
25
B. 1019
C. 1016
D. 1013
A. 10

43.

(02)
At 80, the r.m.s. speed of the molecules in a fixed mass of an ideal gas is c. If the temperature is increased to
160, the r.m.s. speed of the gas molecules would become
A. 2c.
B. 1.4c.
C. 1.2c.
D. 1.1c.

44.

(03)
A fixed mass of an ideal gas undergoes changes from state X to state Z via state Y as shown in the plot of pressure
p
p against volume V below.
Y

Which graph best shows how the absolute temperature T of the gas varies with its volume V?
A T
B. T
C. T
D. T
Y

Z
Z

X
0

X
3

V 0

45. (04)
An ideal gas of volume V and pressure p undergoes a change from state X to state Z via state Y along the path as
shown in the p-V plot.

p
X
Y
Z
V
0
Which of the following descriptions about the gas at X, Y and Z is correct?
A. The gas is at its coolest at X and is at its hottest at Y.
B. The gas is at its coolest at X and is at its hottest at Z.
C. The gas is at its coolest at Y and is at its hottest at X.
D. The gas has the same degree of hotness at X, Y and Z.

MC Question
Chapter 18 Gases
46.

Out of
Syllabus

(MC)18P9

(04)
In which of the following graphs does the shaded area represent an amount of energy?
(1)

(2)

0
Q
V

0
e
F

= charge on a capacitor
= p.d. between plates

A. (1) only

(3)

1/V

= extension of a wire
= force causing the extension

B. (3) only

C. (1) and (2) only

V
p

= volume occupied by a gas


= pressure exerted by the gas

D. (2) and (3) only

47.

(05)
Two vessels contain hydrogen gas and oxygen gas respectively. Both gases are assumed to be ideal and they have
the same pressure and temperature. Which of the following physical quantities must be the same for the two
gases ?
. The volume of the gas
B. The mass per unit volume of the gas
C. The r.m.s. speed of the gas molecules
D. The number of gas molecules per unit volume

48.

(05)

Pressure
Y

1.5 p
p
X

2T

Absolute Temperature

gas in a vessel of fixed volume leaks gradually. The gas in the vessel changes from state to state along the
path shown in the plot of pressure against absolute temperature. What percentage of the original mass of the
gas leaks out from the vessel in this process ?
. 10%
B. 20%
C. 25%
D. 50%
49.

(06)
The ideal gas equation derived from the kinetic theory can be expressed as p =

1 Nm 2
c .
3 V

In this equation, what does the product Nm represent ?


. the total mass of the gas
B. the mass of one mole of the gas
C. the number of molecules in unit volume of the gas
D. he number of molecules in one mole of the gas

Answer
1.
11.
21.
31.
41.

A
B
C
A
D

2.
12.
22.
32.
42.

E
D
B
E
B

3.
13.
23.
33.
43.

B
B
C
A
D

4.
14.
24.
34.
44.

C
E
D
B
C

5.
15.
25.
35.
45.

B
C
C
C
A

6.
16.
26.
36.
46.

E
E
A
E
C

7.
17.
27.
37.
47.

A
B
A
B
D

8.
18.
28.
38.
48.

D
E
D
D
C

9.
19.
29.
39.
49.

D
C
B
E
A

10.
20.
30.
40.

D
D
A
B

MC Question
Chapter 18 Gases

(MC)18P10

MC Question
Chapter 2 Kinematics

1.

(MC)2P1

(80)

Out of Syllabus

The displacement y of a point P on a plucked string varies with time t as shown in the graph above. Which of the
following correctly shows the variation of the velocity v of the point P with time?
B.
C.
D.
E.

A.

2.

4 m s-1

(82)
3 m s-1

Out of syllabus

Y
6m

Two bodies X and Y are moving with constant velocities in the directions indicated by the arrows. At time t = 0,
they are at the positions shown. At time t.= 2 s, the magnitude of the velocity of Y relative to X is
B. 5 m s-1
C. 6 m s-1
D. 8 m s-1
E..10 m s-1
A. 4 m s-1

3.

(83)
A ball bounces up and down from the floor. Which of the following graphs shows the variation of the velocity v
with time t?
A.
B.
C.
D.
E.
v

4.

(83)
A man is walking due east at 1.00 m s-1 on the deck of a ship steaming north at 1.73 m s-1. In what direction will
the man be walking relative to the surface of the earth ?
B. N 60o E
C. NE
D. N26.9o E
E. N40.9o E
A. N 30o E

5.

(83)

v
t

The velocity v of a particle varies with time t as shown. Which of the following graphs best represents the
variation of the displacement s of the particle with time t ?
A.
B.
C.
D.
E.

MC Question
Chapter 2 Kinematics
6.

(MC)2P2

(86) (88)
A football player is running at a velocity of 3 m s-1 due north. After a violent collision with another player, he is
moving at a velocity of 4 m s-1 due east. Which of the following arrows best represents the direction of his change
of velocity?
A.
B.
C.
D.
E.

out of syllabus
7.

(88)
A boy wished to swim across a river with parallel banks as shown in the diagram below. Assuming the water is
flowing at 0.5 m s-1, and the swimming speed of the boy is 1 m s-1, along which direction should the boy swim if
he wishes (1) to reach the opposite bank in the shortest time; and (2) to take the shortest route to the opposite
bank?
(l)

shortest time
X
X
Y
Y
Z

A.
B.
C.
D.
E.
8.

(2) shortest route


Y
Z
X
Z
Y

water
current

(89)
A ball bounces up and down on the ground in the vertical direction. Which of the following graphs best describes
the variation of its acceleration a with time t ?
A.

B.

C.

t
0

0
t

D.

t
0

E.

t
0

9.

(89)
Two ships X and Y travel with equal speeds. X moves due North and Y moves due East. Which of the following
best gives the direction of the velocity of X relative to Y?
A.
B.
C.
D.
E.

10.

(90)
When a man is running due north, he feels that the wind is blowing towards him from the east. What is the actual
direction of the wind ?
A. from the east
B. from north-east
C. from south-east
D. from south-west
E. from north-west

Out of syllabus

MC Question
Chapter 2 Kinematics

11.

(MC)2P3

(98)
The velocities of the longitudinal and transverse waves produced by an earthquake are 5.0 km s-1 and 9.0 km s-1
respectively. A seismograph records the arrival of the transverse waves 64 s before that of the longitudinal waves.
How far away was the earthquake ?
A. 256 km
B. 320 km
C. 576 km
D. 659 km
E. 720 km

12. (01)

An object accelerates uniformly along a straight line from X to Z. It passes X and Z with speeds u and v
respectively. What is its speed when it passes Y, which is the mid-point of XZ?
u2  v2
2

A.
B.

uv
2uv
C.
uv
uv
D.
2
E. It cannot be found as the distance between X and Z is not known.

13. (02)
Two small identical objects P and Q are released from rest from the top of a building 80 m above the ground. Q is
released 1 s after P. Neglecting air resistance, what is the maximum vertical separation between P and Q in the
air?
A.
B.
C.
D.

5m
10 m
35 m
45 m

Answer
1.B

2.B

3.B

4.A

5.B

6.A

7.D

8.C

9.C

10.C 11.E

12.A 13.C

MC Question
Chapter 4,5 Newtonian Mechanics
1.

(MC)4P1

(80)
A body is subjected to an applied driving force F and a constant resistive force. Which of the graphs below
represents the variation of the acceleration a of the body with the applied driving force F ?
A.
B.
C.
D.
E. a
a
a
a
a

F
0

2.

(82)
A solid of mass m starts from rest and travels for a given time under the action of a given force. The speed which
it acquires is proportional to
1
1
C. m
D.
E. m2
A. m
B.
m
m

3.

(82)
A builder using a pulley system to lift a bucket of cement of weight 150 N exerts a steady force F and pulls 30 m
of rope through the system in order to raise the bucket 10 m. The friction in the system is small but NOT
negligible. The value of the force F is most probably
A. less than 50 N.
B. exactly equal to 50 N.
C. between 50 N and 150 N.
D. between 150 N and 450 N.
E.
more than 450 N.

4.

(83) Out of syllabus


A spaceship burns fuel and moves with constant acceleration in a straight line. Which of the graphs below best
represents the variation of its momentum p with time t?
p
A. p
B. p
C. p
D.
E. p

5.

(83)
A ball bounces up and down from the floor. Which of the following graphs shows the variation of the velocity v
with time t?
A.
B.
C.
D.
E.
v

6.

(84)

Out of syllabus

A horizontal force P is applied to a wooden block at lest on a rough horizontal table. P is increased uniformly
from zero. Which of the following graphs best describes the variation of the friction F acting on the block with
P?
F
F
F
F
A.
B.
C.
D.
E. F

P
0

P
0

P
0

MC Question
Chapter 4,5 Newtonian Mechanics

(MC)4P2

7.

(85)
A lift of mass M carries a man of mass m. When the lift is being hauled upward by a rope, the lift rises with an
acceleration a. The reaction between the man and the floor of the lift is
A. mg
B. m(g - a)
C. m (g + a)
D. m(g + a) - Ma
E. m(g + a) + Ma

8.

(85)
Two bodies P and Q are connected by a light string A. The weights of P and Q
are 4 N and 10 N respectively. P is connected to the roof by another light string
B. If string B is cut and the bodies are allowed to fall, the net force acting on P
during free fall is
A. 0 N.

B. 4 N

C. 6 N

D. 10 N

B
P
A

E. 14 N

Q
9.

(86)

B
M
10m
A

30o

30o

A small block M of mass 1 kg is transported across a small hill along the road ABC by an applied force F which is
always parallel to the road. The speed of M is kept constant throughout the journey and the kinetic friction
between the block and the road is 2.60 N. The total work done by F in transporting M from A to C is
A. Zero.
10.

11.

B. 104 J.

C. 152 J.

D. 200 J.

E. 304 J.

(86)
An object of mass 3 kg is placed on a smooth plane inclined at 30o to the
horizontal. It is connected by a light string passing over a frictionless pulley to
another object, of mass 2 kg, as shown below. When the system is released the
tension in the string will be
A. 18 N.
B. 20 N.
C. 24 N.
D. 25 N.
E. 30 N.
(87)
An object of mass 3 kg is tied to another object of mass 2 kg with a string passing
over a fixed smooth pulley. The weight of the pulley is negligible. When the
objects move under the action of gravity, the vertical upward force acting on the
axle of the pulley is
A. zero.
B. 10 N.
C. 24 N.
D. 48 N.
E. 50 N.

3 kg
2 kg
30o

axle

3 kg

2 kg

12.

(87)
Steel ball bearings, each of mass m, are fired at the rate of n ball bearings per second towards a fixed vertical steel
block with a horizontal speed v. They rebound from the block with the same speed. The average force exerted on
the block is
A. zero.
B. m n v.
C. 2 m n v.
D. m g n v.
E. 2 m g n v.

13.

(88)
A smooth block of mass 2 kg slides down a wedge. The wedge, of mass 10 kg, is placed on a horizontal table, and
its inclined plane makes an angle of 30o with the horizontal. If the wedge remains stationary all the time, the
normal reaction of the table acting on the wedge is
A. 15 N.
B. 85 N.
C. 100 N.
D. 115 N.
E. 120 N.

MC Question
Chapter 4,5 Newtonian Mechanics

(MC)4P3

(88) Out of syllabus


A force F is applied to an initially stationary particle from time t = 0. F varies sinusoidally with time t as shown in
the following graph:
F

14.

t
0

Which of the following graphs best represents the variation of the subsequent velocity v of the particle with t ?
A.
B.
C.
D. v
E. v
v
v
v
t
0

15.

Out of syllabus
(89)
Two objects of weights 2 N and 3 N are suspended from a fixed point by two
identical light springs A and B as shown in the diagram. The force constants of
the springs are both 1 N cm-1. What are the extensions of springs A and B ?

spring A
2N

A.
B.
C.
D.
E.

Extension of spring B
3 cm
2 cm
2 cm
5 cm
3 cm

Extension of spring A
5 cm
5 cm
3 cm
3 cm
2 cm

spring B
3N

16.

(89)

X
30o

30o

In the figure above, X and Y are blocks of mass 1 kg and 2 kg respectively. S is a spring balance of negligible
mass and P is a smooth pulley fixed at the top of two smooth inclined planes. What is the reading of S when X is
held stationary ?
A. 5 N
B. 10 N
C. 15 N
D. 20 N
E. 30 N

17.

(89)
A man weighs an object with a spring balance in a lift. Before the lift moves the scale reads 50 N. The lift goes
down and then stops. The reading on the scale is
A. 50 N throughout the journey.
B. more than 50 N when the lift starts, and remains steady until it comes to rest.
C. less than 50 N when the lift starts, and remains steady until it comes to rest.
D. more than 50 N as the lift starts, and less than 50 N as it comes to rest.
E.
less than 50 N as the lift starts, and more than 50 N as it comes to rest.

18.

(91)
An object of mass m slides with constant acceleration a down a plane making an angle T with the horizontal. The
frictional force acting on the object is
A. mg ma
B. mg ma sinT
C. Mg sinT ma
D. (mg ma) sin T
E. (mg + ma) sinT

MC Question
Chapter 4,5 Newtonian Mechanics

(MC)4P4

19.

(92)
A man in a lift measures his weight with a compression balance and discovers that his weight is reduced by 10%.
The lift is probably
(1) moving upwards and accelerating at g/10.
(2) moving upwards and decelerating at g/10.
(3) moving downwards and accelerating at g/l0.
(4) moving downwards and decelerating at g/10.
(5) moving downwards with uniform velocity.
A. (l) & (3) only
B. (1) & (4) only
C. (2) & (3) only
D. (2) & (4) only
E. (5) only

20.

(92)

Three blocks A, B and C of masses m, 2m and 2m respectively are placed on a smooth horizontal ground as shown
in the figure above. A constant horizontal force is applied to block A so that the three blocks move with the same
acceleration towards the right. What is the resultant force acting on block B ?
A. F/5
B. F/3
C. 2F/5
D. 3F/5
E. F
21.

(92)

A simple pendulum is swinging in a vertical plane. When it is at the position shown, which of the following
diagrams best represents the forces acting on the bob? Neglect air friction.
A.
B.
C.
D.
E.

22.

(93)

F1

F2
The resultant of two forces F1 and F2 acting at a point has a minimum value of 1 N and a maximum value of 7 N.
When the two forces act at right angles to each other, the magnitude of their resultant is
A. 3 N

23.

B. 5 N

C. 6 N

D.

50 N

E. 8 N

(93)
A box moves at a uniform velocity of 2 m s-1 on a frictionless horizontal surface. Sand falls into the box with
negligible speed at a rate of 90 kg per minute. What horizontal force is required to keep the box moving uniformly
at 2m s-1 ?
A. 0 N
B. 3 N
C. 6 N
D. 90 N
E. 180 N

MC Question
Chapter 4,5 Newtonian Mechanics
24.

(MC)4P5

(94)
A man of weight W stands on a compression balance placed inside a lift. The velocity-time graph of the lift is
shown below. (The upward direction is taken to be positive.)
velocity

t1

time
t2

Which of the following graphs best shows the variation of the reading on the balance, R, with time, t ?
A.
B.
C.
D.
E.
R
W

25.

(94)

t1

t
t2

t1

t
t2

t1

t
t2

R
W

t1

t
t2

t1

t
t2

P
rod

motor

propeller

The above figure shows a propeller-motor system connected by a light, rigid rod to a fixed point P on the ceiling.
The system remains stationary when the motor is on. Which of the following diagrams correctly represents the
forces acting on the propeller-motor system ?
A.
B.
C.
D.
E.

26.

(94)
When an object falls freely, its total energy
A. increases during the fall.
B. decreases during the fall.
C. remains constant during the fall.
D. is zero at the beginning of the fall.
E.
is at a maximum at the end of the fall.

27.

(95)
When given a slight push, a toy car moves freely with constant velocity down a plane inclined at 20o to the
horizontal. If the mass of the car is 0.5 kg, find the force parallel to the inclined plane for pulling the car up the
plane with constant velocity.
A. 1.7 N
B. 3.4 N
C. 4.7 N
D. 6.7 N
E. 9.4 N

28.

(95)
Two small spheres A and B of masses 1 kg and 2 kg respectively are released from
rest at heights 4h and h above the ground as shown. Which of the following
statements is/are correct ? (Assume air resistance is negligible.)
4h
(1) The acceleration of sphere A doubles that of sphere B.
B
(2) The time taken for sphere A to reach the ground is double that of
sphere B.
h
(3) The kinetic energy of sphere A when reaching the ground is double
that of sphere B.
A. (1) only
B. (3) only
C. (1) & (2) only
D. (2) & (3) only
E. (1), (2) & (3)

MC Question
Chapter 4,5 Newtonian Mechanics
29.

(MC)4P6

(96)
Out of syllabus
The figure shows two blocks A and B, each of mass m, connected by two
light springs to a fixed support. Each spring has a force constant k. What is
the total extension of the system when it is at static equilibrium ?
A. mg/(2k)
B. mg/k
C. 3mg/(2k)
D. 2mg/k
E. 3mg/k

30.

(96)
Which of the following pairs of quantities of a moving object must be in the same direction?
(1) acceleration and change in momentum
(2) displacement and velocity
(3) velocity and acceleration
A. (1) only
B. (3) only
C. (1) & (2) only
D. (2) & (3) only
E. (1),(2) & (3)

31.

(97)
A

Two wooden blocks A and B are connected by a string which passes over a smooth, fixed pulley as shown. The
maximum friction between any two surfaces is 2 N. If a horizontal force F is applied to block B, find its minimum
value for moving B.
A. 2 N
B. 4 N
C. 6 N
D. 8 N
E. 10 N

32.

33.

(98)
Two identical light springs are connected with two masses of 1.0 kg and 0.5 kg as
shown. Which of the following statements is/are correct ?
(1) The tension in the upper spring is double that in the lower spring.
(2) The force acting on the ceiling by the whole system is 15 N.
(3) The tension in the upper spring will remain unchanged if the two masses
are exchanged in position.
A. (1) only
B. (3) only
C. (1) & (2) only
D. (2) & (3) only
E. (1), (2) & (3)

1.0 kg

0.5 kg

(98)
P

Two blocks A and B of mass ratio 1:2 are placed on a horizontal frictionless surface as shown above P and Q are
horizontal forces acting on A and B respectively (with P > Q ) so that the blocks move to the right with constant
acceleration. Find the force acting on B by A.
PQ
PQ
2( P  Q )
2P  Q
P  2Q
B.
C.
D.
E.
A.
3
3
3
3
3

34.

(98)
The initial momentum of an object is 2 N s in the direction S 45oE and its final momentum is 1 N s due east. If
this change takes place in 0.5 s, the average force acting on the object during the change is
final
A. 1 N due north
North
momentum
B. 1 N due south
45o
C. 0.5 N due north
D. 2 N due south
initial
E.
2 N due north
momentum

MC Question
Chapter 4,5 Newtonian Mechanics
35.

(98)

(MC)4P7

F/N
4

t/s

0
5

The graph shows the time variation of the net force, F, acting on an object of mass 1 kg. The object is initially at
rest and its subsequent motion is along a straight line. Which of the following statements is/are correct ?
(1) The object is accelerating in the first 5 seconds.
(2) The maximum speed of the object is less than 10 m s-1.
(3) The final speed of the object is zero.
A. (1) only
B. (3) only
C. (1) & (2) only
D. (2) & (3) only
E. (1), (2) & (3 )

(99)
30 cm

36.

F
horizontal
surface

40 cm

A block of mass 5 kg is placed on the inclined surface of the wedge shown above. All contact surfaces are
assumed to be smooth. What is the magnitude of the horizontal force F exerted on the wedge so that it remains
stationary while the block is sliding down the inclined surface with acceleration ?
A. 0 N
B. 18 N
C. 24 N
D. 30 N
E. 32 N
37.

(99)

Out of syllabus

F
A

Two objects A and B of equal mass m are connected by two identical light springs and are placed on a horizontal
smooth surface. A horizontal force F is applied to B so that the system is in equilibrium. If the applied force F is
suddenly removed, what are the magnitudes of the acceleration of each object at the instant when force F is
removed ?
Acceleration of B
Acceleration of A
F
A.
zero
m
B.
zero
zero
F
F
C.
2m
m
F
D.
zero
m
F
F
E.
m
m
E/J

38.

(99)
1
v2 / m2 s-2
0

The above graph shows the variation of kinetic energy E with the square of velocity v of a moving mass m. What
is the momentum of the mass when it is moving at a speed of 2 m s-1?
A. 1 N s
B. 2 N s
C. 4 N s
D. 8 N s
E. 16 N s

MC Question
Chapter 4,5 Newtonian Mechanics
39.

(00)

(MC)4P8

A
B

Two books A and B are placed on a horizontal table surface as shown. A horizontal force F is applied to A but the
system remains stationary. Which of the following statements is/are correct ?
(1)
(2)
(3)

The frictional force acting on B by the table surface is greater than F


The frictional force acting on A by B is towards the left.
The system would remain stationary if F is applied to B instead.

A. (1) only

B. (3) only

C. (1) and (2) only

D. (2) and (3) only

E. (1), (2) and (3)

40.

(00)
A student holds one end of a string to which a block of mass 8 kg is tied at the other end. He raises the block with
an acceleration by pulling the string in an upward direction. If the maximum tension that the string can withstand
is 120 N, find the maximum acceleration of the block before the string breaks.
A. 5.0 m s-2
B. 7.5 m s-2
C. 10.0 m s-2
D. 12.5 m s-2
E.
15.0 m s-2

41.

(00)
A trolley of mass 0.5 kg moves with a certain acceleration down a runway which is inclined to the horizontal at
15o. If the angle of inclination is increased to 20o the acceleration of the trolley would be doubled. Find the
average frictional force, assuming the same in both cases, acting on the trolley.
A. 0.72 N
B. 0.80 N
C. 0.88 N
D. 0.96 N
E. 1.04 N

42.

(00)
Two objects A and B of masses m and 2m respectively are initially at rest on a smooth, horizontal surface. If each
of them is acted upon by the same force for the same period of time, the ratio of the gain in kinetic energy of A to
that of B is
A. 2:1
B. 1:2
C. 1:1
D. 1:4
E. 4:1

43.

(00) Out of syllabus


A light spring is fixed to the bottom of a vertical tube. A ball is released from rest at a
height h above the upper end of the spring as shown. After rebounding several times
the ball eventually comes to rest and stays on top of the spring. Assume all contact
surfaces are smooth and the spring obeys Hooke's law throughout. Which of the
following statements is/are correct ?
The compression of the spring is proportional to the mass of the ball.
The compression of the spring is independent of the height h.
The gravitational potential energy lost by the ball is equal to the strain energy
stored in the spring.
A. (1) only
B. (3) only
C. (1) and (2) only
D. (2) and (3) only

(1)
(2)
(3)

E. (1), (2) and (3)

1.A 2.B 3.C 4.D 5.B 6.D 7.C 8.B 9.B 10.A 11.D 12.C 13.D 14.D 15.A
16.B 17.E 18.C 19.C 20.C 21.A 22.B 23.B 24.D 25.D 26.C 27.B 28.D 29.E 30.A
31.C 32.D 33.D 34.E 35.C 36.C 37.A 38.C 39.D 40.A 41.C 42.A 43.C

MC Question
Chapter 6 Linear Momentum
1.

(80)

T
v

(MC)6P1

A particle of mass m strikes a barrier with speed v and rebounds with the same speed v. Which of the following
statements is/are correct ?
(1) The angle I must be equal to the angle T.
(2) The change in the component of momentum pcrpendicular to the barrier is 2 m v sin T.
(3) The change in the component of momentum parallel to the barrier is zero.
A. (1) only
B. (3) only
C. (1) & (2) only
D. (2) & (3) only
E. .(1), (2) & (3)

2.

(81)
Before the start of a race, the momentum of each competitor is less than his momentum during the race. Which of
the following statements is/are correct ?
(1) This situation violates Newton's law of conservation of momentum.
(2) The law of conservation of momentum applies only to collisions between two objects.
(3) A force acts on each competitor to increase his momentum as he starts to race.
A. (1) only
B. (3) only
C. (1) & (2) only
D. (2) & (3) only
E. (1), (2) & (3)

3.

(82)
Two objects of masses m and 4m move towards each other along a straight line with kinetic energies E and 4E
respectively. The total linear momentum of both masses taken together is
A. 3 2mE

4.

B. 4 2mE

C. 5 2mE

D. 15

2mE

E.17 2mE

(83)
A sphere X of mass m, travelling with speed u, makes a head-on collision with a similar sphere Y which is at rest.
After the collision the velocities of X and Y are v1 and v2 respectively. Which one of the following is a possible
pair of values for v1 and v2 ?
v1
v2
A.
-u
2u
B.
u/4
3 u /4
C.
3u/4
u/4
D.

u/ 2

E.

u/2

u/ 2
3u / 2

5.

(84)
In an inelastic collision between two bodies, and in the absence of external forces,
A. kinetic energy and momentum are both conserved.
B. kinetic energy is not conserved but momentum is conserved.
C. kinetic energy is conserved but momentum is not conserved.
D. neither kinetic energy nor momentum is conserved.
E.
kinetic energy is not conserved and the momentum changes direction.

6.

(84)
An alpha particle collides with a stationary helium nucleus ( 24 He ) in a cloud chamber. Which of the following
diagram represents the most probable set of tracks ?
A. D
B.
C.
D.
E.
D
D
D
4
2 He

4
2 He

4
2 He

D
D

4
2 He

D
4
2 He

MC Question
Chapter 6 Linear Momentum

(MC)6P2

7.

(86)
A sphere X moving with velocity a on a smooth horizontal plane makes a head-on collision with another sphere Y
of the same mass which is initially at rest. If the collision is perfectly elastic, which of the following statements
is/are correct ?
(1) Kinetic energy is conserved in this collision.
(2) Linear momentum is conserved in this collision.
(3) X and Y stick together and move off with the same velocity after the collision.
A. (1) only
B. (3) only
C. (1) & (2) only
D. (2) & (3) only
E. (1), (2) & (3)

8.

(91)

A gas atom of mass m moving with a uniform speed v makes an elastic collision with the wall of the container as
shown in the diagram. What is the magnitude of the change in the momentum of the gas atom ?
C. m v cosT
D. 2 m v sinT
E. 2 m v cosT
A. 2 m v
B. m v sin T

9.

(94)
A softball of mass 0.5 kg flies horizontally with a speed of 20 m s-1 towards a player. After being hit by the bat, it
flies away at 30 m s-1 perpendicular to its original direction. Find the magnitude of the impulse acting on the
softball.
B. 18 kg m s-1
C. 20 kg m s-1
D. 25 kg m s-1
E. 36 kg m s-1
A. 5 kg m s-1

10.

(95)

A trolley travels with constant velocity to the right on horizontal ground and collides with a light helical spring
attached to a wall fixed to the ground (earth) as shown. At the instant that the trolley comes momentarily to rest
during collision, what has happened to the initial momentum of the trolley ?
A. It has been transferred to the earth.
B. It has been stored in the spring.
C. It has been dissipated as sound and heat.
D. It has been destroyed by the friction due to the ground.
E.
It has been lost because the collision was not perfectly elastic.
11.

(96)

u
m

m 2m

X, Y and Z are three spheres of the same size but with masses m, m and 2m respectively, lying on a smooth
horizontal track with Y and Z in contact as shown. X is moving to the right with velocity u and makes a head-on
collision with Y. If all collisions are perfectly elastic, which of the following gives the possible velocities of the
three spheres after all collisions ? (Take to the right as positive.)
X
Y
Z
A.
0
0
u/2
B.
0
u/3
u/3
C.
-u/3
0
2u/3
D.
0
-u/3
2u/3
E.
u/4
u/4
u/4

MC Question
Chapter 6 Linear Momentum

(MC)6P3

12.

(97)
A ball of mass m is projected vertically downwards with speed v from a certain height and rebounds from the
ground back to the same height. Which of the following statements is/are correct ? (Neglect air resistance.)
(1) The collision between the ball and the ground is not perfectly elastic.
1
(2) The loss in energy of the ball in the collision is
mv 2 .
2
(3) If the ball is projected vertically upwards from the same height with the same speed, it would rebound
to a greater height.
A. (1) only
B. (3) only
C. (1) & (2) only
D. (2) & (3) only
E. (1), (2) & (3)

13.

(97)
A nucleus originally at rest splits into two fragments of unequal mass. The fragment with smaller mass has a
larger
(1) momentum.
(2) speed.
(3) kinetic energy.
A. (1) only

14.

B. (3) only

C. (1) & (2) only

D. (2) & (3) only

E. (1), (2) & (3)

(99)

The above figure shows an open wagon moving with negligible resistance in vertically falling rain. An
appreciable amount of rain falls into the wagon and accumulates there. What are the effects of the accumulating
rain on the speed, momentum and kinetic energy of the wagon? (Ignore the effects of the rain drops hitting the
front of the wagon.)
momentum
kinetic energy
speed
A. decreased
unchanged
decreased
B. decreased
unchanged
unchanged
C. decreased
decreased
decreased
D. unchanged
unchanged
unchanged
E.
unchanged
increased
increased

15.

(02)

P, Q and R are three identical spheres moving with the same speed along a smooth horizontal track. They undergo
head-on elastic collisions with spheres X, Y and Z respectively which are initially at rest. After collision, P
continues to move in its original direction, Q becomes stationary while R reverses its motion. After collision,
which sphere (X, Y or Z) has the greatest magnitude of
(1) momentum
(2) kinetic energy?

A.
B.
C.
D.

(1)
Y
Y
Z
Z

(2)
Z
Y
X
Y

MC Question
Chapter 6 Linear Momentum
16.

17.

(02)
The impulse experienced by an object is equal to its change in
A. momentum.
B. kinetic energy.
C. velocity.

(04)

(MC)6P4

D. acceleration.

Out of syllabus
m
A

Two spheres A and B of the same size, which have masses m and M respectively. A moves horizontally with a
speed u and makes a head-on elastic collision with B, which rests on a smooth horizontal surface as shown. Referring
to each of the following descriptions about B, how is m compared with M?
(1)
(2)
(3)

B attains the greatest kinetic energy after collision


B attains the greatest magnitude of momentum after collision
B attains the greatest speed after collision

A.
B.
C.
D.

(1)
M=m
M=m
M=m
M << m

1. E
11. C

(2)
M=m
M << m
M >> m
M >> m

2. B
12. C

3. A
13. D

(3)
M << m
M >> m
M << m
M << m

4. B
14. A

5. B
15.D

6. D
16.A

7. C
17. C

8. D

9. B

10. A

MC Question
Chapter 3 Projectile motion

(MC)3P1

1.

(80)
A metal sphere is released from rest and allowed to fall vertically under gravity through oil. Which of the
following statements is/are correct?
(1) The acceleration decreases as the speed increases.
(2) At the terminal speed, the upward forces on the sphere have their maximum values.
(3) If the sphere were dropped in air instead of in oil, there would be no terminal speed.
A. (1) only
B. (3) only
C. (1) & (2) only
D. (2) & (3) only
E. (1), (2) & (3)

2.

(81)
Projectiles X and Y are launched simultaneously from the top of a cliff. Suppose X is launched horizontally with
speed 20 m s-1, and Y is launched at an angle of 60o above the horizontal with speed 40 m s-1. (The motion of both
X and Y is in the same plane.) Which of the following statements is/are correct ?
(1) X and Y travel equal vertical distances in equal times.
(2) X and Y travel equal horizontal distances in equal times.
(3) X and Y never meet.
A. (1) only
B. (3) only
C. (1) & (2) only
D. (2) & (3) only
E. (1), (2) & (3)

3.

(83)

X
h

2m
Z
3m

A particle is released from X and slips down a smooth curve to Y, at the edge of a table 2 m high, where it travels
horizontally. It then leaves the table and travels freely under gravity, until it hits the ground at Z, at a horizontal
distance of 3 m from Y. The vertical distance, h, of X above Y is
A. 9/8 m
B. 4/3 m
C. 2 m
D. 3 m
E. 4 m
4.

(84)
An object is thrown horizontally from the top of a cliff at a speed of 20 m s-1. What will be the speed of the object
after 3 s ?
A. 20m s-1
B. 25m s-1
C. 30m s-1
D. 36m s-1
E. 50m s-1

5.

(84)
The velocity v of a small steel ball falling in a viscous liquid inside a long vertical tube varies with time t, as
v
follows:

t
0
Which of the following graphs best represents the variation of the viscous force F acting on the body?
A.
B. F
C. F
D. F
E. F
F

6.

t
0

t
0

(87)
A
A sphere is projected downwards from A with a speed of 10 m s-1 at
10 m
an angle of 30o to the horizontal. The sphere rebounds from the
ground, first at B and then at C. If the collisions are perfectly elastic,
the horizontal distances BC is equal to
A. 8.7 m.
B. 10.0 m.
C. 26.0 m.
D. 43.5 m.
E.100.0 m.

MC Question
Chapter 3 Projectile motion

(MC)3P2

7.

(87)
A parachutist of mass m falls in air under the influence of gravity. The air resistance is equal to bv, where v is his
speed and b is a constant. After falling a height s from rest, he reaches a terminal speed u. His kinetic energy at
that instant is
D. mgs + (m3 g2)/(2b2)
E. m3 g2 / (2b2)
A. mgs.
B. mgs bus.
C. mgs (m3 g2)/(2b2)

8.

(90)
A hunter aims his gun at a target which is at rest at point M, and his gun
makes an angle T with the horizontal. Exactly as the gun is fired the
target drops from M with zero initial velocity. If t he bullet is to strike the
target, the angle T depends on
(1) u, the initial speed of the bullet.
(2) h, the vertical height of the target above the level of the gun.
(3) d, the horizontal distance of the gun from the target.
A. (1) only
B. (3) only
C. (1) & (2) only
D. (2) & (3) only
E. (1), (2) & (3)

T
d

gun

9.

(91)
A spacecraft of mass 4.0 u 104 kg was travelling on its way to the moon with the rocket motors shut down. At the
instant when it was travelling at 1500 m s-1, the rocket motors were turned on for 5 seconds to make a course
correction. If the rocket gave a thrust of 1.0 u 105 N at right angles to the direction of travel, throughout what
angle would the flight path of the spacecraft be turned ?
A. 1.6 u 10-3 rad
B. 5.7 u 10-3 rad
C. 8.3 u 10-3 rad
-2
-2
D. 1.6 u 10 rad
E. 8.3 u 10 rad

10.

(92)
A ball-bearing is dropped into viscous oil. Which of the following correctly describes its motion before the
terminal speed is reached ?
Velocity Acceleration
A. decreases decreases
B. decreases increases
C. increases constant
D. increases decreases
E.
increases increases

11.

(93)
Two identical coins P and Q are placed at the edge of a table. At the same
instant, P is pushed slightly and falls vertically to the ground while Q is
projected horizontally and reaches the ground through a parabolic path (as
shown). Which of the following statements is/are correct? (Neglect air
resistance)
(1) P and Q reach the ground at the same time.
(2) P and Q have the same acceleration.
(3) P and Q have the same vertical speed on reaching the ground.
A. (1) only
B. (3) only
C. (1) & (2) only
D. (2) & (3) only

12.

Q
P

E.

(1), (2) & (3)

(94)
Three bombs are released from a bomber flying horizontally with constant velocity to the right. They are released
from rest (relative to the bomber) one by one at one-second intervals. Neglecting air resistance, which of the
following diagrams correctly shows the positions of the bomber and the three bombs at a certain instant?
A.
B.
C.
D.
E.

MC Question
Chapter 3 Projectile motion
13.

(96)
An object is thrown vertically upward and experiences an air resistance opposing its motion with magnitude
proportional to its speed. Which of the following graphs best represents the variation of the acceleration, a, of the
object with time, t, starting from the moment when the object leaves one's hand up to the time when it returns the
ground ? (Take downward as positive.)
a
A. a
B.
C. a
D. a
E. a

0
-g

-g

g
t

0
-g

(96)
A particle is projected horizontally from a table with an initial speed u and attains a speed v Just before hitting the
ground. What is the time of flight of the particle? (Neglect air resistance.)
A.

D.

15.

-g

-g

14.

(MC)3P3

v
g

B.
v2 u2
g

vu
g

C.

v2 u2
2g

E. It cannot be found as the vertical distance fallen is not known.

(98)
A small object is thrown horizontally towards a vertical wall 1.2 m away. It hits the wall 0.8 m below its initial
horizontal level. At what speed does the object hit the wall ? (Neglect air resistance.)
wall

1.2 m
0.8 m

A.2 m s-1
16.

B. 3 m s-1

C. 4 m s-1

D. 5 m s-1

(01)
A stone is projected at an angle of 45q to the horizontal with an initial kinetic energy E.
when the stones is halfway up, its kinetic energy is
A.
B.
C.
D.
E.

E
.
4
E
.
2
3E
.
4
E
.
2

E.

E. 7 m s-1

Neglecting air resistance,

MC Question
Chapter 3 Projectile motion
17.

(MC)3P4

(03)
The figure shows part of the stroboscopic picture of a particle initially projected horizontally into the air.
of each square of the grid is 5 cm long. Estimate the frequency of the strobe lamp used.
(Neglect air resistance.)

The side

5 cm
5 cm
A.
B.
C.
D.

5.8 Hz
7.1 Hz
10.0 Hz
12.5 Hz

18. (04)
The figure shows the barrel of a gun that aims directly at a point P 40 m from the muzzle of the gun. The barrel
makes an angle T with the vertical.

If the speed of the bullet is 50 ms-1 when it leaves the gun, calculate the separation between the bullet and point P
when the bullet is vertically below P. (Neglect air resistance.)
A.
B.
C.
D.

3.2 m
4.8 m
7.8 m
It cannot be found as the value of T is not known.

Answer
1.C
11.E

2.D
12.D

3.A
13.C

4.D
14.D

5.A
15.D

6.C
16.C

7.E
17.C

8.D
18.A

9.C

10.D

MC Question
Chapter 7 Circular motion

(MC)7P1

1.

(80)
Racing tracks for cars are banked on the corners at an angle to the horizontal. Which of the following is a possible
reason for this design feature ?
(1) To reduce the frictional force between the car and the track necessary to prevent skidding.
(2) To reduce the radius of curvature of the path which a car travelling at a given speed can safely follow.
(3) To increase the component of the weight of the car towards the centre of its path.
A. (1) only
B. (3) only
C. (1) & (2) only
D. (2) & (3) only
E. (1), (2) & (3)

2.

(82)
A stone of weight W tied to a piece of string is swung in a vertical circle. At the topmost point of is path, the
tension in the string is T and the centripetal force is F. Which of the following statements is true?
A. F = W + T.
B. F = W T.
C. The net force acting downwards on the stone is F+ T+ W.
D. The net force acting downwards on the stone is F T + W.
E.
The net force acting downwards on the stone is F T W.

3.

(83)

III
I

IV

II

A boy is whirling a stone, tied to a piece of string, in a vertical circle as shown above. The string suddenly breaks
at P. Which of the paths (I - V) represents a possible path for the stone from just before the string breaks until the
stone hits the ground ?
A. I
B. II
C. III
D. IV
E. V
4.

(84)

The diagrams represents the rear view of a motorcar moving, away from an observer, on a level road at a constant
speed around a bend of which the centre of curvature is at P. Which of the arrows best represents the direction of
the resultant of the forces exerted by the road on the car ?
A.
B.
C.
D.
E.

5.

(86)

P
h

A particle is placed at a height h on a smooth loop-the-loop track, as shown. The radius of the loop is r. P is now
released from rest. If P is to complete the loop, the minimum value of h should be
A. 2r.
B. 2.5r.
C. 3r.
D. 4r.
E. 5r.

MC Question
Chapter 7 Circular motion
6.

(MC)7P2

(88)

A car of mass m travels into a region where the track is an arc of a vertical circle of radius r. At the bottom of this
arc, the car travels at speed v. At this position the vertical force exerted upwards by the track on the car is
A. mv2/r.
B. mg.
C. mv2/r mg.
D. mg mv2/r
E. mg + mv2/r
7.

(89)

A smooth conical container rotates about the axis AB as shown. A marble remains at rest relative to the container
at a radial distance r from the axis. If the velocity of the marble is v, then v2 is equal to
A. g r sin 30o
B. g r tan 30o
C. g r /tan 30o
D. g r cos 30o
E. g r / cos 30o
8.

(90) Out of syllabus


A spring of unstretched length 10.0 cm, has one end fixed to the ceiling. A mass is suspended at the other end, and
the extension so produced is 3.0 cm. When the mass is set to rotate in a horizontal circular path the length of the
spring is 16.0 cm. The angle between the spring and the vertical is
A. 15o
B. 30o
C. 45o
D. 60o
E. 75o

9.

(91)

Out of syllabus

The diagram shows part of the route of a roller-coaster in an amusement park. The cart descends from H,
completes a circular loop A and moves to B. If the cart of passengers is to complete the central circular track
safely, what is the minimum velocity of the cart at the bottom of the circular track A? (Assume that there is no
friction between the cart and the track.)
A. 10 m s-1
B. 20 m s-1
C. 22.4 m s-1
D. 24.5 m s-1
E. 30 m s-1
10.

(93)
A

The figure shows a small heavy bob P attached to a fixed point A on the ceiling by a light inextensible string. The
bob is pulled aside with the string taut and then released from rest. Which of the following descriptions is/are
true?
(1) When moving towards the lowest point of its path, the angular speed of the bob is increasing.
(2) The centripetal acceleration of the bob is constant.
(3) When the bob is at the lowest point, the tension in the string equals the centripetal force.
A. (1) only
B. (3) only
C. (1) & (2) only
D. (2) & (3) only
E. (1), (2) & (3)

MC Question
Chapter 7 Circular motion
11.

12.

(MC)7P3

(94)
A student whirls a small bucket of water in a vertical circle of radius 0.6 m. For no spilling, what is the minimum
speed of the bucket at the highest point of its path ?
A. 2.45 m s-1
B. 3.46 m s-1
C. 4.08 m s-1
D. 4.90 m s-1
E. 5.77 m s-1

Out of syllabus
(95)
A small object with a mass of 0.05 kg is released from rest at the rim of a heavy, smooth semi-spherical bowl of
radius 10 cm. Find the force acting on the object by the bowl when it passes the bottom of the bowl.
A. 0.5 N
B. 1.0 N
C. 1.5 N
D. 2.0 N
E. 2.5 N

13.

(95)
An aircraft flies along a horizontal circle of radius 10 km with a constant speed of 155 m s-1. Calculate the angle
between its wings and the horizontal.
A. 11.5o
B. 12.0o
C. 12.5o
D. 13.0o
E. 13.5o

14.

(96)

Out of syllabus

P
0.5m

A small object P of mass 0.3 kg is attached to one end of a light, rigid rod of length 0.5 m, which is free to rotate
about the other end O as shown, in the above figure. The object is swung to rotate in a vertical circle so that it
attains a speed of 2 m s-1 at its topmost position. What is the force exerted on one end of the rod at this instant?
A. a compressive force of 0.6 N
B. a tensional force of 0.6 N
C. a tensional force of 2.4 N
D. a tensional force of 5.4 N
E.
a compressive force of 5.4 N
centre of mass

15.

(97)
8m

20o

The figure shows a car moving round a comer with a radius of 8 m on a banked road of inclination 20o At what
speed would there be no friction acting on the car along OA ?
A. 5.0 m s-1
B. 5.2 m s-1
C. 5.4 m s-1
D. 5.6 m s-1
E. 5.8 m s-1
16.

(97)

Out of syllabus
A
h

A ball bearing is released from rest at a height h on a smooth track and completes the circular loop of the track. If
R is the reaction acting on the ball bearing at the highest point A of the loop, which of the following graphs
correctly shows the variation of R with h ?
A.
B.
C.
D.
E.
R
R
R
R
R

h
0

h
0

h
0

MC Question
Chapter 7 Circular motion
17.

(99)

Out of syllabus

(MC)7P4

v
P
A small ball-bearing is projected with velocity v from the lowest position P of a vertical circular track which is
not smooth. The ball-bearing starts to leave the track at Q. Which of the following diagrams best represents all the
forces acting on the ball-bearing at Q?
A.
B.
C.
D.
E.

(00)
A car of mass m is moving with speed v on a banked road along a circular path of horizontal radius r. The angle of
inclination of the road is T. If the centripetal force arises entirely from a component of the normal reaction N from
the road, which of the following relations is correct ?
A. N cosT = mg
N
B. N = mg cos T
gr
v2
C.
sin T
r
gr
v2
D.
T
tan T
E.

19.

18.

N
sin T

mv 2
r

mg

(00)
glass
tube

T
rubber
bung

W
A student performing a centripetal force experiment whirls a rubber bung attached to one end of a string which
passes through a glass tube with smooth openings, and has a weight W hanging at its other end. The weight of the
rubber bung is much smaller than W. The rubber bung is set into a horizontal uniform circular motion with
angular speed Z while the length of the string beyond the upper opening of the glass tube is L and this portion of
the string makes an angle T with the vertical as shown. Which of the following statements is/are correct?
(1) If L is kept constant, T will decrease with Z.
(2) If T is kept constant, L will increase with Z.
(3) When W increases, T will increase.
A. (1) only
B. (2) only
C. (3) only

D. (1) and (2) only

E. (2) and (3) only

MC Question
Chapter 7 Circular motion
20.

(MC)7P5

(02) Out of syllabus


A particle P is moving in a horizontal circle in a clockwise direction as shown (top view). The following
*
diagrams show the direction of the acceleration a of the particle. Which of them is/are possible?

(1)

P
A.
B.
C.
D.

(3)

(2)

(1) only
(3) only
(1) and (2) only
(2) and (3) only

21. (03)

pendulum in

Out of syllabus

horizontal position

A simple pendulum is pulled horizontal and then released from rest with the string taut. Which of the following
statements about the tension in the string is not correct when the pendulum reaches its vertical position?
A.
B.
C.
D.

The tension equals the weight of the pendulum bob in magnitude.


The tension attains its greatest value.
The tension does not depend on the length of the pendulum.
The tension depends on the mass of the pendulum bob.

22 (04)

Out of syllabus
0.5 m
v
ball

Suppose a looping the loop experiment is performed on the moon, where the gravitational field strength is 1.6
Nkg-1 on its surface. If the radius of the circular loop is 0.5 m, find the minimum speed v to launch the ball at the
lowest point of the smooth track so that it can complete looping without leaving the track.
A.
B.
C.
D.

0.9 ms-1
1.8 ms-1
2.0 ms-1
4.0 ms-1

1.C 2.A 3.B 4.D 5.B 6.E 7.C 8.D


16.E 17.A 18.A 19.C 20.C 21.A 22.C

9.C

10.A 11.A 12.C 13.E 14.A 15.C

MC Question
Chapter 10Gravitation

(MC)10P1

1.

(80)
Two identical spheres, each of mass m and radius r are in contact. One sphere is displaced by a distance 4r, along
the line of centres, away from the first sphere. What is the ratio of the original gravitational force between the
spheres to the final gravitational force between them?
A. 3 : 1
B. 9 : 1
C. 16 : 1
D. 25 : 1
E. 36 : 1

2.

(81)
The moon orbits the earth once every 27.3 days, with a mean orbital radius of R. What is the period of an earth
satellite with an orbital radius of R/30 ?
A. 4 hr
B. 22 hr
C. 68 hr
D. 260 days
E. 4500 days

3.

(81)
An astronaut in an orbiting satellite is sometimes said to be weightless. This condition occurs when
(1) the gravitational attraction between the astronaut and the earth is just sufficient to provide the
centripetal force which keeps him in orbit.
(2) there is no reaction of the floor of the satellite on the astronauts feet.
(3) the gravitational pull of the earth is exactly cancelled by the gravitational pull of the moon.
A. (1) only
B. (3) only
C. (1) & (2) only
D. (2) & (3) only
E. (1), (2) & (3)

4.

(82)
A communication satellite in a circular orbit of radius R has a period of 24 hours. The period of a satellite in a
circular orbit of radius R/4 is
A. 3 hours
B. 6 hours
C. 12 hours
D. 24 hours
E. 96 hours

5.

(84) Out of syllabus


Taking the Earth to be a perfect sphere of uniform density rotating about its polar axis, which of the following
statements concerning the observed acceleration due to gravity, g, at the surface of the Earth is NOT true ?
(1) g at the equator is smaller than that at the poles.
(2) If the rate of rotation of the Earth slows down, g at the equator increases.
(3) If the radius of the Earth increases with its density remaining unchanged, g at the poles decreases.
A. (1) only
B. (3) only
C. (1) & (2) only
D. (2) & (3) only
E. (1), (2) & (3)

6.

Out of syllabus
(85)
If v1 is the minimum speed for a projectile to escape from the earth and v2 is the orbital speed of a satellite circling
close to the earth, then v1 / v2 =
A. 1/ 2 .

7.

C. 1.

D.

2.

E. 2.

(87) Out of syllabus


A space capsule is launched with speed u from the surface of the Earth to a maximum height above the ground
equal to the radius of the Earth. A rocket is then fired horizontally which keeps the space capsule revolving in a
circular orbit round the earth at that altitude with speed v. The ratio u: v is equal to
A. 1 : 2

8.

B. 1 /2.

B. 1 : 2

C. 1 : 1

D.

2 :1

E. 2 : 1

(88)
Out of syllabus
A body is suspended by a string and allowed to swing as a simple pendulum. When it is moved from the north
pole to the equator, its period will
A. remain constant.
B. decrease.
C. increase.
D. decrease and then increase.
E.
increase and then decrease.

MC Question
Chapter 10Gravitation

(MC)10P2

9.

(88)
A satellite moving round the Earth in a circular orbit of radius R has a period T. What would the period be if the
orbit were of radius R/4 ?
A. T/8
B. T/4
C. T/2
D. 2T
E. 4T

10.

(89) Out of syllabus

Two points X and Y are at distances a and 2a from the centre of the Earth as shown in the diagram. The
gravitational potential at X is - 8 kJ kg-1. When a 1 kg mass is taken from X to Y, the work done on the mass is
A. - 4 kJ.
B. - 2 kJ.
C. + 2 kJ.
D. + 4 kJ.
E. + 8 kJ.
11.

(89)
Assuming the Earth to be a perfect sphere, what would its angular velocity of rotation have to be for an object at
the equator to be weightless (i.e. to give a spring balance reading of zero) ?
(Radius of the Earth = 6.4 u 106 m.)
A. 2.4 u 10-12 rad s-1
B. 1.6 u 10-6 rad s-1
C. 1.3 u 10-3 rad s-1
D. 8.0 u 102 rad s-1
E.
6.4 u 105 rad s-1

12.

(91)
In which of the following situations is the magnitude of the normal reaction of the supporting surface, R, equal to
the weight of the body, mg?
(1) At rest on a rough inclined plane.
(2) On the floor of a spacecraft in circular orbit around the earth.
(3) On the floor of a lift moving upwards with uniform velocity.
A. (1) only
B. (3) only
C. (1) & (2) only
D. (2) & (3) only
E. (1), (2) & (3)

13.

(92) Out of syllabus


An object of mass m is released from a spacecraft at a distance 3R from the centre of the Earth which has radius R
and mass M. On reaching the Earth's surface, the increase in kinetic energy of the object is
B. 2GmM / (3R)
C. GmM / (2R)
D. GmM / R
E. 2GmM / R
A. GmM / (3R)

14.

(93) Out of syllabus


The period of a simple pendulum undergoing simple harmonic motion may be increased by
A. using a heavier pendulum bob.
B. increasing the amplitude of oscillation.
C. placing the pendulum at the top of a mountain.
D. placing the pendulum at the North Pole.
E.
shortening the string attached to the bob.

15.

(93)
A communication satellite appears stationary vertically above an observer at the equator. The height of the
satellite above the observer is 3.6 u 107 m. Calculate the mass of the earth.
Given : Radius of the earth = 6.4 u 106 m
Gravitational constant = 6.7 u 10-11 N m2 kg-2
A. 4.5 u 1024 kg
B. 5.0 u 1024 kg
C. 5.5 u 1024 kg
D. 6.0 u 1024 kg E. 6.5 u 1024 kg

MC Question
Chapter 10Gravitation
16.

17.

(MC)10P3

(93)
For planets or satellites in circular orbits around a celestial body such as the sun or the earth, the period T is
related to the radius of orbit r by Kepler's 3rd Law T2 = k r3 where k is a constant.
Which of the following statements concerning the constant k is correct?
A. It is a dimensionless constant whose value is not affected by the choice of units.
B. It is a universal constant whose value is not affected by the choice of units.
C. 1t is a universal constant whose value depends on the choice of units.
D. It would have a certain value for the earth moving around the sun, but a different value for another planet
moving around the sun.
E.
1t would have a certain value for all planets moving around the sun, but a different value for all satellites
moving around the earth.
(93) Out of syllabus
The velocity of escape from the earth is Vo. For a planet with radius twice that of the earth and with density three
times that of the earth, the velocity of escape from the planet would be
A.

3 Vo / 2

B. 2 Vo

C.

D. 2 3 Vo

6 Vo

E. 2 6 Vo

18.

(94) Out of syllabus


Two satellites A and B of the same mass are moving in circular orbits round the earth. The radius of A's orbit is r
and that of B's orbit is 2 r. Their total mechanical energies are EA and EB respectively. Which of the following
descriptions about EA and EB is correct?
(Gravitational potential energy is taken to be zero at infinity.)
A. EA > 0 and EB = 2EA
B. EA > 0 and EB = EA/ 2
C. EA > 0 and EB = -2EA
D. EA < 0 and EB = 2EA
EA < 0 and EB = EA/ 2
E.

19.

(95)
Out of syllabus
On a certain planet, an object is thrown vertically upwards with an initial velocity of v1 and it returns to the
ground after time t. If the velocity of escape from the planet is v2, find the radius of the planet.
2

A.

20.

2v1 t
v2

B.

4v1 t
v2

C.

2v 2 t
v1

D.

v2 t
4v1

E.

v2 t
2v1

(96)
In which of the following cases is/are the resultant force on the object zero?
(1) a satellite moving round the earth
(2) a feather falling freely in a vacuum cylinder in a laboratory
(3) a gas bubble rising with terminal velocity in water
A. (1) only
B. (3) only
C. (1) & (2) only
D. (2) & (3) only

E. (1), (2) & (3)

21.

(96)
Two satellites of the same mass travel around the earth in circular orbits of different radii. The satellite in the orbit
with smaller radius has
A. a greater speed.
B. a longer period.
C. a smaller acceleration towards the earth's centre.
D. a greater angular momentum about the earth's centre.
E.
a greater sum of gravitational potential energy and kinetic energy.

22.

(97)
A low-altitude satellite near the earth's surface has a speed of 7.90 km s-1. The radius of the earth is about 4 times
that of the moon and the ratio of the average density of the earth to that of the moon is about 5 : 4. The speed of a
low-altitude satellite near the moon's surface would be
A. 1.77 km s-1
B. 2.21 km s-1
C. 2.47 km s-1
D. 3.57 km s-1
E. 4.42 km s-1

MC Question
Chapter 10Gravitation

(MC)10P4

23.

(98)
Out of syllabus
The mass ratio of the earth and the moon is 81 : l and the earth-moon separation is 3.8 u 108 m. At which position
between the earth and the moon is the gravitational potential at a maximum ? (Ignore all other planets.)
A. On the surface of the earth
B. 3.4 u 108 m from the earth
C. 1.9 u 108 m from the earth
D. 0.4 u 108 m from the earth
E.
On the surface of the moon

24.

(99) Out of syllabus


Which of the following statements about a communication satellite in parking orbit above the earth's surface is
incorrect ?
A. It is accelerating towards the centre of the earth at all times.
B. It must be in a circular orbit above the earth's equator.
C. It is always vertically above the same place on the earth's surface.
D. It must be rotating in the same sense and with the same angular speed as the earth.
E.
It is at a height where its gravitational potential energy is numerically equal to its kinetic energy.

25.

(00)
X and Y are two planets. Each of them has a low-altitude satellite revolving in a circular orbit close to the planet.
If the two satellites are observed to have the same period, then X and Y must have nearly the same
A. mass.
B. average density.
C. radius.
D. acceleration due to gravity at the planet's surface.
E.
gravitational potential at the planet's surface.

26.

(00) Out of syllabus


A student used a simple pendulum to measure acceleration due to gravity at the earth's surface. The experimental
value was found higher than the standard value. Which of the following is a possible reason for this?
A. The effect due to air resistance is not negligible.
B. The stop watch used for the experiment runs too slowly.
C. The length of the string has been taken as the effective length of the pendulum.
D. The experiment has been performed at a place above sea-level.
E.
The experiment has been performed at a place below sea-level.

27.

(02)
Out of syllabus
Due to air resistance, changes may occur to a satellite orbiting the earth (assuming nearly circular orbit). Which of
the changes below is incorrect?
A. The total mechanical energy of the satellite will decrease.
B. The angular momentum of the satellite about the earths center will decrease.
C. The linear speed of the satellite will increase.
D. The time needed for the satellite to complete one revolution will increase.

28.

(03)
Given that the universal gravitational constant is 6.7 u 10-11 N m2 kg-2 and the radius of the earth is 6.4 u 106 m,
what is the average density of the earth?
A. 5.6 u 103 kg m-3
B. 7.3 u 103 kg m-3
C. 2.3 u 104 kg m-3
D. 6.0 u 1024 kg m-3

MC Question
Chapter 10Gravitation

(MC)10P5

29.

(03)
Out of syllabus
A satellite of mass m is launched from the earths surface into an orbit at a height of 3R above the earths surface,
where R is the radius of the earth. What is the gravitational potential energy gained by the satellite during this
process?
A. mgR/3
B. mgR/4
C. 2mgR/3
D. 3mgR/4

30.

(04)
A planet has a diameter 2 times that of the earth and a mass 3 times that of the earth. What is the approximate
gravitational field strength, in N kg-1, on the planets surface?
A. 7.5
B. 10
C. 15
D. 30

(04)
Out of syllabus
Long ago, astronomers wanted to find out whether the ring of Saturn is a rigid body or a group of satellites
revolving around Saturn. The linear speeds v of different layers of the ring were measured and how v varies with
the distance r from the centre of Saturn was determined. Which of the following relations between v and r
supports the suggestion that
(1) the ring is a rigid body,
(2) the ring is a group of satellites revolving around Saturn ?
(1)
(2)
1
A.
vvr
vv
r
1
vvr
vv
B.
r
1
vv
C.
vv r
r
1
vv
D.
vv r
r
32. (05)
An object of mass 5.0 kg has a weight of 8.2 N on the surface of the moon. The radius of the moon is R. What is
the gravitational field strength in N kg-1, at a point distance 2R from the centre of the moon ? (Assume that the
moon is a sphere of uniform density.)
A
1.6
B. 0.8
C. 0.4
D. 0.2
31.

33.

(06)
The earth is at a distance r metres from the centre of the sun. It takes 365 days for the earth to revolve once
N m2 kg-2, find
around the sun in a circular path. Given that universal gravitational constant G = 6.7 u 10-11
the mass of the sun in terms of r. (Neglect the effects of other planetary objects.)
A.
1.9 u 10-4 r3 kg
B.
5.9 u 10-4 r3 kg
C.
1.8 u 104 r3 kg
D.
1.2 u 106 r3 kg

Answer
1.B

2.A

3.C

4.A

5.B

6.D

7.D

8.C

9.A

10.D

11.C

12.B

13.B

14.C

15.D

16.E

17.D

18.E

19.D

20.B

21.A

22.A

23.B

24.E

25.B

26.B

27.D

28.A

29.D

30.A

31.B

32.C

33.B

MC Question
Chapter 25 Wave Propagation

1.

(80)
Which of the following gives typical orders of magnitude of the wavelengths in metres of
(1) infra-red
(2) ultra-violet rays
(3) medium-wave radio waves and
(4) gamma radiation

A.
B.
C.
D.
E.

2.

(MC)25P1

(1)
102
10-4
10-4
10-7
10-4

(2)
10-4
10-7
10-7
10-4
10-7

(3)
10-2
10-2
10-2
102
102

(88)

(4)
10-7
10-7
10-12
10-12
10-12

A
P

A loudspeaker at O produces a progressive sound wave of frequency 330 Hz which propagates along OA with a
speed of 330 m s-1. The phase difference between the air vibrations at P and Q, 0.5 m apart, is
A. dependent on the distance OP.
B. zero.
C. 0.5 radians.
D. S / 2 radians.
S radians.
E.
3.

(90) Out of syllabus


A rubber band is of unstretched length " and force constant k. When it is stretched to a length 2 ", the speed of
transverse waves on it is v. What will be the speed of transverse waves when it is stretched to a length 3 "?
A. v

4.

B.

C. 3v /2

2v

D.

3v

E. 2v

(92)
Figure (a)
1

10 11

Figure (b)
1

4 5 678 9 10

11

Direction of sound wave

Figure (a) shows the positions of equally spaced molecules in a solid lattice. A longitudinal sound wave travels
from left to right through the solid. At a certain instant, the displaced positions of the molecules are shown in
Figure (b). Immediately afterwards, what will be the directions of motion of particle 1 and particle 7 ?
Particle 1
Particle 7
A. to the right
to the right
B. to the right
to the left
C. to the left
to the right
D. to the left
to the left
E.
at rest
at rest

MC Question
Chapter 25 Wave Propagation

5.

(93)

(MC)25P2

displacement

time in seconds
0

0.20

0.45

0.70

A displacement-time graph of a particle in a plane progressive wave is shown. What is the frequency of this
wave ?
A. 1.43 Hz
B. 2 Hz
C. 2.22 Hz
D. 4 Hz
E.
5 Hz

6.

displacement / m

displacement / m

(94)

0.3

0.6

0.9

position/m

0.5

1.5

2.5

time/ms

Figure (b)

Figure (a)

Figure (a) represents the displacement-position graph of a travelling wave at a certain instant and Figure (b)
represents the displacement-time graph of a particle in the wave. The speed of the wave is
A. 300 m s-1
B. 150 m s-1
C. 1.2 m s-1
D. 0.6 m s-1
E. 0.3 m s-1

7.

(95)

direction of propagation of wave

The figure above shows a sound wave travelling to the right in air. Air particles A and B are at the centre of a
compression and a rarefaction respectively. Which of the following gives correctly the directions of motion of A
and B at the moment shown ?
Particle A
Particle B
A. to the right
to the left
B. to the right
at rest
C. to the right
to the right
D. at rest
to the right
E.
to the left
to the right

8.

(96)
The least separation between two points with a phase difference of S/6 on a progressive wave is 0.05 m. If the
velocity of the wave is 150 m s-1 the frequency of the wave is
A. 125 Hz.
B. 250 Hz
C. 375 Hz.
D. 500 Hz
E.
1000 Hz.

MC Question
Chapter 25 Wave Propagation

9.

(MC)25P3

(97)

P
Q

The above figure shows a transverse wave propagating to the right along a string. At the moment shown, which of
the labelled particles has its velocity and acceleration in opposite directions?
A. P
B. Q
C. R
D. S
E. T

10.

(98) Out of syllabus, but you may try


The displacement-time graphs of two oscillating particles P and Q are shown below.

Which of the following phase relations between P and Q is correct?


S
A. P leads Q by
.
4
B.

11.

P leads Q by

C.

2
P leads Q by S.

D.

Q leads P by

E.

Q leads P by

S
4

S
2

.
..

(98)

The above figure shows a transverse wave propagating along a sting. At the instant shown, the particle D on the
sting is moving downward. Which of the following deductions is/are correct?
(1) The wave is propagating to the left.
(2) Particle B takes longer than particle A to return for the first time to the respective equilibrium positions
along the dotted line.
(3) Particles C and D are moving in opposite directions at the instant shown.
A. (1) only
B. (3) only
C. (1)&(2)only
D. (2) & (3) only
E. (1), (2) & (3)

MC Question
Chapter 25 Wave Propagation

12.

(00)

(MC)25P4

Displacement
of air particles
+

P
position

The above figure shows an instantaneous wave profile representing a sound wave travelling to the right in air.
Which of the following about the part of the wave at P at this instant is/are correct ? (Take the displacement
towards the right as positive.)
(1) P is a centre of compression.
(2) The air particle at P has the greatest kinetic energy.
(3) The air particle at P is moving towards the right.
A. (1) only
B. (3) only
C. (1) & (2)only
D. (2) & (3) only
E. (l),(2)&(3)

13.

(01)
The SI unit of time, second, used to be defined as 9 192 631 770 times the period of the electromagnetic wave
emitted from a caesium-133 atom. In which region of the electromagnetic spectrum is this wave to be found?
A. infra-red
B. ultra-violet
C. radio wave
D. microwave
E.
visible

14.

(02)

Out of syllabus but you may try

Q
time

The waveforms in the figure show the time variation of two physical quantities P and Q. What is the phase
relationship between P and Q?
A.
B.

P leads Q by
P leads Q by

C.

Q leads P by

D.

Q leads P by

S
2

S
4

S
2

S
4

MC Question
Chapter 25 Wave Propagation

(MC)25P5

15.

(02)
The SI unit of length, metre, is defined as n times the wavelength of a certain orange-red light emitted from a
krypton-86 atom. The number n is of the order
A. 104.
B. 106.
C. 108.
D. 1010.

16.

(03)

R
Q
P
The figure shows the instantaneous positions of points P, Q and R on a sinusoidal wave propagating to the right.
The three points will reach their respective equilibrium positions at different times in the sequence
A. R. P, Q.
B. R, Q, P.
C. P, R, Q.
D. P, Q, R.
17.

(03)

Energy and Use of Energy


Glass

Light
source

Detector

Two different types of tinted glass, X and Y, are used to make filters for sunglasses. A sample of each is placed
in turn between a light source and a detector as shown. Both samples are identical in size and shape.
Intensity

Level without
sample

400 500 600 700 800

wavelength/nm

The intensity of the transmitted light received by the detector for each sample is plotted against the wavelength of
light. Which of the following deduction is/are correct?
(1)
(2)
(3)

Sample X is better for absorbing red light than sample Y.


Sample Y is better at protecting the eye from ultra-violet radiation.
The view is dimmer when seen through sample Y than through sample X.

A.
B.
C.
D.

(1) only
(3) only
(1) and (2) only
(2) and (3) only

MC Question
Chapter 25 Wave Propagation

18.

(MC)25P6

(05)
direction of propagation
P

Q
R

The above figure shows the shape of a transverse wave travelling along a string at a certain instant. Which of the
following statements about the motion of the particles P, Q and R on the string at this instant is correct ?
A. Particle P is moving downwards.
B. Particle Q is stationary.
C. Particle R is at its maximum acceleration.
D. Particles P and Q are in phase.

19.

(06)

TOP VIEW

S /6

S /6
wall

Water waves of frequency 2 Hz travelling with speed 8 m s-1 strike a straight wall. The parallel straight lines
represent wave crests and they make an angle of S/ 6 with the wall as shown. What is the phase difference
between the waves at two points 1 m apart along the wall ?
A.

20.

21.

S
8

B.

S
4

C.

S
3

D.

S
2

Out of syllabus
(06)
A metal rod of length 1.0 m is hit with a hammer at one end. It takes 5.5 u l0-4 s for the compression pulse
generated to travel to the other end and reflected back to the end hit by the hammer. Find the Young modulus of
the metal. (Given : density of the metal = 9.0 u 103 kg m-3)
A. 1.2 u1011 Pa
B. 0.6 u 1011 Pa
C. 3.3 u 107 Pa
D. 1.6 u 107 Pa
Energy and Use of Energy
(06)
The intensity of the solar radiation falling normally on the earth's surface due to the overhead sun is about
2
500 W m-2. It is known that about
of the solar radiation reaching the earth is absorbed by the atmosphere.
3
Estimate the total power output of the sun. (Given : earth - sun separation = 1.5 u 108 km)
A. 1 u 1020W
B. 4 u 1020 W
C. 1 u 1026W
D. 4 u 1026W

MC Question
Chapter 25 Wave Propagation

22.

(07)
Which of the following best gives the orders of magnitude of the wavelengths of ultra-violet radiation and
microwaves in air ?

A.
B.
C.
D.

23.

(MC)25P7

ultra-violet radiation
10-8 m
10-8 m
10-10 m
10-10 m

microwaves
10-2 m
10-5 m
10-2 m
10-5 m

Out of syllabus
(07)
A uniform heavy rope hangs freely from a support. The support is made to oscillate horizontally with a small
amplitude so as to send transverse waves down the rope. A standing wave is set up as these waves are reflected at
the free end. Which of the following statements is/are correct ?
(1) The tension is the same throughout the rope.
(2) The wavelength is the same throughout the rope.
(3) The frequency of oscillation is the same at all vibrating points of the rope.
A. (1) only
B. (3) only
C. (1) and (2) only
D. (2) and (3) only

Answer
1. E
11. A
21. D

2. E
12. E
22. A

3. D
13. D
23. B

4. C
14. A

5. B
15. B

6. A
16. A

7. A
17. C

8. B
18. C

9. E
19. B

10. A
20. A

MC Question
Chapter 26 Wave Phenomena I

1.

(MC)26P1

(83)
M2
M1

v2
i

v1

Light travels in media M1 and M2 with speeds v1 and v2 respectively. When light travelling from medium M1 strikes
a boundary between the two media with angle of incidence i, it suffers total internal reflection. This indicates that
(1) v1 is less than v2.
(2) sin i is less than v1 / v2.
(3) sin i is greater than v2 / v1.
A. (1) only
B. (3) only
C. (1)&(2)only
D. (2) & (3) only
E. (1), (2) & (3)
2.

(84)

Out of syllabus
light source

observer

Fig. 1

Fig. 2

Fig. 3

Three polaroid sheets are arranged in the three different ways as shown in Figures 1, 2 and 3 and an observer looks
towards the light source as shown in the diagrams. The bold lines indicate the directions of the transmission axes
in each case. Which of the following statements correctly describes the effect of the combinations?
Figure 2
Figure 3
Figure 1
A.
opaque
opaque
opaque
B.
opaque
opaque
transparent
C.
opaque
transparent
opaque
D.
opaque
transparent
transparent
E.
transparent
opaque
transparent
3.

(84)
Light travels between two media X and Y. If the refractive index of X is greater than the refractive index of Y,
which of the following is/are possible ray diagram(s)?
(1)
(2)
(3)

Y
X
A. (1) only
4.

B. (3) only

X
C. (1) & (2)only

D. (2) & (3) only

(85) Out of syllabus


When parallel light is incident at the Brewster angle in air on the surface of a glass block,
A.
the light is totally reflected.
B.
the reflected light is partially polarised.
C.
the transmitted light is unpolarised.
D.
the reflected and refracted wavefronts are at right angles to each other.
E.
the angle of incidence is equal to the angle of refraction.

E. (1),(2)& (3)

MC Question
Chapter 26 Wave Phenomena I

(MC)26P2

5.

(86) Out of syllabus


When a beam of unpolarised light travelling in air falls on the surface of a block of glass, it is possible to find an
angle of incidence such that
A.
none of the light is reflected.
B.
all of the light is reflected.
C.
the reflected light is completely plane polarised.
D.
the transmitted light is completely plane polarised.
E.
the reflected light and transmitted light are both completely plane polarised.

6.

(87)
A stroboscope is used to freeze a wave pattern. When the flashing frequency of the stroboscope is slightly reduced,
the wave pattern appears to
A.
move forward.
B.
move backward.
C.
move forward and then backward.
D.
move backward and then forward.
E.
remain stationary.

7.

(87)

P
A wave pulse is moving with uniform speed along a rope attached to a fixed wall. A graph of the vertical
displacement s against time t for a point P on the rope would be:
A.
B.
C.

D.

8.

E.

s
0

s
t

s
0

Out of syllabus
(87)
travels along the positive y-axis. A metal grid is placed in the xz-plane. The electromagnetic wave detected by the
detector is
z
A.
of zero intensity.
detector
direction
B.
unpolarised.
of
O
C.
plane-polarised in the yz-plane.
y
unpolarised
D.
plane-polarised in the xz-plane.
x
wave
E.
plane polarised in the xy-plane.

MC Question
Chapter 26 Wave Phenomena I

9.

(88)

(MC)26P3

Out of syllabus
light
source S

Polarized light is obtained by passing a narrow beam of unpolarized light from source S through a tank of water to
which a drop of milk has been added. Which of the following statements is/are correct ?
(1) Light from source S must be monochromatic.
(2)
Completely polarized light is detected at position M.
(3)
The drop of milk provides particles to scatter the light.
A. (1) only
B. (3) only
C. (1)&(2) only
D. (2) & (3) only
E. (1), (2) & (3)

Out of syllabus
10. (89)
A rotating disc, with a small hole near its edge is illuminated by a lamp which flashes at a rate of 50 times per
second. The hole is observed to be moving backwards slowly relative to the actual direction of rotation of the disc.
The disc is probably rotating at
A.
24 revolutions per second.
B.
26 revolutions per second.
C.
50 revolutions per second.
D.
98 revolutions per second.
E.
102 revolutions per second.
11. (90) Out of syllabus
When parallel light is incident at the Brewster angle in air on the surface of a glass block, which of the following
statements is/are correct ?
(1)
The refracted light is plane-polarised.
(2)
The reflected light is plane-polarised.
(3)
The reflected ray and the refracted ray are at right angles to each other.
A. (1) only
B. (3) only
C. (1) & (2)only
D. (2) & (3) only
E. (1), (2) & (3)
12. (90)

V
P
shallow

deep
IV

III
II
I
Q

The figure shows wave crests moving in the direction of the arrow towards the interface PQ between a shallow
region and a deep region as shown in the figure above. Which of the lines shown may represent one of the wave
crests in the deep region ?
A. I
B. II
C. III
D. IV
E. V
13. (90)

A disc with a spot on it rotates at a constant speed in a darkened room. A student shines a stroboscopic lamp on it.
When the flashing rate is 8 flashes per second, the disc appears stationary with two spots on it as shown. When the
flashing rate is reduced to 2 flashes per second, the disc would appear to be stationary with
A.
no spot.
B.
one spot.
C.
two spots.
D.
four spots.
E.
eight spots.

MC Question
Chapter 26 Wave Phenomena I

14. (91)

(MC)26P4

Out of syllabus

light
source

P
A

observer
Light from an unpolarised source is allowed to fall on a piece of polaroid P and then on a second polaroid A. In the
position shown in the diagram, the intensity of the light emerging from polaroid A is a maximum. As A is slowly
rotated, the intensity of light emerging is reduced to half the maximum value at angleT1, and to a minimum at
angle T2. Which of the following gives the correct values of angles T1 and T2 ?
A.
B.
C.
D.
E.

T1

T2

30
45
45
60
60

90
90
180
90
180

15. (92) Out of syllabus


When unpolarised light travelling in air falls on the surface of a glass block, it is possible to find an angle of
incidence such that
A.
none of the light is reflected.
B.
the reflected light and the transmitted light are both plane polarised.
C.
all the light is reflected.
D.
the transmitted light is plane polarised.
E.
the reflected light is plane polarised.

16. (92)

Out of syllabus

A light source S is viewed through two parallel pieces of polaroid P and Q. Q is gradually rotated until the field of
view becomes dark. Which of the following conclusions can be drawn ?
(1) The experiment shows that light is a transverse wave.
(2) The experiment shows that light from source S must be polarised.
(3) When Q is rotated further by 90, the field of view becomes dark again.
A. (1) only
B. (3) only
C. (1) & (2) only
D. (2) & (3) only
E. (1), (2) & (3)

Out of syllabus
17. (93)
Which of the following equations correctly gives the relation between the polarizing angle i and the refractive
index n of a material ?
A.
sin i = n
B.
n sin i = 1
C.
cos i = n
D.
n tan i = 1
E.
tan i = n

MC Question
Chapter 26 Wave Phenomena I

(MC)26P5

18. (94)
The speed of light in a certain material is 1.6 u 108 m s-1. Find the critical angle for that material and air.
(Speed of' light in air = 3 u 108 m s-1)
A.
28.1
B.
32.2
C.
41.8
D.
48.0
E.
57.8

Energy and Use of Energy


19. (94)
A 10 W light bulb gives a certain illumination on a surface 1.5 m away. At what distance would a 40 W light bulb
give the same illumination ? (You may regard a light bulb as a point source)
A.
1.5 m
B.
3.0 m
C.
6.0 m
D.
12.0 m
E.
24.0 m
20. (94)
Out of syllabus
Which of the following waves can be polarised?
(1)
microwaves
(2) X-rays
(3)
ultrasonic waves
A. (1) only
B. (3) only
C. (1) & (2) only
21. (95)

D. (2) & (3) only

E. (1), (2) & (3)

intensity
Y
X
T
T
0o

30 o

60 o

90o

When a beam of light travels from a medium X to another medium Y, the variation in intensity of the refracted
beam when angle T varies from 0 to 90 is as shown. What is the ratio speed of light in X / speed of light in Y?
A. 1 : 2
B. 1 : 3
C. 2 : 1
D. 2 : 3
E. 3 : 1
22. (96)
When monochromatic light travels from glass to air, the emergent light, relative to the incident light, shows an
increase in
(1) frequency.
(2) wavelength.
(3)
velocity.
A. (1) only
B. (3) only
C. (1) & (2) only
D. (2) & (3)only
E. (1),(2) & (3)

Out of syllabus
23. (96)
When a beam of unpolarized light travelling in air falls on a water surface, it is possible to find an angle of
incidence such that
(1)
all the light is reflected.
(2)
the light passing into water is plane polarized.
(3)
the reflected light is plane polarized.
A. (1) only
B. (3) only
C. (1) & (2) only
D. (2) & (3) only
E. (1), (2) & (3)

MC Question
Chapter 26 Wave Phenomena I

(MC)26P6

24. (97)

Three identical long spiral springs are connected respectively to a wall, a light thread and a heavy spring. If a
crest-shaped pulse is sent along each spiral spring towards the respective boundary, which of the following
diagrams correctly show(s) the reflected pulse ?
(Transmitted pulse is not shown.)

A. (1) only

B. (3) only

C. (1)& (2) only

D. (2)&(3)only

E. (1),(2)&(3)

25. (97)
A beam of monochromatic light passes through three media of refractive indexes n1, n2 and n3 respectively as
shown. The boundaries between the media are parallel.

n1
n2
n3
Which of the following relations between n1, n2 and n3 is correct ?
A.
n1 > n3 > n2
B.
n3 > n1 > n2
C.
n1 > n2 > n3
D.
n2 > n1 > n3
E.
n3 > n2 > n1
26. (98)
A wave is incident on a boundary between two media X and Y at an angle of incidence i. AB represents the incident
wavefront and CD the refracted wavefront as shown below.

Which of the following statements is/are correct?


(1) The frequency of the wave changes when passing from X to Y.
(2) The end B of the wavefront travels to D at the same time as the end A travels to C.
(3) When angle i increases, it is possible for the wavefront to be totally reflected at the boundary.
A. (1) only
B. (3) only
C. (1)&(2)only.
D. (2) & (3) only
E. (1), (2)&(3)

MC Question
Chapter 26 Wave Phenomena I

(MC)26P7

Out of syllabus
27. (98)
Sunlight is scattered by air molecules in the atmosphere. Which of the following statements concerning this
process is/are correct ?
(1) Light is absorbed by air molecules and is then re-emitted.
(2) The scattered light observed on the earth's surface is partially plane-polarized.
(3) Red light is scattered more than blue light is.
A. (1) only

B. (3) only

C. (1) & (2) only

D. (2) & (3) only

E. (1), (2) & (3)

28. (01)
Out of syllabus
Three polaroid sheets P, Q and R are placed along a straight line with a lamp and a detector as shown.
P

lamp

Initially the directions of polarisation of P and Q are parallel but are both normal to that of R. What happens to the
intensity I recorded by the detector when Q is being rotated slowly through 90o until its direction of polarisation is
parallel to that of R ?
A.
I remains unchanged.
B.
I increases throughout.
C.
I decreases throughout.
D.
I increases and then decreases.
E.
I decreases and then increases.
29.

Out of syllabus
(02)
In which of the following the phenomenon of polarization cannot be observed ?
A. Light reflected from the surface of water.
B. Microwaves emitted from a transmitter.
C. Light scattered from a clear sky.
D. Ultrasonic waves used in sonar.

30. (03)

(I)
M
(II)
N
(III)

In the figure, M and N are two parallel boundaries separating media (I), (II) and (III). A light ray undergoes total
internal reflection and then refraction as shown. Arrange the speeds of light in the three media in descending order.
A.
(I) > (II) > (III)
B.
(I) > (III) > (II)
C.
(II) > (III) > (I)
D.
(III) > (I) > (II)

MC Question
Chapter 26 Wave Phenomena I

31.

(05)

(MC)26P8

cladding (n2)

core (n1)

The figure shows the structure of an optical fibre. It consists of an inner core of refractive index n1 and an outer
cladding of refractive index n2. Which of the following conditions is necessary for the ray to travel along the core
of the fibre?
A. n1 > n2 and the angle i should be greater than a certain value.
B. n1> n2 and the angle i should be less than a certain value.
C. n1 < n2 and the angle i should be greater than a certain value.
D. n1 < n2 and the angle i should be less than a certain value.
32. (06)
The refractive indices of water and glass are 1.33 and 1.50 respectively. Which of the following statements is/are
correct ?
(1) Light travels faster in water than in glass.
(2) The frequency of light is reduced when it travels from water to glass.
(3) Light bends towards the normal when it travels from glass to water.
A. (1) only
B. (3) only
C. (1) & (2) only
D. (2) & (3) only
33. (07)
A diver at a depth of x below the water surface looks up and finds that the sky appears to be within a circle of
radius r. The critical angle of water in this case is given by
A. tan-1 (r / x).
B. sin-1 (r / x).
C. tan-1 (x / r).
D. sin-1 (x / r).

MC Question
Chapter 26 Wave Phenomena I

34. (07)

unpolarzied
light of
intensity I

(MC)26P9

60o

Out of syllabus
P
Q
observer
A beam of unpolarized light of intensity I passes normally through two polaroids P and Q aligned parallel to each
other. The polarizing axis of each polaroid is represented by an arrow. The polarizing axis of P is vertical while
that of Q makes 60 with the vertical. What is the intensity of the transmitted beam detected by the observer,
assuming that the only absorption mechanism is due to polarization ?
I
A.
8
I
B.
4
3I
C.
8
I
D.
2

Answer
1.
11.
21.
31.

B
D
A
B

2.
12.
22.
32.

B
D
D
A

3.
13.
23.
33.

A
B
B
A

4.
14.
24.
34.

D
B
D
A

5. C
15. E
25. B

6. A
16. A
26. D

7. D
17. E
27. C

8. E
18. B
28. D

9. B
19. B
29. D

10. D
20. C
30. B

MC Question
Chapter 27 Wave Phenomena II

1.

2.

(80)
Which of the following statements about wave motion is correct?
(1) Diffraction cannot be exhibited by longitudinal waves.
(2)
Refraction prevents the formation of shadows with perfectly sharp edges.
(3)
Reflection is sometimes accompanied by a phase change of it radians.
A. (1) only
B. (3) only
C. (1) & (2) only
D. (2) & (3) only

(MC)27P1

E. (1), (2) & (3)

(80)

Out of syllabus

"
X and Y are two transparent but partially reflecting glass sheets, separated by a small distance ". When light of
wavelength O. is incident normally on plate X, the light emerging from Y is of very low intensity when the distance
" is equal to (where n is a small integer)
1
nO
A.
2
1
(2n  1)O
B.
2
C.
nO
1
D.
(4n  1)O
4
1
(2n  1)O
E.
4
3.

Out of syllabus
(81)
When two notes of nearly equal frequencies f1 and f2 with f2 > f1 are sounded together, beats are heard. Beats are
periodic variations in
A.
pitch, with beat frequency (f2 f1 ).
B.
pitch, with beat frequency (f2 + f1 ) / 2.
C.
loudness, with beat frequency (f2 f1 ).
D.
intensity, with beat frequency (f2 f1 )/2.
E.
intensity, with beat frequency (f2 + f1) / 2.

4.

(81)
In a Young's slits experiment to produce interference fringes on a screen, the separation of the interference fringes
is increased by increasing
(1)
the distance between the slits and the screen.
(2)
the wavelength of the light.
(3)
the distance between the source and the slits.
A. (1) only
B. (3) only
C. (1) & (2)only
D. (2) & (3) only
E. (l),(2)&(3)

5.

(82)
A diffraction grating ruled with 5000 lines per cm is illuminated normally by white light. If the wavelengths for
yellow light and violet light are 600 nm and 400 nm respectively, which of the following statements is FALSE ?
A.
The central image is white.
B.
The violet end of the first-order spectrum is closer to the central image than is the red end of the first-order
spectrum.
C.
The second-order image of yellow light coincides with the third-order image of violet light.
D.
The angular displacement of the second-order image of yellow light from the central image is sin-1 0.6.
E.
There is no fourth-order image for violet light.

MC Question
Chapter 27 Wave Phenomena II

6.

(83)

(MC)27P2

Out of syllabus but you may try


S
A

X
C

The arrangement shown below is used in a version of Young's experiment. X is a monochromatic light source. A, B
and C are narrow parallel slits. Bright fringes are observed on the screen S. What would happen if the whole set-up
were placed in water instead of air ?
(1)
The central bright fringe will become dark.
(2)
The fringes on both sides become multi-coloured.
(3) The separation between the fringes decreases.
A. (1) only
B. (3) only
C. (1) & (2)only
D. (2) & (3) only
E. (l), (2) & (3)
7.

8.

9.

(83)
Out of syllabus
When monochromatic light is incident normally on a wedge-shaped thin film, an interference pattern may be seen
by reflection. Which of the following changes would increase the number of fringes per unit length as seen by an
observer ?
(1) increasing the wavelength of the light
(2)
increasing the angle of the wedge
(3)
increasing the refractive index of the film material
A. (1) only
B. (3) only
C. (1)&(2)only
D. (2) & (3) only
E. (1),(2) & (3)

Out of syllabus
(84)
Young's slits are used to produce interference fringes with light of wavelength 600 nm. A thin sheet of mica of
refractive index 1.6 is placed in front of one of the slits and the centre of the fringe-system is displaced through 8
fringe widths. The thickness of the mica is
A.
120 nm.
B.
3000 nm.
C.
4000 nm.
D.
7700 nm.
E.
8000 nm.
(84)
Out of syllabus
In the tuning of a violin string, a pitch pipe of frequency 427 Hz was blown at the same time as the string was
plucked and 5 beats were heard every 2 seconds. Then the violin string was slightly tightened with a
fine-adjustment screw. When the pitch pipe and the string were sounded together again no beats were heard. The
initial frequency of the note produced by the string before any adjustment was made must have been
A.
422.0 Hz
B.
424.5 Hz
C.
429.5 Hz
D.
432.0 Hz
E.
437.0 Hz

10. (84) Out of syllabus


Newton's rings produced when a biconvex lens rests on a plane glass plate are observed using a travelling
microscope. If the biconvex lens is very slowly moved vertically upwards from the lower glass plate, which of the
following would be observed ?
A.
The central spot remains dark all the time.
B.
The rings disappear immediately.
C.
The rings move towards the centre.
D.
The rings move out from the centre.
E.
The rings are no longer concentric.

MC Question
Chapter 27 Wave Phenomena II

(MC)27P3

Out of syllabus
11. (85)
The surface of a material of refractive index 1.8 is coated with a thin film of liquid of refractive index 1.5 and
thickness 200 nm. White light falls normally on the thin film. Which of the following wavelengths (in air) of light
is NOT reflected from the thin film ?
A.
400 nm
B.
450 nm
C.
600 nm
D.
750 nm
E.
800 nm
12. (85)
White light is directed normally onto a diffraction grating and the diffracted light is observed through the telescope
of a spectrometer. The appearance of the zeroth order and the rust order diffraction pattern will look like:
(W: white; R: red; B: blue)
A. W W W
B. R B W B R C. B R W R B
D.R B W B R
E. B R W R B

continuous spectrum

continuous spectrum

line spectrum

line spectrum

Out of syllabus
13. (85)
Two signal generators are connected to display the formation of beats on a C.R.O. screen. For one particular
setting of the two signal generators, the following pattern is observed on the C.R.O. :
If the C.R.O. time base is set at 0.2 ms cm-1, the beat frequency is

2.5 cm

A. 100 Hz.

B. 200 Hz.

C. 400 Hz.

D. 1000 Hz.

E. 2000 Hz.

14. (86)
Out of syllabus
Two stretched wires are tuned to vibrate transversely at the same frequency of 1200 Hz. When the tension in one
of the wires is reduced by 1%, beats are heard as both wires vibrate. The beat frequency is
A.
3 Hz.
B.
6 Hz.
C.
12 Hz.
D.
24 Hz.
E.
1188 Hz.

15. (86)
In the diffraction of light round an obstacle, the angle of diffraction is increased when
A.
the wavelength of the incident light wave is increased.
B.
the wavelength of the incident light wave is decreased.
C.
the amplitude of the incident light wave is increased.
D.
the amplitude of the incident light wave is decreased.
E.
the width of the obstacle is increased.

MC Question
Chapter 27 Wave Phenomena II

16. (87)

Out of syllabus
S

(MC)27P4

M
O
B

A plane mirror M is illuminated by monochromatic light from a slit S. The virtual image of S by reflection and S
itself act as 2 coherent sources and the interference pattern is observed on the screen AOB at a distance from the
mirror. Which of the following statements about the interference pattern on the screen is/are correct ?
(1)
No interference pattern can be seen in the region OB on the screen.
(2) As the mirror M moves downward, the separation of the fringes decreases.
(3) As the mirror M moves horizontally away from the screen, the separation of the fringes increases.
A. (1) only
B. (3) only
C. (1) & (2)only
D. (2) & (3)only
E. (1),(2)&(3)

17. (88)

Out of syllabus

When monochromatic light is incident normally on a wedge-shaped thin air film, an interference pattern may be
seen by reflection. Which of the following is/are correct?
(1) Parallel fringes are observed.
(2)
If water is introduced into the region between the plates, the fringe separation decreases.
(3)
If the angle of the wedge is increased, the fringe separation decreases.
A. (1) only
B. (3) only
C. (1)&(2) only
D. (2) & (3) only
E. (1), (2) & (3)

18. (89)
Out of syllabus
The coated lens of a camera appears purple in daylight. According to the manufacturers, the coating has a
refractive index of 1.25 and the material of the lens has a refractive index of 1.50. If the wavelength of yellow light
in air is 520 nm, the approximate thickness of the coating is
A.
104 nm.
B.
130 nm.
C.
180 nm.
D.
210 nm.
E.
260 nm.
19. (89)
Light of wavelength O is incident normally on a diffraction grating with p lines per millimetre. The second-order
diffraction maximum is at an angle T from the central position. For a second grating with 3p lines per millimetre
illuminated normally by light of wavelength 5O/4, the angle between the first-order diffraction maximum and the
central position is I. Which of the following relations is correct?
A.
sin I = (5 sin T)/12
B.
sin I = sin (5T /12)
C.
sin I = sin (15T /4)
D.
sin I = (15 sin T ) / 8
E.
sin I = sin (15T /8)

MC Question
Chapter 27 Wave Phenomena II

(MC)27P5

20. (90)
L1

L2
B

2 loudspeakers L1 and L2 are connected to a signal generator G. A microphone is moved along the line AB and the
variation in intensity is noted. Which of the following statements concerning the above arrangement is/are
correct ?
(1) If the separation of the 2 loudspeakers is less than the wavelength of the sound emitted, no alternation of
maxima and minima can be detected along AB.
(2) If the frequency of the sound waves emitted is increased the separation between adjacent maxima along AB
will be increased.
(3) If the 2 loudspeakers are vibrating in antiphase, no alternation of maxima and minima will be detected along
AB.
A. (1) only
B. (3) only
C. (1) & (2) only
D. (2) & (3) only
E. (1), (2) & (3)

21. (90)

The figure shows two pulses travelling to the right along a rope. The right-hand end of the rope is fixed to a wall.
The following figures represent predicted positions of the pulses at later times (where appropriate, the arrows
indicate the direction of a pulse). Which of the following could NOT arise from the initial given condition ?
(1)
(2)
(3)

A. (1) only

B. (3) only

C. (1) & (2)only

D. (2) & (3) only

E. (1),(2) & (3)

22. (90)
The intensity of a sound wave is proportional to the square of the amplitude of the wave. If two waves of the same
frequency are superimposed in phase, the total intensity is proportional to
A.
the mean value of the intensities of the two waves.
B.
the square of the sum of the two amplitudes.
C.
the square of the mean value of the two amplitudes.
D.
the square of the difference of the two amplitudes.
E.
the sum of the intensities of the two waves.
23. (90)
White light diffracted by a single slit falls on a white screen. Which of the following statements is/are correct ?
(1)
The centre of the diffraction pattern is white.
(2) The first minimum is closer to the centre for red light than for blue light.
(3)
The central band width is increased as the slit width is increased.
A.
(1) only
B. (3) only
C. (1) & (2) only
D. (2)&(3) only
E. (1),(2)&(3)

MC Question
Chapter 27 Wave Phenomena II

(MC)27P6

24. (91)
In experiments with optical diffraction gratings, the ratio: grating spacing/wavelength of light is of the order of
A.
10-6
B.
10-3
C.
1
D.
103
E.
106
25. (91) Out of syllabus
In an arrangement for viewing Newton's rings, if the lens which rests on a glass plate were moved slowly upwards
by one wavelength (of the viewing light), which of the following would be observed ?
A.
The central spot becomes bright.
B.
The rings disappear.
C.
The rings move towards the centre.
D.
The rings move out from the centre.
E.
The rings are no longer concentric.
26. (91)
When a diffraction grating is replaced by another with more lines per mm, which of the following quantities is/are
increased ?
(1)
the angle of diffraction for every spectral line
(2)
the angular separation of red and blue lines in the first order spectrum
(3) the number of orders which can be observed
A.(1) only
B. (3) only
C. (1) & (2) only
D. (2) & (3) only
E. (1),(2)& (3)
27. (91) Out of syllabus
Orange light of wavelength 600 nm (in air) is incident normally from air onto a liquid film whose refractive index
is 1.25. For what minimum value of the film thickness will the greatest amount of light be transmitted through the
film?
A.
120 nm
B.
180nm
C.
240 nm
D.
300 nm
E.
480 nm

28. (92)
3.22 m
signal
generator

X
3.30 m

Two loudspeakers are connected to the same signal generator. A microphone positioned at X detects maximum
intensity. When the microphone is moved upwards, maximum intensity is also detected at Y. Which of the
following may give possible values of the wavelength of the sound emitted from the loudspeakers ?
(1) 0.04m
(2) 0.08 m
(3) 0.16 m
A. (1) only
B. (3) only
C. (1) & (2) only
D. (2) & (3) only
E. (1), (2) & (3)

MC Question
Chapter 27 Wave Phenomena II

(MC)27P7

29. (94) Out of syllabus


Three tuning forks X, Y and Z are of slightly different frequencies. If X and Y are sounded together, 3 beats per
second are heard while X and Z sounded together give 1 beat per second. Which of the following is/are correct
deduction(s) ?
(1)
4 beats per second are heard when Y and Z are sounded together.
(2)
The frequency of Z is higher than that of Y.
(3) If X has the highest frequency, then the frequency of Y must be the lowest.
A. (1) only
B. (3) only
C. (1)&(2)only
D. (2) & (3) only
E. (1), (2) & (3)
30. (95) Out of syllabus
A coating material of refractive index 1.25 is used for the blooming of a lens having a larger refractive index. For
normal incidence, if green light is to be transmitted in its greatest amount through the lens, which of the following
thickness of the coating would do?
(Given : wavelength of green light in air is 550 nm)
(1) 137.5 nm
(2) 220 nm
(3) 330 nm
A. (1) only
B. (3) only
C. (1)&(2)only
D. (2) & (3) only
E. (1), (2) & (3)
31. (95)
In Young's double-slit experiment, which of the following combinations of monochromatic light, the
slit-separation and the slit-to-screen distance would produce the widest fringe separation on the screen ?
Monochromatic light
Slit-separation
Slit-to-screen distance
A.
red light
1 mm
1m
B.
red light
1 mm
2m
C.
red light
2 mm
1m
D.
green light
1 mm
2m
E.
green light
2 mm
1m
32. (96)

Out of syllabus

T
Monochromatic light of wavelength 589 nm is shone normally on a wedge-shaped thin air film to form
interference fringes. If the distance between the sixth and ninety-sixth dark fringes is 15.8 mm, calculate the angle
of the wedge, T.
A. 0.024
B. 0.048
C. 0.072
D. 0.096
E. 0.190
33. (96)
A beam of white light is shone normally on a diffraction grating. The diagram below shows the spectra of the first
two orders, which may not be drawn to scale. The first-order spectrum starts at an angle of 20 from the zeroth
order. The respective angular separations between the two ends (red and violet) of a spectrum are D and E for the
first- and second-order spectra.

Which of the following statements is/are correct ?


(1) In the first-order spectrum, P is the violet end.
(2) E is greater than D.
(3) There is no third-order spectrum.
A. (1) only
B. (3) only
C. (1)&(2)only

D. (2) & (3) only

E. (1),(2) & (3)

MC Question
Chapter 27 Wave Phenomena II

(MC)27P8

34. (97)

In a Young's double-slit experiment, light of wavelength 400 nm is used. If the path difference between the light
from the two slits X and Y to a point P on the screen is 3000 nm, which of the following is/are correct ?
(1) P is the 7th dark fringe.
(2)
The fringe separation on the screen increases if the light source is moved closer to the slits.
(3) P becomes a bright fringe if light of wavelength 500 nm is used.
A. (1) only
B. (3) only
C. (1)& (2) only
D. (2) & (3) only
B. (1),(2)&(3)
35. (97)
A plane transmission grating is placed at the centre of a circular 0 - 360 protractor. A beam of monochromatic
light is incident normally on the grating. The zeroth-order maximum occurs at a scale reading of 90 and a
first-order maximum occurs at a scale reading of 65. At what scale reading would a second-order maximum be
observed ?
A. 148
B. 140
C. 130
D. 58
E. 40
36. (97) Out of syllabus
In an arrangement for viewing Newton's rings, which of the following would be observed if water is introduced to
the space between the lens and the glass plate?
A.
The rings disappear.
B.
The central spot becomes bright.
C.
The separation between the rings increases.
D.
The rings shift towards the centre.
E.
There is no observable change.
37. (98)
A beam of red (R) light and violet (V) light strikes normally on a diffraction grating. Several orders of diffraction
of the light beam are produced on each side of the central axis. The wavelength of the red light is 700 nm and that
of the violet light is 400 nm. How will the second- and third- order diffracted beams of both colours appear ? (The
central axis is marked with a dot and the diffracted beams of other orders are not shown.)

Central axis
A.

R V

B.

V R

C.

R R

D.

R V

E.

V R

MC Question
Chapter 27 Wave Phenomena II

(MC)27P9

38. (99)

In a Young's double slit experiment, a monochromatic light source of wavelength 700 nm is used and the
separation of the slits is 0.1 mm. If 15 bright fringes are observed, what is the angle subtended by those fringes at
the centre of the double slit?
A.
4.8
B.
5.2
C.
5.6
D.
6.0
E.
6.4
39. (00)

Out of syllabus but you may try

Two transverse pulses travel towards each other along a stretched string as shown. They are identical and
symmetric in shape but one is inverted. At the moment the pulses pass each other the string becomes straight. At
this moment the wave energy
A.
becomes wholly elastic potential in the string.
B.
becomes wholly kinetic due to transverse motion of the string.
C.
becomes partly elastic potential and partly kinetic in the string.
D.
converts into internal energy and is dissipated as heat energy in the string.
E.
becomes momentarily zero.
40. (00) Out of syllabus

air

glass

thin film
A glass lens is made non-reflecting for yellow light by coating it with a thin film of transparent material of a
refractive index less than that of glass. Which of the following statements is/are correct?
(1)
The surface of the glass lens appears yellow in white light.
(2)
The minimum thickness of the film is 1/4 of the wavelength of yellow light in the film.
(3)
As a result of the destructive interference between the light rays reflected from the two interfaces, energy
which would have been wasted as reflected light increases the amount of transmitted light.
A. (1) only
B. (3) only
C. (l) & (2) only
D. (2) & (3) only
E. (1),(2)&(3)

MC Question
Chapter 27 Wave Phenomena II

(MC)27P10

41. (01)
In which of the following cases can the principle of superposition be applied to two overlapping waves of the same
nature ?
A.
Only when the two waves have the same frequency.
B.
Only when the two waves have the same amplitude.
C.
Only when the two waves travel in opposite directions.
D.
Only when the two waves are coherent.
E.
It can be applied in all cases.
42. (01)
A student prepares a double-slit set-up as shown below. However, no interference fringe can be observed on the
screen.
a = 1 mm
screen
sodium
lamp
d = 0.01 m

x = 0.1 m

D=1m

Your suggestion for improvement may be


A.
using a mercury lamp to provide radiation of a shorter wavelength.
B.
reducing x.
C.
reducing D.
D.
increasing a.
E.
reducing d.
43. (01) Out of syllabus
Which of the following statements concerning Newton's rings is/are correct ?
(1) A convex lens with long focal length should be used.
(2) Adjacent rings become farther apart when moving outwards from the central spot.
(3) The central spot is bright when some liquid is introduced between the convex lens and the reflector.
A. (1) only
B. (3) only
C. (1) and (2) only
D. (2) and (3) only
E. (1), (2) and (3)
44.

(02)

first-order
maximum

grating
T
incident
light

central
maximum
first-order
maximum

When a certain monochromatic light is passed through a diffraction grating, a pattern of maxima and minima is
observed as shown. Which of the following combinations would produce the largest angle T between the first
order maxima?

A.
B.
C.
D.

Grating (lines per mm)


200
200
400
400

Colour of light used


blue
red
blue
red

MC Question
Chapter 27 Wave Phenomena II

(MC)27P11

45. (03)
Light of wavelength 500 nm is incident normally on a plane diffraction grating having a ruling of 5000 lines per
cm. Which of the following descriptions is/are correct ?
(1)
The spacing between the rulings is 2000 nm.
(2) The angle through which the second-order spectrum is diffracted is 30o
(3)
No third-order spectrum is observable.
A. (1) only
B. (3) only
C. (1) and (2) only
D. (2) and (3) only
46. (04)
displacement

P
0

time
Q

Two waves P and Q of equal amplitude and frequency arrive at the same point X. The figure shows the
displacement-time graphs of waves P and Q at point X. What is the phase difference between the resultant wave
and wave P at point X?
A.
30q
B.
45q
C.
60q
D.
90q
47. (04)
(1)

(2)

Figure (1) shows two transverse pulses travelling towards each other along a stretched string. They are identical
and symmetric in shape but one is inverted. At the moment when the pulses overlap, the string becomes straight
as shown in Figure (2). P, Q and R are points on the string situated symmetrically within the overlapping part. At
the instant when the string is straight, which of the following statements about the strings motion at P, Q and R is
correct?
A.
P, Q and R are moving.
B.
P, Q and R are at rest.
C.
Only Q is moving.
D.
Only Q is at rest.
48. (04)
A Young's double-slit experiment was performed using a monochromatic light source. Which of the following
methods would result in a greater fringe separation on the screen?
(1)
Using a monochromatic light source of longer wavelength
(2)
Using a double slit with greater slit separation
(3)
Using a double slit with larger slit width
A. (1) only
B. (1) and (2) only
C. (2) and (3) only
D. (1), (2) and (3)

MC Question
Chapter 27 Wave Phenomena II

(MC)27P12

49. (05)

S1

S2

In the figure, S1 and S2 are two dippers in a ripple tank generating circular waves of wavelengths O1 and O2
respectively with O1 = 2 O2. The solid lines represent crests. S1 and S2 are generating crests at the instant shown. P
is a point equidistant from S1 and S2. Which of the following statements is correct ?
A. The frequency of the waves generated by S1 is twice that of S2.
B. The waves at P always reinforce each other.
C. The waves at P do not obey the principle of superposition as their wavelengths are not equal.
D. The waves at P still obey the principle of superposition but they do not always reinforce each other.
50. (05)
Out of syllabus
In the figure, XY is an optically flat glass plate while PQ is a glass specimen with a uniform indentation along PQ.
When monochromatic light of wavelength O falls normally on the system, interference fringes are observed as
shown below. The maximum depth of indentation is h. What is the value of h, in terms of a, b and O? (Diagram not
drawn to scale.)
Monochromatic light

Q
b

A.
B.
C.
D.

h = bO/2a
h = bO/a
h = 2bO /a
h = (a - b)O / 2a

51. (06)

Two identical wave pulses travel along a stretched string with one end fixed to the wall as shown above. Which of
the following can be the resultant waveform at different instants when the pulses meet each other ?
(1)
(2)
(3)

A.
B.
C.
D.

(1) only
(3) only
(1) & (2) only
(2) & (3) only

MC Question
Chapter 27 Wave Phenomena II

(MC)27P13

Out of syllabus but you may try


52. (06)
In a Young's double-slit experiment, the intensity of illumination detected at the centre of the fringe pattern is I. If
one of the identical slits is covered, the intensity measured at the same position would be
I
A.
4
I
B.
2
I
C.
2
D. I
Out of syllabus
53. (06)
Which of the following phenomena can be explained by interference of waves ?
(1) The formation of nodes when one end of a stretched string is fixed while the other end is connected to a
vibrator
(2)
The purplish appearance of the coated lens of a camera under sunlight
(3) The production of beats when two tuning forks of nearly the same frequency are sounded together
A. (1) only

B. (1) & (2) only

C. (2) & (3) only

D. (1), (2) & (3)

54. (07)
3rd

A beam of monochromatic light falls at normal


incidence on a diffraction grating. Third-order
diffracted beams are found at angles of 45 to the
incident direction. What is the highest order of
diffracted beam that can be observed ?
A.
B.
C.
D.

2nd
45o

1st

45o

1st

Incident beam

3rd order
4th order
5th order
6th order

2nd

diffraction
grating
3rd

55. (07)
In a Young's double-slit experiment using monochromatic light of wavelength 550 nm, the fringes are observed
through the eyepiece of a travelling microscope at a distance of 1.0 m from the double slits. If the separation
between the first and the fifth dark fringes is 5.0 mm, find the slit separation.
A. 0.3 u 10-4 m
B. 1.1 u 10-4 m
C. 4.4 u 10-4 m
D. 5.5 u 10-4 m

Answer
1.
11.
21.
31.
41.
51.

B
A
B
B
E
D

2.
12.
22.
32.
42.
52.

E
B
B
D
E
A

3.
13.
23.
33.
43.
53.

C
E
A
E
A
B

4.
14.
24.
34.
44.
54.

C
B
C
B
D
B

5.
15.
25.
35.
45.
55.

E
A
C
A
C
C

6.
16.
26.
36.
46.

B
C
C
D
B

7.
17.
27.
37.
47.

D
E
C
B
D

8.
18.
28.
38.
48.

E
A
C
C
A

9.
19.
29.
39.
49.

B
D
B
B
D

10.
20.
30.
40.
50.

C
A
B
D
A

MC
Chapter 28 Stationary Waves

1.

(82)

(MC)28P1

S1

S2

The figure above shows each of the stationary traces S1 and S2 observed on the screen of a CRO connected to a
microphone, when two tuning forks F1 and F2 respectively are sounded in turn. (The time base remains the same
in each case.)
Which of the following statements is/are correct ?
(1) The period of F1 is greater than the period of F2.
(2) The pitch of F1 is greater than the pitch of F2.
(3) The speed of sound from F1 is greater than the speed of sound from F2.
A. (1) only

2.

B. (3) only

C. (1) & (2) only

D. (2) & (3) only

E. (1), (2) & (3)

Out of syllabus
(83)
In a standing waves in a closed pipe,
(1) some air molecules do not vibrate.
(2) no energy comes out of the pipe.
(3) in the fundamental mode of vibration air molecules in the middle of the pipe oscillate through a larger
amplitude than molecules at any other part of the pipe.
A. (1) only

B. (3) only

C. (1) & (2) only

D. (2) & (3) only

E. (1), (2) & (3)

3.

(87) Out of syllabus


An open tube of length L and a string of length L produce the same fundamental note. However when each is
sounded independently the sound produced can be distinguished readily. This is because
A. the string produces some harmonics which are not produced by the open pipe.
B. the open pipe produces some harmonics which are not produced by the string.
C. they produce completely different harmonics.
D. the pipe produces longitudinal standing waves while the string produces transverse standing waves.
E.
they produce the same harmonics but the amplitudes of the corresponding harmonics are different.

4.

(88)

Out of syllabus
displacement

2a

3a

The figure above shows the variation of the displacement of air molecules along the x-axis in a standing sound
wave at a particular time. At what positions will the pressure remain constant with respect to time ?
A. x = 0 and x = 2a only
B. x = a and x = 3a only
C. x= 0 and x = a only
D. x = 2a and x = 3a only
E.
x = 0, x = 2a, x = 2a and x=3a

MC
Chapter 28 Stationary Waves

5.

(MC)28P2

(91)

W2

N
W

Two trains of travelling waves W1 and W2, of the same amplitude, wavelength and speed move in opposite
directions. The period of both of the waves is T and their amplitude is A. At time t = t1, the waves W1 and W2 are
as shown in the figure above. Which of the following statements is/are correct ?
(1) The wavelength of the waves is four times the distance MN.
(2) At time t = t1 + T/4 , the displacement at point M is 2A.
(3) Point N is a displacement node.
A. (1) only
B. (3) only
C. (1) & (2) only
D. (2) & (3) only
E. (1), (2) & (3)

6.

(91)
Q

Out of syllabus

A stationary sound wave vibrating in its fundamental mode is set up in a tube closed at one end. P and Q are two
points at the end and the middle of the tube along its axis. Neglecting end-correction, what is the phase difference
between the vibrations of the air molecules at point P and Q ?
A. 0
B. S / 4 radian
C. S /2 radian
D. S radian
E.
5 S /4 radian

7.

(92)
vibrator

Stationary wave patterns can be produced on an elastic string using the experimental set-up shown above by
adjusting the frequency f of the vibrator. Which of the following statements concerning the experiment is/are
correct ?
(1) Stationary wave patterns can be observed for more than one value of the frequency f.
(2) When the frequency f increases, the number of loops for the stationary wave pattern to be observed also
increases.
(3) For a stationary wave pattern to occur, the length of the string must be equal to an integral number of
wavelengths.
A. (1) only
B. (3) only
C. (1) & (2) only
D. (2) & (3) only
E. (1), (2) & (3)

MC
Chapter 28 Stationary Waves

8.

(93)

soft
sound

soft
sound

(MC)28P3

L
M

A loudspeaker L produces sound waves with frequency 1000 Hz. The sound waves are reflected from a wall S.
When a microphone M is moved between L and S, the loudness of the sound detected varies.
(Speed of sound in air = 340 m s-1 )
Which of the following statements is/are true ?
(1) The variation in the loudness of the sound is due to diffraction.
(2) The separation between consecutive positions of soft sound is 0.34 m.
(3) Increasing the sound frequency will make the positions of soft sound closer.
A. (1) only
B. (3) only
C. (1) & (2) only
D. (2) & (3) only
E. (1), (2) & (3)
9.

(94)

l /2
P

l /2
Q

Out of syllabus

A stationary sound wave vibrating in its fundamental mode is set up in a pipe open at both ends. If an air particle
at P oscillates with amplitude a, what are the amplitudes of oscillation of the air particles at Q and R ? (Neglect
end corrections)
Amplitude at Q
Amplitude at R
A.
0
a/2
B.
a/2
0
C.
0
a
D.
a
0
E.
a
a
10.

(94)

vibrator

stand

In the above experimental set-up, different stationary wave patterns are produced on an elastic string by adjusting
the frequency f of the vibrator. Which of the following statements is/are correct ?
(1) When f increases, the number of antinodes increases.
(2) When f increases, the speed of the waves on the string increases.
(3) The waves produced in air by the string has the same speed as the waves on the string.
A. (1) only
B. (3) only
C. (1) & (2) only
D. (2) & (3) only
E. (1), (2) & (3)
11.

(95) Out of syllabus


A uniform wire is clamped at both ends which are 0.4 m apart. The tension in the wire is 65 N and the area of its
cross-section is 4.5 u 10-7 m2. When the wire is plucked to produce transverse vibrations, the frequency of the
fundamental note obtained is 160 Hz. The density of the wire is
A. 5600 kg m-3
B. 8800 kg m-3
C. 12600 kg m-3
D. 14000 kg m-3
E.
22000 kg m-3

12.

(97) Out of syllabus


Two pipes A and B, with pipe A closed at one end and pipe B open at both ends, are sounded to give fundamental
notes of the same frequency. The length ratio of pipe A to pipe B is
A. 1 :4
B. 1 :2
C. 1:1
D. 2: 1
E. 4: 1

MC
Chapter 28 Stationary Waves

13.

(98)

(MC)28P4

Out of syllabus

The above figure shows a long vertical glass tube containing water. The water level is lowered until the first
resonance occurs when a vibrating tuning fork is placed at the opening of the tube. Which of the following
statements is/are correct ? (Neglect end corrections.)
(1) The sound produced by the air column above the water has the same frequency as the tuning fork.
(2) Resonance will occur again if the length of the air column above the water is increased to three times its
original value.
(3) Resonance will occur again if another tuning fork of twice the frequency is used.
A. (1) only
B. (3) only
C. (1) & (2) only
D. (2) & (3) only
E. (1), (2) & (3)
14.

(99)
Two dippers S1 and S2 1.2 cm apart are attached to a vibrator. They vibrate in phase in a ripple tank and generate
waves of wavelength 0.8 cm.

S2

S1
1.2 cm

Which of the following statements is/are correct ?


(1) S1 and S2 are positions of antinodes.
(2) There is only one point between S1 and S2 which is always stationary.
(3) The separation between two nearest antinodes between S1 and S2 is 0.4 cm.
A. (1) only
B. (3) only
C. (1) & (2) only
D. (2) & (3) only
15.

16.

E. (1), (2) & (3)

(02)
A stretched string is 1 m long and is fixed at both ends. Stationary waves of 400 Hz and 450 Hz but no other
intermediate frequency can be formed. What is the speed of the mechanical waves along the string?
A. 25 m s-1
B. 50 m s-1
C. 100 m s-1
D. 200 m s-1
(03) Out of syllabus
Two cylindrical tubes are identical except that one is open at both ends and the other is closed at one end. The air
columns in the tubes are set to oscillate at their respective fundamental frequencies. Which of the following is/are
the same for the sound waves in the tubes ?
(1) the number of displacement nodes
(2) the wave speed
(3) the wavelength
A. (1) only
B. (3) only
C. (1) and (2) only
D. (2) and (3) only

MC
Chapter 28 Stationary Waves

17.

(MC)28P5

Out of syllabus

(04)
A vibrating tuning fork is held near the mouth of a glass tube as shown.

Which of the following deductions is INCORRECT if resonance of the air column is observed?
A.
B.
C.
D.
18.

The observed loudness is greater than just with the fork alone.
There is a longitudinal stationary wave set up inside the tube.
There is a pressure antinode at the closed end of the tube.
The fundamental frequency of the air column in the tube must be equal to the frequency of the tuning fork.

(05)

P
2 mm
X

45 cm

Two identical sinusoidal waves, travelling in opposite directions along string XY fixed at both ends, form a
stationary wave. The separations between X and Y is 45 cm. The above figure shows the shape of the string at an
instant when the antinode P attains its maximum displacement of 2 mm form the equilibrium position. What is the
amplitude and wavelength of each of the travelling wave on the string ?

A.
B.
C.
D.
19.

Wavelength
30 cm
15 cm
30 cm
15 cm

Amplitude
1 mm
1 mm
2 mm
2 mm

(05)

Metal plate

Transmitter

Q
P
Probe detector

A microwave transmitter emits plane-polarized waves of frequency 1.0 u 1010 Hz which are reflected from a large
metal plate as shown. When a probe detector is moved between P and Q, the intensity of the waves detected
varies along the line PQ. Which of the following statements is/are correct ?
(1) The variation in intensity is due to interference of waves.
(2) The separation between consecutive positions of minimum intensity is 3 cm.
(3) If the transmitter is rotated 90 about the line PQ , a position of minimum intensity will become a position
of maximum intensity.
A. (1) only
B. (3) only
C. (1) & (2) only
D. (2) & (3) only

MC
Chapter 28 Stationary Waves

(MC)28P6

Out of syllabus

20.

(06)
A tuning fork of frequency 512 Hz is held over the mouth of a closed tube. If the speed of sound in air is 338 m
resonance will occur when the length of the air column in the tube is approximately
A.
8.3 cm.
16.5 cm.
B.
33.0 cm.
C.
41.5 cm.
D.

21.

(07)
Out of syllabus
Two pipes of lengths 42.6 cm and 41.5 cm respectively, each closed at one end, give beats of frequency 5 Hz
when sounding their fundamental notes together. Neglecting end corrections, find the speed of sound in air.
A. 331 m s -1
B. 326 m s -1
C. 321 m s -1
D. 316 m s -1

Answer
1. A
11. B
21. C

2. A
12. B

3. E
13. C

4. B
14. B

5. E
15. C

6. A
16. C

7. C
17. D

8. B
18. A

9. C
19. A

10. A
20. B

MC
Chapter 29 Acoustics
1.

2.

(MC)29P1

Out of syllabus
(80)
A man stands beside a railway track as a train approaches and then passes him, sounding its whistle all the time.
The true frequency of the whistle is 99 Hz. The highest note the man hears is of frequency 110 Hz. What is the
frequency of the lowest note which he hears ?
A. 88 Hz
B. 89 Hz
C. 90 Hz
D. 99 Hz
E.
100 Hz
(83)
Out of syllabus
An ambulance with its siren sounding a note of constant frequency fs drives along straight road. Which of the
graphs below best represents the variation of frequency f heard by a man standing close to the road as the
ambulance drives past ?
A. f
B.
C.
f
f
fs

fs

fs

time

time
f

D.

E.

fs

time

f
fs

time

time

3.

(84)
Out of syllabus
A sound reproduction system produces a sound level of 95 dB above threshold. Assuming the threshold of hearing
to be 10-12 W m-2, 95 dB corresponds to an intensity of
A. 5.6 u 10-8 W m-2.
B. 0.00316 W m-2.
C. 0.0316 W m-2.
D. 1.12 W m-2.
E.
6.3 u 104 W m-2.

4.

(85)

normal
air
water
ultra-sound
beam

A beam of ultrasound is being emitted from a submarine under water towards the water surface. Which of the
following statements is true ?
A. The refracted beam leaving the surface will bend away from the normal.
B. The refracted beam will bend towards the normal.
C. The refracted beam will travel in the same direction as the incident beam.
D. Total internal reflection will occur.
E.
The refracted beam will travel along the water surface.
5.

(85) Out of syllabus


A sound source S approaches an observer O at a constant speed v. Which of the following statements is/are
correct?
(1) The frequency of the source as heard by O increases linearly with time as S approaches O.
(2) The sound appears louder and louder as S approaches O.
(3) The wavelength of the wave appears to be shorter as S approaches O.
A. (1) only
B. (3) only
C. (1) & (2) only
D. (2) & (3)only
E. (1), (2) & (3)

MC
Chapter 29 Acoustics
6.

7.

(MC)29P2

Out of syllabus
(86)
An aeroplane flies horizontally at a low altitude with a constant speed of 300 m s-1. It transmits a radio signal of
frequency 30 MHz as it passes a receiving station. What is the difference in the frequencies received a long time
before and a long time after the passage of the plane ?
A. 10 Hz
B. 30 Hz
C. 60 Hz
D. 120 Hz
E. 150 Hz
Out of syllabus
(86)
If two independent sources, each separately at a noise level of 70 dB, are sounded together, they will produce a
noise level of
A. 35 dB.
B. 70 dB.
C. 73 dB.
D. 90 dB.
E.
140 dB.

8.

(87) Out of syllabus


If 3/4 of the sound energy produced by a typewriter is absorbed by a sponge rubber pad placed underneath, the
sound level produced will fall by
A. 0.25 dB.
B. 0.75 dB.
C. 3.00 dB.
D. 6.00 dB.
E.
12 dB.

9.

(87) Out of syllabus


Two identical sound sources send out sound waves to an observer. One of the sources is now moved away from
the observer. The observer then hears alternate loud and weak signal, and loud signals are detected whenever the
source moves through a distance of 0.50 m. If the speed of the source is 1/10 of the speed of sound, the
wavelength emitted by the stationary source is
A. 0.25 m.
B. 0.45 m.
C. 0.50 m.
D. 0.55 m.
E.
1.00 m.

10.

(88) Out of syllabus


If the sound level of a source increases by 6 dB, the power emitted by the source will have its initial value
multiplied by approximately
A. log10 6.
B. 2.
C. 4.
D. 6.
E.
106.

11.

(88)
Out of syllabus
A moving train sounds a whistle of frequency 500 Hz. The apparent frequency heard by an observer standing
close to the railroad is 462 Hz. If the speed of sound in air is 300 m s-1 , the train is moving at a speed of
A. 23 m s-1 away from the observer.
B. 23 m s-1 towards the observer.
C. 25 m s-1 away from the observer.
D. 25 m s-1 towards the observer.
E.
28 m s-1 away from the observer.

MC
Chapter 29 Acoustics

(MC)29P3

12.

(89)
Out of syllabus
If the threshold of hearing is 10-12 W m-2, a sound level of one microwatt per m2 is above threshold by
A. 6 dB.
B. 11 dB.
C. 60 dB.
D. 110 dB.
E.
120 dB.

13.

(89) Out of syllabus


A stationary radar source emits waves of frequency f, and wavelength O which are reflected from an object
moving towards the source at a speed u. The reflected waves reaching a receiver standing near to the radar source
will have an apparent wavelength of
A. O 2u/f.
B. O u/f.
C. O.
D. O + u / f.
E.
O + 2 u / f.

14.

(90)
Which of the following represent the approximate noise levels
(i) in a quiet school library ?
(ii) near the airport when an aircraft is taking off and flying overhead ?
(i)
(ii)
A. 30 dB
60 dB
B. 60 dB
90 dB
C. 30 dB
90 dB
D. 90 dB
60 dB
E.
60 dB
30 dB

15.

(90)
Out of syllabus
Sound waves of frequency f are emitted by a source S. When S is moved with speed u (relative to the ground)
towards a stationary observer O a rise in pitch of 'f is detected. Which of the following statements is/are correct ?
(1) The speed of sound waves relative to the observer is unaffected by the motion of S.
(2) If both S and O move in the same direction with speed u, no rise in pitch will be detected.
(3) If S is at rest with O moving towards it at speed u, the rise in pitch will also be 'f.
A. (1) only

B. (3) only

C. (1) & (2) only

D. (2) & (3)only

E. (1), (2) & (3)

16.

(91)
Out of syllabus
A transistor radio produces a sound of intensity level 40 dB at a point 5 m away from it. What is the intensity
level in decibels 20 m from the radio, which may be regarded as a point source ?
A. 10 dB
B. 28 dB
C. 30 dB
D. 34 dB
E. 40 dB

17.

(91)
Ultrasonic waves are used instead of audible sound waves to measure the depth of the sea because
(1) ultrasonic waves will not be interfered with by other sound waves in the water.
(2) ultrasonic waves can penetrate a greater depth of water with less spreading-out of waves.
(3) small-sized objects can also be located.
A. (1) only

B. (3) only

C. (1) & (2) only

D. (2) & (3)only

E. (1), (2) & (3)

MC
Chapter 29 Acoustics

(MC)29P4

18.

(92) Out of Syllabus


A loudspeaker produces a sound intensity level of 80 dB at a point P. If the electrical power to the loudspeaker is
halved, the intensity level at P will be
A. 20 dB
B. 30 dB
C. 40 dB
D. 50 dB
E.
77 dB

19.

(93)
Out of syllabus
A hydrogen source in a laboratory emits a line spectrum with one of the lines having wavelength 656.3 nm. For a
star receding from the earth at a speed of 200 km s-1, what would be the wavelength of the corresponding line
from the star observed on the earth ?
A. 655.5 nm
B. 655.9 nm
C. 656.7 nm
D. 657.1 nm
E.
657.5 nm

20.

(93) Out of syllabus


A radio produces a sound intensity level of 50 dB at a point 2 m away from it. If the power output of the radio is
doubled, what is the sound intensity level at a point 6 m from the radio ?
(You may regard the radio as a point source.)
A. 30.9 dB
B. 43.5 dB
C. 46.5 dB
D. 48.2 dB
E.
49.7 dB

21.

Out of syllabus
(94)
An ambulance, sounding its siren to produce a note of 800 Hz, approaches a stationary pedestrian P at a steady
speed of 40 m s-1. Calculate the frequency of the sound heard by P. (Speed of sound in air = 340 m s-1)
A. 706 Hz
B. 716 Hz
C. 800 Hz
D. 894 Hz
E.
907 Hz

22.

(95) Out of syllabus


An aircraft flies near the earth's surface over a stationary observer on a windless day. The frequency of the notes
from the engine received by the observer is 300 Hz when approaching, and becomes 150 Hz when leaving.
Assume the speed of sound in air to be 336 m s-1. The speed of the aircraft is
A. 56 m s-1
B. 84 m s-1
C. 112 m s-1
D. 168 m s-1
E.
224 m s-1

23.

Out of syllabus
(95)
Two loudspeakers are connected to the same signal source of negligible impedance. At a point equidistant from
the two loudspeaker, a maximum intensity of intensity level 60 dB is detected. If one loudspeaker is disconnected,
the intensity level at that point will be
A. 57 dB
B. 54 dB
C. 46 dB
D. 30 dB
E.
15 dB

MC
Chapter 29 Acoustics

(MC)29P5

24.

(96) Out of syllabus


The 3-cm microwaves are used in a radar speed trap. When the emitted and reflected signals are superposed, beats
are produced. Find the minimum beat frequency above which a driver would be prosecuted for exceeding a speed
limit of 100 km h-1.
A. 1000 Hz
B. 1900 Hz
C. 2400 Hz
D. 3400 Hz
E.
4700 Hz

25.

(97) Out of syllabus


A moving ship sounds a foghorn and an echo is received from a cliff behind it. Which of the following wave
characteristics of the original sound is/are different from that of the echo ?
(1) frequency
(2) amplitude
(3) speed
A. (1) only

26.

27.

28.

B. (3) only

C. (1) & (2) only

D. (2) & (3)only

E. (1), (2) & (3)

(99) Out of syllabus


On a straight road, a car and an ambulance moves with uniform speeds u1 and u2 respectively (with u2 > u1) in the
same direction. Initially the ambulance is behind the car and it overtakes the car after some time. The siren on the
ambulance continuously emits a sound of a certain frequency. What is the ratio of the wavelength of sound
received by an observer in the car before the ambulance overtakes to that after the ambulance overtakes ?
(Given : velocity of sound in air = v )
v  u2
v  u2
u  u1
v  u 2  u1
v  u 2  u1
B.
C. 2
D.
E.
A.
v  u2
v  u2
u 2  u1
v  u 2  u1
v  u 2  u1
(99) Out of syllabus
Students A and B are standing 5 m apart. A shouts to B and the intensity level in db recorded by B is x. A then
walks away from B so that they are 10 m apart. If A shouts again with the same power, what is the intensity level
in db recorded by B ?
x
1
1
x
B.
x
C. log10
D. x 3
E. x 6
A.
4
4
2

(00)

Out of syllabus

A dot vibrator is moving across the surface of water in a ripple tank with a steady speed as shown. The figure
represents the water surface showing the pattern of the water waves against a background of centimetre squares.
The speed of the water waves is 20 cm s-1. What is the speed of the dot vibrator ?
A. 10 cm s-1
B.8 cm s-1
C. 5 cm s-1
D. 4 cm s-1
E.2 cm s-1
29.

(01)
Out of syllabus
What is the intensity ratio of a note of 60 dB compared with another note of 40 dB which has the same frequency?
A. 1.5
B. 15
C. 20
D. 100
E. 200

MC
Chapter 29 Acoustics
30.

(01)
Out of syllabus
A boy sitting at the rim of a rotating merry-go-round blows a whistle continuously with frequency fo. An
observer on the ground hears the sound of the whistle. Which of the graphs below gives the possible variation of
the observed frequency f of the sound in a period of revolution T?
A.

B.

f
f0

f0

time

D.

time

time

E.

f0

f0

32.

C.
f

f0

31.

(MC)29P6

time

time

(02)
Out of syllabus
When a source emitting sound waves of a fixed frequency moves towards a stationary observer in air, the pitch of
the sound heard by the observer is different from that when the source is at rest. This is due to a change in the
(1) number of waves emitted per second from the source.
(2) speed of sound in air.
(3) wavelength of sound in air.
A. (1) only
B. (3) only
C. (1) and (2) only
D. (2) and (3) only

Out of syllabus
(02)
The spectra produced by element X, one on earth and the other on a distant star, are compared.
Spectrum form element X on earth:

Red

Green

Violet

Spectrum from element X on the distant star:

What is the most probable reason for the difference in the spectra readings?
A. The universe is expanding.
B. The star is approaching the earth.
C. The star is receding from the earth.
D. The star is spinning about its axis.
33.

(03) Out of syllabus


The average sound intensity level of human speech is about 70 dB. When four people are talking at the same
time, what would be the approximate sound intensity level?
A. 71 dB
B. 74 dB
C. 76 dB
D. 130 dB

Answer
1. C
11. C
21. E
31. B

2. D
12. C
22. C
32. D

3. B
13. A
23. B
33. C

4. B
14. C
24. B
34.

5. D
15. C
25. C
35.

6. C
16. B
26. A
36.

7. C
17. E
27. E
37.

8. D
18. E
28. A
38.

9. B
19. C
29. D
39.

10. C
20. B
30. D
40.

MC
Chapter 30 Optical instruments

(MC)30P1

1.

(80)
For which of the following object distances would a converging lens of focal length 30 cm form a real magnified
image of the object?
A. 7.5 cm
B. 15 cm
C. 22.5 cm
D. 40 cm
E.
60 cm

2.

(81)
15 cm

object

A converging lens of focal length 15 cm is used as a magnifying glass with the final image at infinity.
least distance of distinct vision is 25 cm, the angular magnification achieved is
A. 0
B. 1
C. 5/3
D. 15
E.
infinite
3.

(81)
The image of an object formed on a screen by a thin converging lens has height a. By moving the lens towards
the screen, a second lens position is found at which the height of the image formed on the screen is b. What is
the height of the object?
A.

4.

If the

1
( a  b)
2

B.

1
ab
2

C.

ab

D.

a3
b

E.

b3
a

(81)
E

eyepiece
focal length f e

objective
focal length f o

The diagram represents a microscope with the object at A. The image of A in the objective is formed at B.
Which of the following statements is correct?
(1) AO is less than fo.
(2) BE is less than fe.
(3) The image at B is real.
A. (1) only
B. (3) only
C. (1) and (2) only
D. (2) and (3) only
E. (1), (2) and (3)

MC
Chapter 30 Optical instruments

5.

(83)

(MC)30P2

screen
converging lens

2f

Two arrows are drawn as shown on a screen, placed at a distance 2 f from a converging lens of focal length f.
Which of the following diagrams correctly represents the image seen when the screen is viewed through the lens?
A.

6.

B.

C.

(84)

D.

E.

L
X
P

II IV

Q
V
III

The diagram shows two incoming parallel rays of light P and Q which pass through a thin converging lens L.
The ray XY after passing through the lens will pass through the point
A. I.
B. II.
C. III.
D. IV.
E.
V.
7.

8.

(84)
Out of syllabus
An astronomical telescope has an objective of focal length 40 cm, and an eyepiece of focal length 2 cm.
used to look at a distant object when its lenses are set 42 cm apart. the final image seen is
A. upright, virtual and at infinity.
B. inverted, virtual and at infinity.
C. upright, real and at infinity.
D. inverted, real and at infinity.
E.
upright, virtual and at the least distance of distinct vision.

(85)

It is

A converging lens L and a lamp S are arranged as shown. The rays from S converge at a point I after passing
through L. Which of the following operations could enable a parallel beam of light to emerge from the lens L?
(1) moving the lens L to the left until parallel rays are obtained
(2) replacing L by a lens with less converging power
(3) placing a diverging lens of suitable focal length in front of the lens L
A. (1) only
B. (3) only
C. (1) and (2) only
D. (2) and (3) only
E. (1), (2) and (3)

MC
Chapter 30 Optical instruments

9.

(MC)30P3

(85)
A train of plane wavefronts is incident upon a diverging lens. F denotes one of the focal points of the lens.
Which of the following diagrams represent(s) what might happen to the wavefronts?
(1)

(2)

(3)

A. (1) only

B. (3) only

C. (1) and (2) only

D. (2) and (3) only

E. (1), (2) and (3)

10.

(85)
Out of syllabus
In a compound microscope,
(1) the final image produced is virtual.
(2) the final image produced is erect.
(3) the focal lengths of both the objective and the eyepiece must be long in order to produce high
magnification.
A. (1) only
B. (3) only
C. (1) and (2) only
D. (2) and (3) only
E. (1), (2) and (3)

11.

(86)
Out of syllabus
A converging lens of focal length 12.5 cm is used as a magnifying glass with the final image formed at infinity.
If the least distance of distinct vision is 25 cm, the angular magnification is
A. 1/2.
B. 1.
C. 2.
D. 4.
E. infinite.

12.

(86)
A real object is placed in front of a concave lens of focal length f. Images are formed by the concave lens for
various object distances u. If the image distances are denoted by v, which of the graphs below shows the variation
of |v| with |u| ?
A. | v |
B. | v |
C. | v |

|f |

|f |

|f |
|u|

|u|

|f |
D.

|f |
E.

|v|

|v|

|f |

|f |
|u|
|f |

|u|
|f |

|u|
|f |

MC
Chapter 30 Optical instruments

13.

(87)

(MC)30P4

I
O

A real image I of an object O is formed by a converging lens L as shown above. A diverging lens L1 is placed
between L and I such that a real image I is obtained. Which of the following statements is/are correct?
(1) I is also an inverted image.
(2) I is larger than I.
(3) I is further away from L than I.
A. (1) only
B. (3) only
14.

C. (1) and (2) only

D. (2) and (3) only

E. (1), (2) and (3)

(87) Out of syllabus

A distant object consisting of two arrows is viewed through an astronomical refracting telescope consisting of two
converging lenses. Which of the following corresponds to the image seen?
A.
B.
C.
D.
E.

15.

(87)

O
I

Rays from a point light source O are refracted by a convex lens L and converge to a point I as shown above. Which
of the following operations would enable a parallel beam of light to be refracted from L ?
(1) moving the lens L towards O
(2) replacing L by a convex lens of shorter focal length
(3) placing a converging lens of suitable focal length between 0 and L
A. (1) only
B. (3) only
C. (1) and (2) only
D. (2) and (3) only
E. (1), (2) and (3)

16.

(90)
Out of syllabus
In a simple astronomical telescope, under normal adjustment, which of the following statements is/are correct?
(1) The first image is formed at the focal plane of the objective.
(2) The first image is real and inverted.
(3) The focal length of the objective is longer than that of the eyepiece.
A. (1) only
B. (3) only
C. (1) and (2) only
D. (2) and (3) only
E. (1), (2) and (3)

MC
Chapter 30 Optical instruments

(MC)30P5

17.

(91)
Out of syllabus
A simple two-convex lens refracting telescope has a magnifying power of 12.5 when the telescope is in normal
adjustment. The focal length of the objective is 0.75 m. The separation between the objective and the eye piece is
A. 0.06 m
B. 0.69 m
C. 0.81 m
D. 1.35 m
E.
16.7 m

18.

(92)
The image of an object formed on a screen by a convex lens has height a. By moving the lens towards the
screen, it is found that there is a second lens position at which another image of height b is formed on the screen.
The height of the object is
A. (a + b) /2

19.

B.

a2  b2

C.

ab

D.

a3 / b

b / a3
E.
Out of syllabus
(92)
The sun subtends an angle of 0.5 at the surface of the Earth. A convex lens of focal length 100 cm is used to form
an image of the sun onto a screen. The diameter of the image is about
A. 1 mm
B. 3 mm
C. 5 mm
D. 9 mm
E.
50 mm

20.

(92)
Out of syllabus
In an astronomical telescope set at normal adjustment, the focal lengths of the objective and the eyepiece are 50
cm and 10 cm respectively. Which of the following gives the separation of the lenses and the angular
magnification of the telescope?
Lens separation
Angular magnification
A.
30 cm
5
B.
40 cm
0.5
C.
40 cm
5
D.
60 cm
0.5
E.
60 cm
5

21.

(93)
Out of syllabus
An object placed in front of a magnifying glass forms an image at infinity with magnifying power 3. What is the
focal length of the magnifying glass? (You may assume the least distance of distinct vision to be 25 cm)
A. 6.3 cm
B. 8.3 cm
C. 12.5 cm
D. 25 cm
E.
75 cm

MC
Chapter 30 Optical instruments

22.

(MC)30P6

(94)
Which of the following ray diagrams is/are correct? (F is the focus of the corresponding optical instrument.)
(1)

(2)

(3)

A. (1) only

B. (3) only

C. (1) and (2) only

D. (2) and (3) only

E. (1), (2) and (3)

23.

(95)
When an object placed far away from a convex lens is gradually moved towards the lens, the separation between
the object and its real image will
A. decrease.
B. decrease and then increase.
C. increase.
D. increase and then decrease.
E.
remain unchanged.

24.

(95)
Out of syllabus
An astronomical refracting telescope consists of two converging lenses of focal lengths 100 cm and 5 cm. Under
normal adjustment, it is used to observe a distant object which subtends an angle of 0.2 when viewed directly.
Which of the following statements is/are correct?
(1) The lens with focal length 5 cm should be the objective.
(2) The height of the first image formed by the telescope is 3.5 mm.
(3) The angle subtended by the final image seen by the observer is 4.
A. (1) only
B. (3) only
C. (1) and (2) only
D. (2) and (3) only
E. (1), (2) and (3)

25.

(96)
Out of syllabus
The image of moon is focused on a screen by a converging lens of focal length 20 cm. If the moon subtends an
angle of 9.5 u10-3 radian at the center of the lens, calculate the diameter of the image.
A. 4.8 u 10-2 cm
B. 9.5 u 10-2 cm
C. 1.9 u 10-1 cm
D. 3.8 u 10-1 cm
E.
7.6 u 10-1 cm

MC
Chapter 30 Optical instruments

26.

(96)

(MC)30P7

Out of syllabus

50 cm

A beam of parallel light rays is incident on a convex lens. The light rays then refract and pass through a concave
lens as shown in the figure. If the focal length of the convex lens is 80 cm and the separation between the two
lenses is 50 cm, what is the focal length of the concave lens ?
A. 20 cm
B. 30 cm
C. 40 cm
D. 50 cm
E.
60 cm
27.

(97)

Out of syllabus

The above diagram shows two light rays from a point source O passing in turn through a convex lens L1 and a
concave lens L2. Which of the following is true of the focal lengths of the lenses?
Focal length of L1
Focal length of L2
A.
5 cm
less than 10 cm
B.
10 cm
less than 10 cm
C.
10 cm
10 cm
D.
5 cm
greater than 10 cm
E.
10 cm
greater than 10 cm

28.

29.

(99) Out of syllabus


An astronomical refracting telescope is adjusted to view a distant object under normal adjustment. Which of the
following statement is/are correct?
(1) The first image formed by the objective is real and is smaller than the object.
(2) The final image formed by the eyepiece is at the least distance of distinct vision from the eye.
(3) If part of the objective is blocked by an opaque obstacle, part of the final image could not be viewed.
A. (1) only
B. (3) only
C. (1) and (2) only
D. (2) and (3) only
E. (1), (2) and (3)
(00)
L
P
X

Y
S

In the above figure, XY is the principal axis of a lens L. PQ and OS are two refracted rays from L which originate
from a point object placed on the left side of L. Which of the following deductions is/are correct?
(1) The lens L must be a concave lens.
(2) The point object must lie along the line OS.
(3) The image of the point object must be virtual.
A. (1) only
B. (3) only
C. (1) and (2) only
D. (2) and (3) only
E. (1), (2) and (3)

MC
Chapter 30 Optical instruments
30.

31.

(MC)30P8

(01) Out of syllabus


The focal length of a concave mirror is f. The mirror will produce a real, inverted and diminished image when the
distance of the object from the pole of the mirror is
A. greater than 2f.
B. equal to 2f.
C. between f and 2f.
D. equal to f.
E.
less than f.
(02) Out of syllabusL1

L2

20 cm

In the figure, L1 and L2 are two thin lenses with the same focal length 40 cm placed coaxially 20 cm apart. A beam
of light originating from a distant object is incident on L1. Which of the following statements about the final
image formed by the system of lenses is correct?
A. It is real and formed on the right of L2.
B. It is real and formed between L1 and L2.
C. It is virtual and formed between L1 and L2.
D. It is virtual and formed on the left of L1.
32.

(03)
In the following ray diagrams, L is a convex lens and F is one of its principal foci. Which of the ray diagrams
is/are possible?
(1)
(2)
(3)
L

F
F

A. (1) only
33.

B. (3) only

C. (1) and (2) only

D. (2) and (3) only

(03) Out of syllabus


Typical microscopes and telescopes are both built from two converging lenses. In which of the following ways is
a telescope similar to a microscope when both are in normal adjustment?
A.
B.
C.
D.

In each the focal length of the objective lens is shorter than that of the eyepiece.
In each the separation of the lenses is equal to the sum of their focal lengths.
Each produces an intermediate image which is linearly magnified and inverted.
In each the final image is inverted and virtual.

MC
Chapter 30 Optical instruments

34.

(MC)30P9

(04)
In the figure, P and Q are two monochromatic light rays. One is red and one is blue. They are parallel to the
principal axis of the glass lens L before being refracted by the lens.
L

Which of the following deductions are correct?


(1)
(2)
(3)
A.
B.
C.
D.
35.

P is the red light ray.


L is a concave lens.
Q travels faster than P in the lens.

(1) and (3) only


(1) and (2) only
(2) and (3) only
(1), (2) and (3)

(05)

A convex lens of focal length 20 cm forms a diminished image of an object (letter F ) on a translucent screen, as
shown. Which of the following deductions is/are correct ?
(1) The separation of the lens and the screen is less than 40 cm.
(2) The separation of the object and the screen is greater than 80 cm.
(3) Keeping the positions of the object and the screen unchanged, another sharp image can also be formed on
the screen if the lens is moved nearer to the object.
A. (1) and only
B. (1) and (2)only
C. (2) and (3) only
D. (1),(2) and (3)
36.

(07)
An object is placed at the focus of a concave lens of focal length 10 cm. What is the magnification of the image
formed ?
A. 0.5
B. 1.0
C. 2.0
D. infinite

Answer
1. D
11. C
21. B
31. A

2. C
12. A
22. B
32. B

3. C
13. E
23. B
33. D

4. D
14. A
24. D
34. B

5. D
15. A
25. C
35. D

6. B
16. E
26. B
36. A

7. B
17. C
27. D

8. E
18. C
28. A

9. B
19. D
29. D

10. A
20. E
30. A

MC Question
Chapter 11 Electric field
1.

(MC)11P1

(80)
Given : 1/4SHo = 9 u 109 m F-1, and the relative permittivity of air = 1.00.
An isolated metal sphere has a total charge of 1.2 u 10-8 C. The insulation of air breaks down when, the electric
field strength is 3 u 106 V m-1. What is the smallest possible radius of the sphere ?
A. 1.7 u 10-13 m

1.8 u 10-8 m

B.

C. 6.0 u 10-6 m

D. 3.6 u 10-5 m

E. 6.0 u 10-3 m

2.

(80)
A charged ball X is suspended by a string. When a uniform electric field E is
applied horizontally, the ball is displaced a horizontal distance a such that the
string makes an angle T with the vertical. Which of the following statements is
correct?
A. E v a for all values of T
B. E v T for all values of a
C. E v cos T for all values of a
D. E v tan T for small values of a only
E.
E v sin T for small values of T only

3.

(81)
A potential difference V is applied between two large parallel plates, distance s apart. An electron of mass m and
charge -e starts from rest at the negative plate and travels across the gap to the positive plate. The time taken is
A.

4.

2s
V

B.

s2
V

C.

(82)
v

2s 2
eV

D.

2ms 2
eV

E.

m
2eV

5
4

1
E

2 3

The diagram shows an electron in a uniform, vertical electric field E. At the instant shown, the electron is
travelling in the direction of the arrow v. Which of the arrows 1 to 5 gives the direction of the acceleration of the
electron at this instant?
A. 1
B. 2
C. 3
D. 4
E. 5
5.

(83)
Two parallel metal plates are placed horizontally with a separation of 0.05 m. A p.d. of 2.0 kV is connected across
the plates. An oil drop with a charge of -1.6 u 10-19 C is observed to remain at rest between the plates. The mass
of the drop is
A. 4.0 u 10-25 kg.
B. 1.6 u 10-18 kg.
C. 1.6u 10-16 kg.
D. 6.4 u 10-16 kg. E. 6.4u 10-14 kg.

6.

(83)
Which of the following statements about the coulomb is/are correct?
(1) When one coulomb of charge flows across a potential difference of one volt, one joule of energy is
released.
(2) The force exerted on a charge of 1 coulomb in an electrostatic field of 1 volt/metre is 1 newton.
(3) 1 coulomb is the charge on 1 mole of electrons.
A. (1) only
B. (3) only
C. (1) & (2) only
D. (2) & (3) only
E. (1), (2) & (3)

MC Question
Chapter 11 Electric field
7.

(MC)11P2

(84)
Y
X

The diagram shows a pattern of electric field lines in which X, Y and Z are points marked on one of the field lines.
It would be correct to say that
(1) X is at a higher potential than Z.
(2) a negative charge placed at Z would accelerate to the left along the tangent to the field line at Z.
(3) the force exerted on a charge at Y would be greater than if the charge were placed at X.
A. (1) only
B. (3) only
C. (1) & (2) only
D. (2) & (3) only
E. (1), (2) & (3)
8.

insulting thread

(85)

+
source of
high voltage

_
A conducting ball is suspended between two metal plates connected through a sensitive centre-zero galvanometer
to a source of high p.d. The ball shuttles back and forth between the plates, making alternate contacts with each
plate. Which of the following statements is/are correct?
(1) The ball carries charges, sometimes positive, and sometimes negative.
(2) If the separation of the metal plates decreases, the frequency of oscillation of the ball increases.
(3) The galvanometer shows a current flowing always in the same direction.
A. (1) only
B. (3) only
C. (1) & (2) only
D. (2) & (3) only
E. (1), (2) & (3)
9.

(85)
Which of these statements concerning electric potential is INCORRECT ?
A. Potential is a scalar quantity.
B. The potential difference between two points, expressed in volts, is numerically equal to the change in the
energy, expressed in joules, when a coulomb of charge is moved from one point to the other.
C. The potential is zero whenever the electric field is zero.
D. The potential due to a point charge varies as 1/r, where r is the distance from the point charge.
E.
The potential gradient at a point is proportional to the strength of the electric field at that point.

10.

(86)
Given: permittivity of free space = 8.85 u 10-12 F m-1. A conducting sphere of radius 0. 1 m carries a positive
charge of 10-4 C. A particle P of mass 2 u 10-5 kg carrying a negative charge of 10-10 C is released from rest at a
distance of 1 m from the centre of the sphere. If the force due to gravity is neglected, the velocity of P when it
strikes the surface of the sphere will be
B. 9.0 m s-1
C. 29.8 m s-1
D. 81.0 m s-1
E. 890 m s-1
A. 2.8 m s-1

11.

(87)
Three charge +Q, -Q and -Q are fixed at the corners W, X and Y respectively of a square as
shown. A fourth charge, q, is fixed at Z, after which the charge at X experiences a NET
electrostatic force indicated by the arrow. q is equal to
A. + Q.

B. + 2Q.

C. + 4 Q.

D. +

2 Q.

+Q

E. +2 2 Q.
q

-Q

Y
-Q

MC Question
Chapter 11 Electric field
12.

13.

14.

(MC)11P3

(88)
The diagram shows points of equal potential joined as equipotential lines.
Which of the following statements is/are correct ?
(1) The electric field at P is in a direction tangential to the line
passing through P.
(2) The electric field is the same at the points P and Q.
(3) Work has to be done in moving an electron from point P to
point R.
A. (1) only
B. (3) only
C. (1) & (2) only
D. (2) & (3) only

- 400 V
- 300 V
R
Q

- 200 V
- 100 V

E. (1), (2) & (3)

(88)
Four particles carrying charges +q, +q, -2q and -2q are placed at the vertices of a square of side a. The electric
potential at the centre of the square is
A. zero.
-2 q
+q
a
 2 q /( 2SH o a )
B.
C.

 2 q /(SH o a)

D.

 2 q /(2SH o a)

E.

 2 q /(SH o a )

a
+q

(88)

-2 q

+
h

vertical
_

A potential difference V is maintained between plates X and Y, separated by a distance h. A particle of mass m and
positive charge q enters the region between X and Y through the hole H with negligible velocity. If it makes no
collisions on the way, it will strike X with kinetic energy
A. mgh.
B. Q V.
C. qV + mgh.
D. qV mgh.
E. mgh - qV.
15.

(89)

IV

III

II
I

A particle carrying a negative charge is free to move in a uniform electric field E. If the particle starts with a
certain velocity from point X, which of the paths shown could represent the route which the particle would follow
from X to Y?
A. I and II only B. I and IV only
C. II and III only
D. III and IV only
E. I, II, 111 and IV
16.

(89)

+
M

Two large metal plates A, B are oppositely charged and placed a small distance apart. A piece of metal M is
placed centrally between the plates. Which of the following graphs shows the variation of electric field intensity E
from A to B?
E
A. E
B. E
C. E
D. E
E.

MC Question
Chapter 11 Electric field
17.

(MC)11P4

small hole

(90)

+Q

18.

The arrangement above shows two concentric hollow metal spheres of inner radius b and outer radius a. A charge
+Q is given to the inner sphere and the outer sphere is earthed. What is the work done in bringing a small positive
charge q from infinity to the surface of the inner sphere?
Qq 1 1
Qq
Qq
Qq
C.
D.
E.
A. zero
B.

4SH o b a
4SH o a
4SH o b
4SH o (a  b)
(90)

A light conducting sphere is hanged from a long insulating thread between oppositely charged metal plates
connected to a high voltage supply. If the sphere is given a positive charge, it will
A. move to the positive plate and stick to it.
B. move to the negative plate and stick to it.
C. remain still.
D. oscillate, touching each plate in turn, beginning with the negative plate.
E.
oscillate, touching each plate in turn, beginning with the positive plate.
19.

(90)
P2
b
P1
b
b

A uniformly charged wire has the form of a circular loop of radius b. P1 and P2 are two points on the axis of the
loop. P1 is at a distance b from the loop centre and P2 is at a distance 2b from the loop centre. At P1, P2, the
potentials are V1 , V2 respectively. V2/ V1 is equal to
A. 1 / 3.
20.

B. 2 / 5.

C. 1 / 2.

D.

E. 4S.

2/ 5

(91)
Which of the graphs below best represents the variation of electrical potential V with distance r from the centre of
a charged hollow metal sphere of external radius a ?
A.
B.
C.
D.
E.
V

r 0

MC Question
Chapter 11 Electric field

(MC)11P5

21.

(92)
How do the two physical quantities below change along the direction indicated by an electric field line from a
point positive charge?
(1) electric field intensity E
(2) potential V
A. Only E will increase
B. Only V will increase
C. Both E and V will increase
D. Both E and V will decrease
E.
Both E and V will remain constant

22.

(93)

a
b
B
A

A positively-charged metal sphere A of radius a is joined by a conducting wire to an uncharged metal sphere B of
radius b placed far away from the first sphere as shown in the diagram. The ratio of the surface charge density on
sphere A to that on sphere B is
B. b2/a2

A. b/a
23.

C. a / b

D. a2/ b2

E.

b/ a

(93)
Two insulated parallel metal plates are connected to the terminal of an EHT. When a charged aluminium foil strip
is placed between the plates, deflection of the foil is observed as shown.
polystyrene
rod
metal
plate

metal
plate
aluminium
foil strip
_
+
EHT

Which of the following statements is/are true ?


(1) The charge on the foil is negative.
(2) Deflection of the foil increases if the separation between the plates decreases.
(3) When moving the foil towards the positive plate, the deflection of the foil increases.
A. (1) only
B. (3) only
C. (1) & (2) only
D. (2) & (3) only
E. (1), (2) & (3)
24.

(93)
An uncharged metal sphere is placed in a uniform electric field. Which of the following best represents the
electric field pattern around the metal sphere ?
A.
B.
C.
D.
E.

25.

(94)
X and Y are two different points in an electric field. A small charged object is released from rest at X. Which of
the following conditions would ensure that the charged object will NOT pass through Y?
A. The electric field at Y is zero.
B. The electric field at Y is stronger than that at X.
C. The electric field between X and Y is not zero.
D. The electric potentials at X and Y are equal.
E.
The electric potential difference between X and Y is not zero.

MC Question
Chapter 11 Electric field

(MC)11P6

26.

(94)
The electric potential energy of a system of charges at infinitely large distances from one another is taken to be
zero. What is the electric potential energy stored in a system of four charges, each of +1 C, placed at the vertices
of a square with length of side 1 m ? (Ho = permittivity of vacuum)
5
1
3
1
1
1
E.
B.
D.
C.
A.
(2 
)
(4  2 )
SH o
SH o
SH o
4SH o
4SH o
2

27.

(95)
A

A, B, C, D are four points on a straight line as shown in the diagram. A point charge +Q is fixed at A. When
another point charge -Q is moved from B to C, which of the following quantities will increase?
(1) The electric potential energy of the system of charges.
(2) The magnitude of the electric field strength at the point D.
(3) The electric potential at the point D.
A. (1) only
B. (3) only
C. (1) & (2) only
D. (2) & (3) only
E. (1), (2) & (3)
28.

(96)
The straight lines in the diagram represent electric field lines. Which of the following statements about this
electric field is/are correct ?
Q
P

(1) A stationary negative charge placed at Q tends to move to P.


(2) The electric field strength at P is stronger than that at Q.
(3) Work has to be done in moving a negative charge from R to P.
A. (1) only
B. (3) only
C. (1) & (2) only
D. (2) & (3) only
29.

E. (1), (2) & (3)

(96)
Five identical point charges, each of charge Q, are fixed evenly on a circle of radius r. How much work has to be
done to bring another point charge Q from infinity to the centre of the circle? (Ho = permittivity of free space)
A. zero

B.

5Q
4SH o r

C.

5Q 2
4SH o r

D.

5Q 2
4SH o r 2

E.

5Q
4SH o r 2

30.

(97)
A charged particle is accelerated across the gap between two parallel metal plates maintained at a certain potential
difference in a vacuum. Assuming there is no gravitational force, the energy acquired by the charged particle in
crossing the gap depends on
(1) the mass of the charged particle.
(2) the width of the gap.
(3) the potential difference between the plates.
A. (1) only
B. (3) only
C. (1) & (2) only
D. (2) & (3) only
E. (1), (2) & (3)

31.

(97)
A metal sphere is charged to a potential of 100 V. If the charge density on its surface is 6.0 u 10-9 C m2, find the
radius of the sphere.
(Given: permittivity of free space = 8.85 u 10-12 F m-1 )
A. 0.04 m
B. 0.15 m
C. 0.21 m
D. 0.35 m
E. 0.60 m

MC Question
Chapter 11 Electric field
32.

(MC)11P7

(98)

D-particle

D-particle

The above diagram shows two D-particles moving directly towards each other with the same speed. Which of the
following statements is/are correct?
(1) The two particles attract each other.
(2) The electric potential energy of the two particles is decreasing at the instant shown.
(3) The total momentum of the two particles remains constant in the subsequent motion.
A. (1) only
B. (3) only
C. (1) & (2) only
D. (2) & (3) only
E. (1),(2) & (3)
33.

(98)

In a cathode ray tube, the beam of electrons from the electron gun is deflected vertically when a p.d. is applied
between the deflection plates, P and Q, of length L and separation d. Which of the following changes will increase
the deflection of the beam for a given p.d. across the deflection plates?
(1) Reducing the p.d. V between the filament and the anode.
(2) Reducing the separation d between the deflection plates.
(3) Reducing the length L of the deflection plates.
A. (1) only
B. (3) only
C. (1) & (2) only
D. (2) & (3) only
E. (1), (2) & (3)
34.

(99)
Which of the following statements about electric field lines is incorrect?
A. They are closest where the field is strongest.
B. They are always perpendicular to equipotential lines.
C. They always point from high electric potential to low electric potential.
D. Work has to be done in moving an electron along the direction of a field line.
E.
They tend to attract one another.

35.

(00)

The figure shows a pattern of electric field lines in which P, Q and R are points marked on one of the lines with
PQ = QR. If the potential at P is 0 V, which of the following can give the possible potentials at Q and at R?
Potential at Q
Potential at R
A. - 200 V
- 450 V
B. - 200 V
- 400 V
C. - 200 V
- 350 V
D. + 200 V
+ 350 V
E.
+ 200 V
+ 450 V

MC Question
Chapter 11 Electric field
36.

(00)

+q

+Q

(MC)11P8

+2q
d

In the above arrangement, two small test charges +q and +2q are brought from infinity to the positions shown.
The two charges are collinear with another charge +Q and their mutual separation is d. Which of the following
statements is/are correct?
(1) Charge +q is higher potential than charge +2q.
(2) The work done in bringing the charges +q and +2q from infinity to their respective positions is the
same.
(3) The potential energy of the system would increase if d decreases.
A. (1) only

B. (3) only

C. (1) and (2) only

D. (2) and (3) only

E. (1), (2) and (3)

37.

(00)
Two parallel plates are connected to an E.H.T of 4.5 kV Electric breakdown occurs when the separation of the
plates is reduced to 1.5mm. Estimate the maximum acceleration of an electron between the plates.
(Given: charge of an electron = 1.6 u 10-19 C; mass of an electron = 9.1 u 10-31 kg)
A. 4.0 u 107 m s-2
B. 1.0 u 109 m s-2
C. 1.2 u 1012 m s-2
D. 1.6 u 1015 m s-2
E.
5.3 u 1017 m s-2

38.

(01)
Two insulated uncharged metal spheres X and Y are in contact with each other. A positively charged rod is
brought near X without any contact while sphere X is earthed as shown.
Y

++
++

At steady state, which of the following descriptions is/are correct ?


(1) Sphere X gains electrons.
(2) Sphere Y loses electrons.
(3) Sphere X acquires a negative electric potential.
(4) Sphere Y acquires a positive electric potential.
A. (1) only
B. (1) and (2) only
C. (1) and (3) only
D. (2) and (4) only
39.

E. (1), (2), (3) and (4)

(02)
X

In the above figure the solid lines represent part of an electric field due to a fixed point charge Q (not shown in
the figure). A charged particle q, subjected only to electric force in the field, travels along the dotted curve shown.
Which of the following conclusions can be drawn ?
A.
B.
C.
D.

q is travelling from X to Y.
The charge of q has the same sign as that of Q.
q has a greater speed at X than at Y.
The electric potential at X due to Q is higher than that at Y

MC Question
Chapter 11 Electric field
40.

(MC)11P9

(03)

B
A
In the figure, the solid curves are concentric circles representing a set of equipotential surfaces in an electric field.
The dotted curve ABC represents the path of a charged particle moving in the field. Which of the following
deductions from the figure is/are correct ? (Neglect the effects of gravity.)
(1) The charged particle is always repelled from the centre of the concentric circles.
(2) The speed of the charged particle at A is equal to that at C.
(3) The kinetic energy of the charged particle at B is greater than that at A.
A. (1) only
B. (3) only
C. (1) and (2) only
D. (2) and (3) only
41.

(03)

A
+
+
+
+
+
+

In the figure, A is an earthed metal plate while B is a positively charged metal plate parallel to A. The separation
between the plates is much less than the length of each plate. If A is moved closer to B, what would happen to the
electric potential of B and the electric field strength between the plates ? (Induced charges are not indicated.)

A.
B.
C.
D.
42.

Electric potential of B
increase
increase
decrease
decrease

Electric field strength


increase
remain unchanged
decrease
remain unchanged

(03)
In the figure, the dotted part represents a spherical metallic shell A of uniform thickness. The shell is neutral as a
whole. A small positively charged object B is placed inside the cavity of the shell but not exactly at its centre.
Positive and negative charges are then induced in the shell. (Distribution of induced charges is not indicated.)
Shell A

B
+

Cavity

Which of the following statements is/are correct when electrostatic equilibrium is reached?
(1) Positive induced charges are distributed uniformly on the outer surface of the shell.
(2) The electric field at any point within the cavity is zero.
(3) The electric potential at the outer surface of the shell is higher than that at its inner surface.
A. (1) only
B. (2) only
C. (1) and (3) only
D. (2) and (3) only

MC Question
Chapter 11 Electric field
43.

(04)

(MC)11P10

+Q
A

In the above figure, a point charge +Q is placed at A. The resulting electric potential at B is V. If a point
charge -2Q is now placed at C, the mid-point between A and B, what is the electric potential at B produced by
both point charges? (Assume that the electric potential at infinity is zero.)
A. -4V
B. -3V
C. -2V
D. 0
44.

(04)

60q
30q

A
Two positively charged particles A and B of masses mA and mB respectively are suspended by two insulating
threads of the same length from O. Due to the electrostatic repulsive forces between A and B, threads AO and
BO make 30q and 60q respectively with the vertical as shown. Find the ratio mA: mB.
A.
B.
C.
D.
45.

3:1

2: 3
2:1
It cannot be found as the charge on each particle is not known.

(04)
A positively charged particle is projected with a certain initial velocity into the electric field due to a charge fixed
at O. Part of its trajectory (curve AB) is sketched as shown. Which of the following deductions is correct?

A
O

B
A.
B.
C.
D.

46.

The trajectory must be a part of an ellipse.


The acceleration of the particle at B is greater than that at A.
The kinetic energy of the particle at B is greater than that at A.
The electrical potential energy of the particle at B is greater than that at A.

(05)
P, Q and R are small identical metal spheres. P and Q are fixed at a certain separation in vacuum and they carry
charges of the same magnitude. The attractive force between is F. Sphere R is initially uncharged. It first touches
P and then it touches Q. What is the electrostatic force between P and Q after R is taken away ?
A. F / 4
B. F / 8
C. 3F / 4
D.
3F/8

MC Question
Chapter 11 Electric field
47.

(MC)11P11

(05)
+
-

A negatively charged oil drop is kept stationary between two horizontal metal plates connected to a d.c. supply as
shown. The oil drop then acquires an additional negative charge. Which of the following changes will be able to
hold the oil drop stationary ?
(1) Disconnecting the plates from the supply and moving the plates closer
(2) Keeping the separation between the plates unchanged and increasing the p.d. between the plates
(3) Keeping the p.d. between plates unchanged and moving the plates further apart.
A. (1) only
B. (3) only
C. (1) & (2) only
D. (2) & (3) only
48.

(05)
The electric field strength near the surface of a charged metal sphere is E. If the charge on the sphere and its
radius are doubled, what is the new electric field strength near the sphere's surface ?
E
E
A.
B.
C. E
D. 2 E
4
2

49.

(06)
An insulated metal sphere S of radius 2r carries a charge +Q. Another insulated metal sphere of radius r carrying
a charge +2Q at infinity is connected to S through a conducting wire. What would the charge on S be after
removing the wire ?
A. +Q / 2
B. +Q
C. +3Q/2
D. +2Q

50.

(06)

-2PC

+2PC
P

Charges +2 PC and -2 PC are situated at points P and Q respectively. X is the mid-point of PQ. Which of the
following conclusions is/are correct ? (The electric potential at infinity is taken to be zero.)
(1) The electric field at X is pointing towards Q.
(2) The electric potential at X is zero.
(3) The electric field strength at X is the strongest among any point between P and Q.
A. (1) only
B. (1) & (2) only
C. (2) & (3) only
D. (1), (2) & (3)
51.

(06)
An insulated metal sphere S of radius 2r carries a charge +Q. Another insulated metal sphere of radius r carrying
a charge +2Q at infinity is connected to S through a conducting wire. What would the charge on S be after
removing the wire ?
A. +Q/2
B. +Q
C. +3Q/2
D. +2Q

Answer
1.E

2.E

3.D

4.E

5.D

6.C

7.E

8.E

9.C

10.B

11.E

12.B

13.B

14.C

15.A

16.B

17.E

18.D

19.D

20.C

21.D

22.A

23.C

24.A

25.D

26.E

27.C

28.C

29.C

30.B

31.B

32.B

33.C

34.E

35.A

36.E

37.E

38.A

39.C

40.C

41.D

42.A

43.B

44.A

45.D

46.B

47.B

48.B

49.D

50.B

51.D

MC Question
Chapter 13 Current Electricity
1.

(80)
P

3 k:

1 k:

(MC)13P1

15 V

A voltmeter reads 3 V when connected across QR in the above circuit. The internal resistance of the battery is
negligible. What is the resistance of the voltmeter?
A. 0.75 k:
B. 1.50 k:
C. 3.00 k:
D. 3.75 k:
E. 4.00 k:
2.

(80)
Torch bulbs marked 3 V, 1.5 W, are to be used in a circuit on a 6 V supply. What is the minimum number of
bulbs connected in the circuit if the steady current drawn from the 6 V supply is to be 2 A and each bulb is to be
fully lit?
A. 2
B. 3
C. 4
D. 8
E. 12

3.

(80)

L1

L2

L3

L4
2V

When the switch S is closed in the above circuit, all the lamps light up, but after the circuit has been used several
times, only L1 lights up. Which of the following possibilities would account for this?
(1) There is a short circuit across L4.
(2) The filament of L2 is burnt out.
(3) The filaments of both L2 and L4 are burnt out.
A. (1) only
B. (3) only
C. (1) & (2) only
D. (2) & (3) only
E. (1), (2) & (3)
4.

(82)
R1 = 1 :
Y

X
R2 = 3 :

In the circuit above, an electron travelling from Y to X through R1 loses energy E1, and an electron travelling from
Y to X through R2 loses energy E2. E1 is equal to
A. E2 / 4.
B. E2 / 3.
C. E2.
D. 3 E2 .
E. 4 E2.

5.

(83)
8:
X

In the above circuit, X and Y are identical cells, of e.m.f. 10 V and internal resistance 4 :. What is the current
passing through the 8 : resistor?
A. zero
B. 5/6 A
C. 1 A
D. 5/3 A
E. 2 A

MC Question
Chapter 13 Current Electricity

(MC)13P2

6.

(83)
A 10 V, 5 W light bulb in a set of Christmas tree lights (which consists of 20 bulbs in series) burns out and Jimmy
goes to buy a replacement. When he gets back, he finds that although the new bulb is marked 5 W, the light it
gives is very dim, although the other bulbs light up brightly. Which of the following is a possible reason for this?
(1) The supply voltage has dropped below 200 V.
(2) The current through the circuit is very much less than 0.5 A.
(3) The new bulb is designed to work at a voltage lower than 10 V.
A. (1) only
B. (3) only
C. (1) & (2) only
D. (2) & (3) only
E. (1), (2) & (3)

7.

(83)

2V

R
0.4 A

0.4 A
G
40 :

50 :

In the circuit shown, no current flows through the galvanometer G. If the internal resistance of the cell is
negligible, what is the value of R?
A. 0.89 :
B. 1.3 :
C. 4.4 :
D. 5.0 :
E. 5.3 :
8.

(84)

12 V

3:

5:
X

A high resistance voltmeter is connected between points X and Y in the circuit shown. When the switch S is open,
the reading on the voltmeter is approximately
A. 0 V.
B. 1.5 V.
C. 2.5 V.
D. 4.0 V.
E. 12 V.
9.

(85)
The e.m.f. of a battery is equal to
(1) the total electrical power it generates divided by the current it delivers.
(2) the total electrical energy it releases, per coulomb of charge when connected to an external circuit.
(3) the potential difference across its terminals when it is on open circuit.
A. (1) only
B. (3) only
C. (I) & (2) only
D. (2) & (3) only
E. (1), (2) & (3)

10. (86)
The figure below shows 4 resistors, of resistance P, Q, R and S in a Wheatstone bridge circuit. Which of the
following statements is/are correct when the bridge is balanced?
x
(1) The p.d. across R is equal to the p.d. across S.
P
S
(2) The potential at point x is equal to the potential at pointy.
(3) The current through G will still be zero when the battery
G
and the galvanometer are interchanged.
A. (1) only
R
Q
y
B. (3) only
C. (1) & (2) only
D. (2) & (3) only
B
E. (1), (2) & (3)
11. (86)
In the circuit shown, the equivalent resistance across XY is
A.
0 :. .
B.
10 :.
C.
20 :.
D.
40 :.
E.
64 :.

8:
8:
8:

8:

8:
Y

X
8:

8:
8:

MC Question
Chapter 13 Current Electricity

(MC)13P3

12. (86)
An electric current is flowing in a copper wire which is part of a circuit. The electrons in the copper wire
A. were at rest until the switch was closed but now move along the wire with a drift velocity of a few millimetres
per second.
B. were at rest until the switch was closed but now move along the wire at a speed approaching the speed of light.
C. had random motion onto which was imposed a drift velocity of a few millimetres per second when the switch
was closed.
D. had random motion onto which was imposed a drift velocity of a few kilometres per second when the switch
was closed.
E. had random motion until the switch was closed but now move along the wire at a speed approaching the speed
of light.
13. (87)

V
B

1:

3V

In the above figure, AB is a resistance wire of uniform cross-section, and S is a sliding contact. The 3 V battery has
negligible internal resistance, and Y is a high resistance voltmeter. Which of the following graphs shows the
correct variation of voltage measured by Y when the contact S is moved from A to B?
A.
B.
C.
D.
E.

3V

3V
A

3V

3V
A

3V

14. (87)
In the circuit shown, no current flows through the galvanometer. If the internal resistance of the cell is negligible,
the value of R is
2V
R
A.
4.3 :.
0.5 A
B.
8 :.
0.5 A
G
C.
10 :.
40 :
20 :
D.
32 :.
E.
160 :.
15. (88)
If a metal conductor, of cross-sectional area A, has n free electrons per unit volume, each carrying a charge e and
moving with a drift velocity v, the
(1) current density (I / A) will be n e v.
(2) current flowing will be e v A.
(3) drift velocity will increase with temperature.
A. (1) only

B. (3) only

16. (89)

C. (1) & (2) only

D. (2) & (3) only

E. (1),(2) & (3)

10 V

X
10 :

In the above circuit, AB is a metre-wire of resistance 10 :. When X is moved to the mid-point of AB, the p.d.
across AX will be
A. 2.5 V.
B. 4.0 V.
C. 5.0 V.
D. 6.0 V.
E. 7.5 V.

MC Question
Chapter 13 Current Electricity
17. (89)

(MC)13P4

2V
A
V

100 :

In the circuit shown, V is a voltmeter of high internal resistance and A is an ammeter of low internal resistance.
What is the voltmeter reading when (a) switch K is open, (b) switch K is closed ?
K closed
K open
A.
0V
0V
B.
0V
2V
C.
1V
1V
D.
2V
0V
E.
2V
2V
18. (89)
In the circuit shown, the equivalent resistance between A and B is
A.
2/7 :.
B.
2/5 :.
C.
2/3 :.
D.
2 :.
E.
5 :.

1:

1:

1:

19. (90)

2:

3 V, 1 :
1A
X

The figure above shows part of a circuit which carries a current of 1 A from X to Y through a cell of e.m.f. 3 V and
internal resistance 1 :. The potential difference between X and Y is
A. 0 V.
B. 1 V.
C. 2 V.
D. 3 V.
E. 4 V.
20. (90)
Which of the following circuits is best used for the measurement of a low resistance R ?
(Polarities of meter terminals are given in the diagram.)
A.
B.
C.
D.

A
.

+
V

3:

+
V

3:

3:

I
6V

In the above circuit, the battery has negligible internal resistance. The current I is
A. 2/3 A.
B. 1 A.
C. 4/3 A.
D. 2 A.
E. 6 A.

+
A

(90)

A
+

21.

E.

+
V

+
V

MC Question
Chapter 13 Current Electricity
22. (91)

(MC)13P5

12 V

5:

5:
S

L1

L2

In the circuit shown, filament lamps L1 and L2 are identical and are seen to be marked 6 V 18 W. Which of the
following statements is/are correct?
(1) When switch S is closed, lamp L1 and lamp L2 glow with the same brightness.
(2) When switch S is closed, lamp L2 glows with its normal brightness.
(3) When switch S is opened, lamp L1 glows with its normal brightness.
A. (1) only
B. (3) only
C. (1) & (2) only
D. (2) & (3) only
E. (1), (2) & (3)
150 :

23. (92)

G
8V

2V

In the circuit shown-above, the galvanometer G reading is zero. So X has a resistance of


A. 15 :
B. 25 :
C. 50 :
D. 100 :

24. (93)

.
E. 150 :

12 V, 3 :

6:

6:

3:

Two cells X and Y, each of internal resistance 3 :, are connected with two 6 : resistors as shown in the above
circuit. If cell X has e.m.f. 12 V and the galvanometer G shows null deflection, what is the e.m.f. of cell Y?
A. 3.0 V
B. 4.0 V
C. 4.8 V
D. 6.0 V
E. 7.2 V

10 cm

25. (94)

80 cm

60 cm

uniform resistance
wire carrying
steady current

G
40 :

In the above circuit, the galvanometer G reads zero. The resistance of resistor X is
A.
40 u (5/2) :
B.
40 u (4/3) :
C.
40 u (2/5) :
D.
40 u (3/4) :
E.
40 u (7/5) :

MC Question
Chapter 13 Current Electricity

(MC)13P6

+ 12 V -

26. (95)
P

Q
J
L1

L2

In the above circuit, PQ is a uniform resistance wire connected to a 12 V constant voltage source. L1 and L2 are
two identical light bulbs, each of rating 12 V, 8 W. What is the power dissipated by L1 when the sliding contact J is
at the mid-point of PQ ?
A. 1 W
B. 2 W
C. 4 W
D. 6 W
E. 8 W
27. (95)
Two cells of negligible internal resistance are connected with
two resistors as shown. What is the potential difference
between X and Y?
A.
1.33 V
B.
1.67 V
C.
2.00 V
D.
2.33 V
E.
2.67 V

Y
1V

2:

3V

4:
X

28. (96)
Six wires, each resistance 1 :, are jointed to form a regular tetrahedron ABCD. A current of 2 A flows across the
tetrahedron at A and B.
C
2A
B

2A
A

D
Which of the following statements is/are correct?
(1) Points C and D are of the same potential.
(2) The current in wire AC is 0.5 A.
(3) The potential drop across AB is 1 V.
A. (1) only
B. (3) only
C. (1) & (2) only

29. (96)

G
E

D. (2) & (3) only

E. (1). (2) & (3)

3V

1 k:

2 k:

500 :

In the above circuit, the galvanometer reading is zero. If both cells have negligible internal resistance, the e.m.f. of
the cell E is
A. 6 V.
B. 7 V.
C. 8 V.
D. 9 V.
E. 10 V.

30. (97)
Which of the following quantities can be increased by increasing the voltage across a metal wire?
(1) the average drift velocity of the conduction electrons
(2) the number of conduction electrons per unit volume
(3) the speed of the electrical signal
A. (1) only
B. (3) only
C. (1) & (2) only
D. (2) & (3) only
E. (1),(2) & (3)

31. (97)
Two cylindrical metal rods, X and Y, are made from the same material and have the same mass. The length of X is
three times that of Y. If currents of 1 A and 2 A pass through X and Y respectively, the ratio of the power
dissipation in X to that in Y is
A. 9 : 1
B. 1 : 4
C. 9 : 2
D. 3 : 4
E. 9 : 4

MC Question
Chapter 13 Current Electricity

(MC)13P7

32. (97)
A

A cell of e.m.f. of 1.5 V is connected in series with a variable resistor of resistance R and an ammeter A of
1
against R is plotted.
resistance 0.5 :. By varying R, a series of ammeter readings, I, are taken and a graph of
I
The value of the y-intercept is found to be 0.88 A-1. What is the internal resistance of the cell ?
A. 0.59 :
B. 0.82 :
C. 1.14 :
D. 1.20 :
E. 1.32 :
33. (97)
Only a small fraction of the electrical energy supplied to a light bulb is converted into light energy. Which of the
following explanations is/are correct?
(1) Energy is absorbed by the glass wall of the bulb.
(2) The resistance of the filament of the bulb is very high.
(3) Most of the energy is emitted as infra-red radiation.
A. (1) only
B. (3) only
C. (1) & (2) only
D. (2) & (3) only
E. (1),(2) & (3)
34. (98)
The variation of current with the voltage applied across a device
is as shown in the figure. What is the change in resistance of the
device when the voltage increases from 1.0V to 2.0V?
A.
It decreases by 0.25 :.
B.
It increases by 0.25 :.
C.
It decreases by 1.50 :.
D.
It increases by 0.60 :.
E.
It decreases by 0.60 :.

current / A
5.0

1.0
0

1.0

2.0

voltage / V

35. (98)
In the circuit shown, light bulbs L1, L2 and L3 consume power at the rate 1 : 1 : 3. If the resistance of L3 is R, the
resistance of L1 is.
L2
A.
3R /16
L1
B.
R /4
C.
3R /4
D.
4R /9
E.
3R
L3

36. (98)
10 k:
6V
2V
5 k:
V
voltmeter, 5 k:

In the above circuit each cell has negligible internal resistance. The voltmeter has a resistance of 5 k:. The
reading on the voltmeter is
A. 0 V
B. 1.0 V
C. 1.2 V
D. 1.5 V
E. 2.0 V

MC Question
Chapter 13 Current Electricity

(MC)13P8

37. (98)
V

In the above circuit, the resistors are identical, the battery has negligible internal resistance and the voltmeter
draws negligible current. What are the possible voltmeter readings before and after closing switch S ?
S open
S closed
A.
2V
4V
B
2V
2V
C.
0V
4V
D.
4V
0V
E.
4V
2V
38. (99)
Three resistors of resistance R1, R2 and R3 are connected in parallel It is known that R1 > R2 > R3. The equivalent
resistance of this combination is R. Which of the following statements is/are correct?
(1) Energy dissipated in moving 1 C of charge through the resistor of resistance R, is greater than that through
R3.
(2) R is smaller than R2.
(3) If the resistor with resistance R1 is removed, the resulting equivalent resistance is increased.
A. (1) only
B. (3) only
C. (1) & (2) only
D. (2) & (3) only
E. (1), (2) & (3)
39. (99)
A resistance wire is connected across the terminals of a battery. Which of the following statements is incorrect?
A.
Before connection to that battery, the conduction electrons in the wire move randomly.
B.
After connection to the battery, an electric field is set up along the length of the wire.
C.
The conduction electrons in the wire are accelerated momentarily in the opposite direction to that of the
electric field.
D.
The conduction electrons collide with one another, giving out heat energy.
E.
The current in the wire is proportional to the average drift velocity of the electrons.
40. (99)
R1
P
R2

Q
R3

In the above circuit, the battery has constant e.m.f. and negligible internal resistance. If switch S is closed, what
would happen to the electric potential at P and at Q?

A.
B.
C.
D.
E.

Potential at P
increased
increased
unchanged
decreased
decreased

Potential at Q
increased
decreased
unchanged
increased
decreased

41. (99)
A
V
R

The above circuit can be used to find the resistance of the resistor R. Which of the following statements is/are
correct?
(1) The current passing through the ammeter is in fact larger than that passing through R.
(2) The ratio of voltmeter reading to ammeter reading is in fact smaller than the resistance of R.
(3) The circuit is suitable for measuring high resistance.
A. (1) only
B. (3) only
C. (1) & (2) only
D. (2) & (3) only
E. (1), (2) & (3)

MC Question
Chapter 13 Current Electricity
42. (00)

(MC)13P9

internal resistance 4 :

In the above circuit, the internal resistance of the battery is 4 :. R is a variable resistor. Which of the following
statements about the circuit is/are correct?
(1) If the resistance of R is very small, the terminal potential difference of the battery is very small but the
power dissipated in the battery is large.
(2) If the resistance of R is very large, the current in the circuit is very small but most of the power
supplied by the battery is dissipated by R
(3) If the resistance of R is 4 : the power supplied to R by the battery is at a maximum.
A. (1) only
43.

B. (3) only

C. (1) and (2) only

D. (2) and (3) only

E. (1), (2) and (3)

(00)
0.2 A

5:

0.3 A

3:

1:

The figure shows some of the resistors in a network of resistors. The magnitudes and directions of some of the
currents are marked as shown. Find the magnitude and direction of the current passing through the resistor R
A.
0.2 A from right to left
B.
0.2 A from left to right
C.
0.4 A from right to left
D.
0.4 A from left to right
E.
It cannot be determined as the value of R is not given.
10 :

44. (01)

60 :
C

R
10 :

60 :
D

The above figure shows a network of resistors. If a voltage of 100 V is applied across terminals A and B, the
potential difference between C and D is 80 V. If the voltage is applied across terminals C and D instead, what is
the potential difference between A and B ?
A. 80 V
B. 60 V
C. 40 V
D. 20 V
E. It cannot be found as the value of R is not known.
45. (01)
X and Y are bulbs with ratings 6 V, 12 W and 6 V, 3 W respectively. If they are connected to a 12 V supply of
negligible internal resistance, which of the following connections allows the two bulbs to work at their respective
rated values?
X
X
A.
B.
C.
Y
X
Y

D.

E.
X

Y
X

MC Question
Chapter 13 Current Electricity
46.

(MC)13P10

9V

(02)
R

S
V3

V1

V2

In the above circuit, the d.c. supply has an e.m.f. of 9 V and its internal resistance is negligible. V1, V2 and V3 are
three voltmeters with the same finite internal resistance. If V1 reads 4 V, find the reading of V3 and the potential
difference across resistor R.
Potential difference across R
Voltmeter reading of V3
A.
3V
1V
B.
5V
1V
C.
1V
5V
D.
1V
3V
47. (02)
A d.c. supply of constant e.m.f. and internal resistance is connected to a variable resistor of resistance R. Which of
the following graphs best shows how the total power P delivered by the supply varies with R?
A. P

C. P

B. P

R
0

D.P

6V

48. (03)

4:
2:

4:

S
In the above circuit, a battery of e.m.f. 6 V and negligible internal resistance is connected to three resistors. What
are the electric potentials at P before and after switch S is closed?
After
Before
A.
+6 V
+3 V
B.
+3 V
+3 V
C.
+2 V
+2 V
D.
+2 V
+3 V

MC Question
Chapter 13 Current Electricity
1:

2:

49. (03)

(MC)13P11

2:

1:

2:

In the above circuit, the battery has constant e.m.f. and negligible internal resistance. A high-resistance voltmeter
connected across terminals a and b reads 4 V. If a low-resistance ammeter is connected across a and b, the
ammeter should read
A.
0.6 A.
B.
1.0 A.
C.
1.2 A.
D.
2.0 A.
50. (04)
B

1 k:

R
S

In the above network of resistors, the resistance of S is infinitely large and the two resistors R are identical. If the
equivalent resistance across CD is 2.5 k:, what is the equivalent resistance across AC?
A.
2.5 k:
B.
3.5 k:
C.
5.0 k:
D.
infinitely large
51. (04)

6 V, 2 :

R
S
6:

In the above circuit the battery has an e.m.f. of 6 V and internal resistance 2 :. S is a standard load resistor of 6 :
and R is a rheostat. What will the setting of R be if the power delivered by the battery to the load resistor S is at a
maximum?
A.
0:
B.
2:
C.
4:
D.
6:
52. (04)
The graph shows the I-V characteristic of two light bulbs X and Y, which
are marked respectively as 200 V, 100 W and 200 V, 60 W. If X and
Y are connected in series to a 200 V mains supply, what is the
approximate power dissipated in each bulb?

A.
B.
C.
D.

X
12 W
15 W
40 W
50 W

Y
36 W
25 W
20 W
30 W

MC Question
Chapter 13 Current Electricity

(MC)13P12

53. (05)
A p.d. of 10 V is applied across a uniform wire of resistance 5.0 :. The number of conduction electrons per unit
volume of the wire is 1.0 u 1028 m-3 and its cross-sectional area is 1.0 mm2. What is the average drift velocity of
conduction electrons in the wire ?
(Given : charge on electron = 1.6 u 10-19 C)
A.
3.1 u 10-4 m s-1
B.
3 u 10-3 m s-1
C.
3.1 u 10-3 m s-1
D.
6.3 u 10-3 m s-1

54. (06)

6V
X

3 k:

3V

6 k:

In the above circuit, the cells have negligible internal resistance. What is the potential difference between the
points X and Y? What is the current in the wire connecting the two points ?
current
potential difference
A.
0V
1.5 mA
B.
0V
0 mA
C.
3V
1.5 mA
D.
3V
0.5 mA
55. (06)

The maximum power dissipation allowed for a certain resistor is 4 W. If three such resistors are connected as
shown, what is the maximum power dissipated in this combination without damaging any of the resistors ?
A. 4 W
B. 6 W
C. 8 W
D. 9 W

Answers
1.C

2.D

3.A

4.C

5. C

6. B

7. C

8. E

9. E

10.E

11.B

12.C

13.C

14.C

15.A

16.B

17.D

18.A

19.E

20.A

21.E

22.C

23.C

24.C

25.C

26.B

27.D

28.E

29.D

30.A

31.E

32.B

33.B

34.E

35.A

36.A

37.A

38.D

39.D

40.E

41.C

42.E

43.A

44.C

45.D

46.D

47.C

48.D

49.C

50.B

51.A

52.A

53.B

54.A

55.B

MC Question
Chapter 14 Magnetic field
1.

2.

(MC)14P1

(80)
Two charged particles X and Y enter a region where a magnetic field acts
perpendicular to the plane of their motion. The resulting paths shown in
the diagram may be affected by the mass, charge and initial speed of the
particles. Which of the following could cause the observed difference in
the paths ? (The other two quantities being equal for X and Y.)
(1) Y has a larger mass than X.
(2) Y has a larger charge than X.
(3) Y has a smaller initial speed than X.
A. (1) only
B. (3) only
C. (1) & (2) only
D. (2) & (3) only

E. (1), (2) & (3)

(80)
Which of the following affects the magnetic flux density on the axis of along solenoid ?
(1) The radius of the solenoid
(2) The number of turns per. unit length
(3) The current in its windings
A. (1) only
B. (3) only
C. (1) & (2) only

D. (2) & (3) only

E. (1), (2) & (3)

3.

(81)
Out of syllabus
A galvanometer has a scale divided into 120 equal divisions. It has a current sensitivity of 10 divisions per
milliampere, and a resistance of 4.0 :. What is its voltage sensitivity, in divisions per millivolt ?
A. 0.25
B. 2.5
C. 4.0
D. 12 E. 30

4.

(81)
A free electron travelling horizontally with speed v enters a uniform vertical magnetic field B. Which of the
following statements is correct ?
(1) The path of the electron is a vertical circle.
(2) The speed of the electron remains constant.
(3) The radius of curvature of the path is inversely proportional to B.
A. (1) only
B. (3) only
C. (1) & (2) only
D. (2) & (3) only
E. (1), (2) & (3)

5.

(82)
A very long solenoid with a metallic core has a radius r and n turns per unit length. It carries a current I. The
magnetic flux density B on its axis is
(1) independent of r.
(2) proportional to n.
(3) independent of the material of the core.
A. (1) only
B. (3) only
C. (1) & (2) only
D. (2) & (3) only
E. (I), (2) & (3)

6.

(82)
X and Y are identical flexible conducting ribbons, suspended from points P and Q.
The bottom parts of the ribbons lie in dishes of a conducting liquid at R and S. When
a current 2I is passed from R to P through X, and a current I is passed from Q to S
through Y, which of the diagrams best represents the shapes assumed by the ribbons
(if the ribbons remain in contact with the conducting liquid at R and S) ?
P Q
Q
P
A.
B.
C.
D.
Q
P
Q
P

E.

MC Question
Chapter 14 Magnetic field

(MC)14P2

7.

(82)
An electron travelling with velocity v enters a region with a uniform electric field E
E
and a uniform magnetic flux density B. E, B and v are mutually perpendicular, as
B into paper
shown in the diagram.
Which of the following statements is/are correct?
v
(1) When B = 0, the path of the electron is parabolic.
(2) When E = 0, the path of electron is circular.
(3) The curvature of the electron's path is independent of v.
A. (1) only
B. (3) only
C. (1) & (2) only
D.(2) & (3) only
E. (1), (2) & (3)

8.

(83)
Two parallel wires repel each other with a force F when the same current passes through them. If the current is
doubled and the distance between the wires is also doubled, the force of repulsion will then be
A. F/4.
B. F/2.
C. F.
D. 2F.
E. 4F.

9.

(85)
Which of the following graphs best represents the variation of the magnetic flux density B along the axis of along
current-carrying solenoid, with the distance x from the centre of the solenoid along the axis ?
A. B
B. B
C. B
D. B
E. B

0
10.

x
0

(85)
B into
paper

A particle, having a mass of 32 u 10-26 kg and a charge of -1.6 u 10-19C, enters a uniform magnetic field of flux
density 0.08 T at a speed of 105 m s-1 , as shown. It will
A. pass undeviated through the magnetic field.
B. be deflected upward in a circular arc of radius 0.25 m.
C. be deflected upward in a circular arc of radius 0.50 m.
D. be deflected downward in a circular arc of radius 0.25 m.
E.
be deflected downward in a circular arc of radius 0.50 m.
11.

(87)

constant
current

The figure shows the essential parts of an apparatus to demonstrate the Hall effect. Which of the following
statements is/are correct ?
(1) In the arrangement above, the Hall p.d. is developed across PQ.
(2) The magnitude of the Hall potential is greater if the applied magnetic flux density is increased.
(3) The magnitude of the Hall potential is less if the width PQ of the specimen is decreased.
A. (1) only
B. (3) only
C. (1) & (2) only
D. (2) & (3) only
E. (1), (2) & (3)

MC Question
Chapter 14 Magnetic field
12.

(MC)14P3

(87)
R

0.5 a

0.4 a

0.3 a

P and Q represent two long, straight, parallel, conducting wires separated by a distance of 0.5a, as shown in the
figure above. Each of them carries a current I flowing into the plane of the paper. The x-component of the
magnetic flux density at the point R is
P I
P I
P I
P I
C. 0 u 4.00
D. o u 4.17
E. o u 5.38
A. zero
B. o u 1.17
2Sa
2Sa
2Sr
2Sa
13.

(87)
X
R

S
Y

An a.c. supply S of frequency 50 Hz is connected to a resistor R via two long, parallel, straight metal wires X and
Y as shown. The force between X and Y
A. is equal to zero.
B. always attracts.
C. always repels.
D. sometimes attracts and sometimes repels; the frequency of variation is 50 Hz.
E. sometimes attracts and sometimes repels; the frequency of variation is 100 Hz.
14.

(88)

I2

I1

x
N

In the figure shown, MN is a fixed long conductor, carrying a current I1. HK is another conductor perpendicular to
MN. When a current I2 is allowed to pass through HK in the direction shown, the force on HK
A. acts in the +y direction.
B. acts in the -y direction.
C. acts in the +x direction.
D. acts in the -x direction.
E. is zero.
15.

(88)
A particle of mass m and charge q moves in a circular orbit in a magnetic field B. The time taken to complete a
single orbit is
B. 2Sm/(Bq).
C. 2mq/(BS).
D. Bm/(2Sq).
E. 2S/(mBq).
A. Bq/(2Sm).

16.

(88)

y
R

0.4 a

0.3 a
0.5 a

P and Q represent two long, straight, parallel, conducting wires separated by a distance of 0.5a, as shown in the
figure above. Each of them carries a current I flowing into the plane of the paper. The magnitude of the
y-component of the magnetic induction at point R is
P I
P I
P I
P I
C. 0 u 4.00
D. o u 4.17
E. o u 5.38
A. zero
B. o u 1.17
2Sa
2Sa
2Sr
2Sa

MC Question
Chapter 14 Magnetic field
17.

(MC)14P4

p
q

(89)

r s

A circular loop carrying a current I1 is placed in a uniform magnetic field B in the xy plane as shown. If the loop is
free to move, the magnetic forces will cause it to
A. rotate about the y-axis as indicated by p.
B. rotate about the y-axis as indicated by q.
C. rotate about the x-axis as indicated by r.
D. rotate about the x-axis as indicated by s.
E. remain stationary.
18.

(90)

S1

S2

A mixed stream of ions (of different charges, polarities and speeds) travel along PQ and passes through a narrow
slit S1. In the region between S1 and S2 , an electric field E and a magnetic field of flux density B are directed
normally to each other. The E -field acts vertically downward and the B-field acts into the plane of the paper. Ions
that are undeflected and emerge from slit S2 have the same
A. polarity.
B. speed.
C. charge.
D. mass.
E. charge to mass ratio.
19.

Out of syllabus
(91)
It is desired to re-design a moving coil galvanometer so as to make it four times as sensitive. Which of the
following would alone achieve the desired result ?
(1) increasing the magnetic flux density of the permanent magnet to twice its value and doubling the
crosssectional area of the coil.
(2) Providing the coil with a shunt so that only a quarter of the input current flows through coil itself.
(3) Changing the suspension characteristics so that four times as great a couple is needed to cause one radian
twist.
A. (1) only

20.

B. (3) only

C. (1) & (2) only

D. (2) & (3) only

E. (1), (2) & (3)

(91)
The magnetic field due to a steady current in the long air-core solenoid which is uniformly wound is 8 x 10-3 T at
X and 1 x 10-3 T at Y as shown below.

X
L

If an identical solenoid is connected to the end of the first so as to extend it to as far as Y, and the same current as
before is passed through the complete solenoid, the magnetic field at Y will then be
B. 8 u 10-3 T
C. 9 u 10-3 T
D. 16 u 10-3 T
E. 17 u 10-3 T
A. 7 u 10-3 T
21.

(92)
Out of syllabus
A moving-coil galvanometer with a coil of area A and N turns has a full-scale deflection for a current i. If the coil
were of area 3A and 2N turns, the current which would give full scale deflection would be
A. i /6
B. 2i /3
C. i
D. 3i /2
E. 6 i

MC Question
Chapter 14 Magnetic field

(MC)14P5

22.

(92)
Two parallel straight conductors separated by a distance r carry currents in the same direction. Which of the
following statements is/are correct ?
(1) The two wires attract each other.
(2)The force acting on each wire is inversely proportional to r2.
(3) The wires produce a magnetic field with maximum flux density midway between them.
A. (1) only
B. (3) only
C. (1) & (2) only
D. (2) & (3) only
E. (1), (2) & (3)

23.

(92)

Y
I

A square coil of N turns and area A carries a current I. The coil is free to rotate about the axis XY which is normal
to a uniform magnetic field B. When the field makes an angle T with the plane of the coil, what are the magnitude
and direction of the torque acting on the coil as detected by an observer at X?
Direction
Magnitude
A. BANI cos T
clockwise
B. BANI sin T
clockwise
C. BANI cos T
anticlockwise
D. BANI sin T
anticlockwise
E. none of the above
24.

25.

(92)
When switch S is closed, the steel strip will
A. remain stationary.
B. remain attracted to the iron core.
C. vibrate at a frequency of 50 Hz.
D. vibrate at a frequency of 100 Hz
E.
vibrate at a frequency of 200 Hz

steel strip

50 Hz
S
iron core

1A

(93)
X

0.2 m
1A

The above figure shows two long parallel straight wires separated by a distance of 0.2 m, carrying currents of l A
in opposite directions. The magnetic field at a point X mid-way between the wires is
(Given permeability constant Po = 4S u 10-7 T m A-1)
A. 0 T
B. 2 u 10-6 T into paper
C. 2 u 10-6 T out of paper
D. 4 u 10-6 T into paper
E. 4 u 10-6 T out of paper
26.

(93)
Two particles P and Q of the same quantity of charge and mass but moving
with different speeds vp and vQ respectively enter a region of uniform
magnetic field directed into the plane of the paper. The subsequent circular
paths are as shown. Which of the following statements is/are correct ?
(1) Both P and Q are positively charged.
(2) vp is smaller than vQ.
(3) The period of circular motion of P is shorter than that of Q.
A. (1) only
B. (3) only
C. (1) & (2) only
D. (2) & (3) only

Q
P

ion source

E. (1), (2) & (3)

MC Question
Chapter 14 Magnetic field

(MC)14P6

27.

(94)
For two long, straight parallel conducting wires carrying the same current, the magnitude of the force acting on a
section of the wires depends on
(1) the distance between the wires
(2) the length of that section of the wires
(3) the directions of current flow in the wires
A. (1) only
B. (3) only
C. (1) & (2) only
D. (2) & (3) only
E. (1), (2) & (3)

28.

(94 ASL)
path of
the electron

uniform magnetic field


of 0.02 T (into paper)

An electron moves in a circular path of diameter 0.01 m in a plane with a uniform magnetic field of 0.02 T
perpendicular to it. Find the speed and the sense of circular motion of the electron.
Given : charge of an electron = 1.6 u10-19 C ; mass of an electron = 9.1 u 10-31 kg
A. 1.76 u 107 m s-1, anticlockwise
B. 1.76 u 107 m s-1, clockwise
C. 3.52 u107 m s-1, anticlockwise
D. 3.52 u107 m s-1, clockwise
E. 7.03 u107 m s-1, anticlockwise
29.

(95)

B
A

X
Y

uniform magnetic field


(out of paper)

Particles A and B moving at the same speed enter a square region of uniform magnetic field as shown. Particle A
leaves at X while particle B leaves at Y. If the charge to mass ratio of particle A is k, what is that of particle B?
A. k/2
B. k/4
C. k
D. 2k
E. 4k
30.

(95)
In a Hall probe, the slice of semiconductor inside has 1025 charge-carriers per m3 . When a steady current of 0.4 A
is passed through the slice and a uniform magnetic field of 0.1 T is applied perpendicularly to it, a Hall voltage of
20 PV is set up. Find the thickness of the slice.
(Given: electronic charge = 1.6 x 10-19 C)
A. 0.9 u10-3 m

31.

B. 1.1 u 10-3m

C. 1.3 u 10-3m

D. 1.5 u 10-3 m

(96)
Four infinitely long straight parallel wires P, Q, R, S carrying equal
currents are situated at the corners of a square as shown. The
currents in P, Q are into paper and those in R, S are out of paper.
What is the direction of the resultant magnetic induction at the
centre of the square?
A. I
B. II
C. III
D. IV
E. V

E. 1.7 u10-3 m

II
I

III
V

IV

MC Question
Chapter 14 Magnetic field
32.

(MC)14P7

(98)

The above diagram shows a rectangular current-carrying coil ABCD in a uniform magnetic field between two pole
pieces. The magnetic field is perpendicular to the plane of the coil. Which of the following statements is/are
correct?
(1) There is a magnetic force acting on the side BC of the coil.
(2) The magnetic forces acting on the coil tend to reduce its area.
(3) The coil will return quickly to its original vertical position when it is disturbed slightly.
A. (1) only
B. (3) only
C. (1) & (2) only
D. (2) & (3) only
E. (1), (2) & (3)

33.

(98)

The above figure shows a charged particle moving in a circle with constant speed v on a plane perpendicular to a
uniform magnetic field. Which of the following graphs represents the relation between the time T for the particle
to complete a circle and its speed v ?
T
T
T
A. T
B. T
C.
D.
E.

34.

(98)
Two long, straight parallel conducting wires, each carrying a current I, are separated by a distance r as shown.
What is the magnetic field at a point P at the same distance r from both wires?
(P0 = permeability of free space)
A.
B.
C.
D.
E.

Po I
to the left
2Sr
3P o I
to the left
2Sr
Po I
to the left
Sr
3P 0 I
to the right
2Sr
P0 I
to the right
2Sr

P
r

current I into paper

MC Question
Chapter 14 Magnetic field
35.

(MC)14P8

(99)
Which of the following graphs correctly shows the variation of the magnetic flux density B along the axis of
along solenoid carrying a steady current with its length " ? .
A.
B. B
C. B

0
D.

"

"

"

E. B

0
36.

"

"

(99)

Two long, straight, parallel conducting wires P and Q are positioned as shown. The same current flows through
both wires and is directed into the plane of the paper. Points A, B and C on the plane of the paper are equidistant
from both wires where C is the mid-point between the wires. Which of the following statements is/are correct?
(1) The magnetic field strength at C is greater than that at A.
(2) The directions of the magnetic field at A and at B are the same.
(3) The magnetic field strength at B will increase if the current flowing in the wires increases.
A. (1) only
B. (3) only
C. (1) & (2) only
D. (2) & (3) only
E. (1), (2) & (3)
37.

(99)
For which of the following does the force between two objects vary inversely as the square of the distance
between their centres ?
(1) Two equal masses joined by a rubber band under tension.
(2) Two long straight parallel conducting wires carrying steady electric currents.
(3) The earth and a satellite in its orbit.
A. (1) only
B. (3) only
C. (1) & (2) only
D. (2) & (3) only
E. (1), (2) & (3)

38.

(00)
Four parallel long straight conductors carrying currents of equal magnitude pass
vertically through the four corners of a square PQRS. The current is directed into
paper in one conductor and is directed out of paper in the other three conductors.
Which of the following arrangement can produce a resultant magnetic induction at
the centre O of the square in the direction shown?
Current into paper
A.
B.
C.
D.
E.

Current out of paper

P
P
Q, R, S
Q
P,R,S
R
P,Q,S
S
P,Q,R
It is impossible to produce a resultant magnetic induction at O in the direction shown.

MC Question
Chapter 14 Magnetic field
39.

40.

41.

42.

(MC)14P9

(00)
A beam of charged particles passes undeflected through a region of crossed uniform electric and magnetic fields.
Which of the following must be common to the particles making up this beam?
A. charge to mass ratio
B. velocity
C. mass
D. sign of charge
E.
magnitude of charge
(01)
Which of the following properties is NOT common to both electric and magnetic fields ?
A.
Both can be shielded using suitable materials.
B.
Both can exert attractive and repulsive forces.
C.
Both can deflect moving charged particles.
D.
Both can be represented by field lines of closed loops
E.
Both can store energy.
(01) Out of syllabus
Which of the following descriptions about a moving-coil meter is/are correct ?
(1) It has a massive soft iron core to provide damping.
(2) It has curved magnetic poles to assist in producing linear scale.
(3) It has weak hair springs to increase the sensitivity.
A. (1) only
B. (3) only
C. (1) and (2) only
D. (2) and (3) only

E. (1), (2) and (3)

(02)

The above figure shows an electron entering a uniform field which may be electric or magnetic. Which of the
following descriptions about the subsequent motion of the electron is correct ?
A.
Only in a magnetic field can the electron be deflected by more than 90o.
B.
Only in an electric field does the force depend on the magnitude of the charge on the electron.
C
Whether the field is electric or magnetic, the speed of the electron will increase.
D.
Whether the field is electric or magnetic, the magnitude and direction of the force acting on the electron are
constant.
43.

(02)

Three long straight parallel wires P, Q and R carrying currents of the same magnitude are situated at the vertices of
an equilateral triangle as shown. The currents in wires P and R are directed out of the paper. Which of the following
indicates the direction of the force acting on wire P?
A.

B. p

C. n

D.

MC Question
Chapter 14 Magnetic field
44.

(MC)14P10

(03)

I1

C I
2

20 cm 10 cm

In the above set-up, AB and CD are two parallel infinitely long wires 20 cm apart, carrying currents I1 and I2
respectively. The magnetic flux density at the point P 10 cm from wire CD is zero. If I2 = 0.6 A, which of the
following statements about I1 is correct?
A. 0.2 A flows in the same direction as I2.
B. 0.2 A flows in the opposite direction to I2.
C. 1.8 A flows in the same direction as I2.
D. 1.8 A flows in the opposite direction to I2.
45.

(03)
A charged particle enters a region of uniform magnetic field whose direction is normal to the initial velocity of the
particle. The subsequent path of the particle is as shown.
Magnetic
field normal
to paper

Which of the following is/are possible explanations to account for the shape of the path?
(1) The flux density of the magnetic field has decreased gradually.
(2) The charged particle has lost its charge gradually.
(3) The charged particle has lost its kinetic energy gradually.
A.
B.
C.
D.
46.

(1) only
(3) only
(1) and (2) only
(2) and (3) only

(04)
Three long, parallel, straight current-carrying wires P, Q and R are placed in the same plane in air as shown.
1A

2A

2A

P
Q
R

If F is the force per unit length between two long, parallel, straight wires, placed a distance d apart and each
carrying a current of 1 A, what is the net force per unit length acting on the wire R shown?
A.
B.
C.
D.

0
F
2F
3F

MC Question
Chapter 14 Magnetic field
47.

(04)
The Hall effect
(1) provides evidence that the charge carriers in metals are negatively charged.
(2) can be used to determine the density of free electrons in a metal.
(3) can be used to determine the flux density of the magnetic field due to an a.c. only.
Which of the above statements are correct?
A. (1) and (3) only
B. (1) and (2) only
C. (2) and (3) only
D. (1), (2) and (3)

48.

(05)
W

(MC)14P11

D-particle's path
Z

Magnetic field
out of paper

Y
protons path

A proton and an D-particle move in a uniform magnetic field. The magnetic field is directed out of the plane of the
paper. Within a square region WXYZ, the proton takes time t1 to complete a half circle from X to Y while the
D-particle follows a quarter circle from X to Z in time t2. What is the ratio t1 : t2 ?
A. 1 :2
B. 1:1
C. 2: 1
D. It cannot be determined as the ratio of their speeds is not given.
49.

(06)
long straight wire

I
P
r

A long straight wire carrying a current I is placed at a distance r from the point P. Both the wire and point P are in
the plane of the paper. When the current 1 increases by 0.1 A, the magnetic flux density B at point P increases by
1.0 u10-6 T. Find r. (Given : permeability of free space Po = 4S u 10-7 H m-1)
A. 1 cm
B. 2 cm
C. 4 cm
D. 8 cm

MC Question
Chapter 14 Magnetic field
50.

(MC)14P12

(06)

The figure shows two long, parallel conductors PQ and RS connected to a low voltage power supply delivering
sinusoidal a.c. Which of the following best shows how the force of interaction F between them varies with time
t?
A.
B.
F

0
C.

D.

F
t

ANSWER

1.D
16.A
31.A
46.D

2.D
17.D
32.C
47.B

3.B
18.B
33.B
48.B

4.D
19.A
34.D
49.B

5.C
20.C
35.B
50.C

6.A
21.A
36.D

7.C
22.A
37.B

8.D
23.A
38.D

9.D
24.D
39.B

10.D
25.D
40.D

11.C
26.C
41.D

12.D
27.C
42.A

13.C
28.B
43.A

14.A
29.A
44.D

15.B
30.C
45.B

MC Question
Chapter 15Electromagnetic Induction

(MC)15P1

1.

(80)
The magnetic flux through a coil of N turns drops at a uniform rate from ) to zero in time t, What is the
magnitude of the e.m.f. induced in the coil?
B. ) t / N
C. N ) / t
D. N t / )
E. )/ Nt
A. N ) t

2.

(80)
When a circular coil rotates about a vertical diameter in a uniform horizontal magnetic field, the maximum
induced e.m.f. is E. If the number of turns is trebled (multiplied by 3), the coil radius is halved and the frequency
of rotation is doubled, what is the new maximum induced e.m.f. ?
A. 3 E/8
B. 3E / 4
C. 3E / 2
D. 3E
E. 6E

3.

(80) Out of syllabus


The self-inductance of a coil is equal to
(1) the magnetic flux linking the coil per unit current flowing in the coil.
(2) the back e.m.f. induced in the coil per unit current carried.
(3) the rate of change of magnetic flux linking the coil.
A. (1) only
B. (3) only
C. (1) & (2) only
D. (2) & (3) only

4.

(81)
Out of syllabus
A long solenoid uniformly wound with N turns is of length ", cross-sectional area A and carries a current I. The
total energy stored in the magnetic field inside the solenoid is given approximately by
A.

5.

6.

E. (1), (2) & (3)

P o AI 2 N 2
"2

B.

P o AI 2 N 2

P o AIN 2

C.

2" 2

"2

D.

P o AIN 2
2" 2

E.

P o AI 2 N 2
2"

Out of syllabus
(81)
The armature of a d.c. electric motor has resistance 2 :. When the applied potential difference is 150 V, a current of
5 A flows. The back e.m.f. is
A. 10 V.
B. 93 V.
C. 140 V.
D. 150 V.
E. 160 V.
(82)

L2

L2

S
L1

L1

S is a long current-carrying solenoid. L1 is a wire loop just inside the solenoid, and L2 is a wire loop just outside the
solenoid. The current in the solenoid is increased at a steady rate, such that the e.m.f, induced in L1 is 1.2 V. The
e.m.f. induced in L2 is approximately
A. 0 V.
B. 0.6 V.
C. 1.2 V.
D. 2.4 V.
E. larger than 2.4 V.
7.

(82)
Out of syllabus
An inductor L, a resistor R and a switch S are connected in series across a cell with e.m.f. E. Initially the switch is
closed, as shown in the figure below

E
The switch S is now opened, and at the instant it is opened, a spark passes across it. This is because
A. the breakdown potential of the air is less than E.
B. at the instant when the switch is opened the whole of the e.m.f. E is across L.
C. the rate of change of the current through the circuit is very large when the switch is opened.
D. the self-inductance of the circuit generates an e.m.f. equal to E.
E.
the self-inductance of the circuit generates an e.m.f. which is proportional to the current flowing in the
circuit.

MC Question
Chapter 15Electromagnetic Induction
8.

(83)
Out of syllabus
Which of the following graphs best shows the change of current I with time t when an electric motor connected to a
d.c. source is switched on? (The magnetic field is supplied by a permanent magnet.)
A.
B.
C.
D.
E.
I

9.

(MC)15P2

(83)
A rectangular coil PQRS is driven with constant velocity in direction perpendicular to a uniform magnetic field as
shown.

Which of the following statements is correct ?


A. An induced current is flowing in the coil in the clockwise direction.
B. An induced current is flowing in the coil in the anticlockwise direction.
C. An electromagnetic force acts on the side PQ in a direction opposing its motion.
D. There is no induced current flowing in the coil.
E.
The magnitude of the magnetic flux through the coil changes with time.
10. (83)

coil

Out of syllabus

L, R
S
_

+
V

A steady potential difference V is applied across a coil of inductance L and resistance R, connected to a switch S, as
shown. After closing the switch S, the current increases and takes time t to reach half of its steady maximum value.
The time t can be shortened by
(1)
inserting a piece of soft iron inside the solenoid.
(2)
applying a higher potential difference to the circuit.
(3)
adding a resistor in series with the coil.
A. (1) only
B. (3) only
C. (1) & (2) only
D. (2) & (3) only
E. (1), (2) & (3)
11. (84)

A large aluminium disc mounted on a horizontal axle is spun in the clockwise direction between the poles of a
powerful horseshoe magnet. Which of the following diagrams shows how the eddy currents flow in the disc ?
A
B.
C.
D.
E.

12. (84)
1000 turns of wire are wound on a solenoid of length 0.30 m and area of cross-section 3.2 u 10-4 m2. When the
solenoid carries a current of 1.5 A, the magnetic flux through the solenoid is (Po = 1.26 u 10-6 H m-1 )
B. 2.0 u 10-6 Wb.
C. 5.7 u 10-4 Wb.
D. 2.0 u 10-3 Wb.
E. 6.3 u 10-3 Wb.
A. 6.0 u 10-7 Wb.

MC Question
Chapter 15Electromagnetic Induction

(MC)15P3

13. (84)
A sinusoidal voltage is generated by an a.c. dynamo. If the speed of rotation of the dynamo coil is doubled, what
will happen to the frequency and the amplitude of the voltage formed?
amplitude
frequency
A. no change
no change
B. doubled
no change .
C. no change
doubled
D. doubled
halved
E.
doubled
doubled
14. (84)
A power station supplied electrical power to a user. The power generated by the station is 1200 kW and is
transmitted at 132 000 V. If the resistance of the lines connecting the power station to the user is 550 :, the power
available to the user is about
A. 700 kW.
B. 1155 kW.
C. 1195 kW.
D. 1200 kW.
E. 31680 kW.
15. (85)

bar magnet

disc

A bar magnet is freely suspended so that it hangs horizontally above a flat aluminium disc-which is rotating in a
horizontal plane. The bar magnet will
A.
rotate in the same direction as the disc.
B.
rotate in the opposite direction to the disc.
C.
remain at rest.
D.
oscillate to and fro in the horizontal plane.
E.
oscillate to and fro in the vertical plane.
16. (85)

Out of syllabus
i

X
L
Y

L is a pure inductor, in series with a resistor R. The current i in the circuit is initially zero at time t = 0 and
afterwards it rises linearly to reach a maximum at t = T; it then falls linearly to zero in the same time. Which of the
following waveforms represents the variation of the potential V at point X with respect to time t?
A
B.
C.
D.
E.V
V
V

t 0

17. (85)
Out of syllabus
An ammeter is connected in series with an electric motor which is running freely, and the reading is noted. When
the motor raises a load at a steady speed, the reading of the ammeter will
A.
remain the same.
B.
increase to a higher value.
C.
decrease to a lower value.
D.
increase to a maximum value and then decrease.
E.
decrease to a minimum value and then increase.
18. (86)
Out of syllabus
X and Y are two long solenoids having the same length and the same cross-sectional area. They are wound on the
same type of core material. X has 500 turns and Y has 1000 turns. If the self-inductance of X is 0.5 H, the
self-inductance of Y is
A. 0.5 H.
B. 0.7 H.
C. 1.0 H.
D. 2.0 H.
E. 4.0 H.

MC Question
Chapter 15Electromagnetic Induction

(MC)15P4

19. (86)

A short bar magnet passes through an air-cored solenoid of similar length with uniform velocity. Which of the
following graphs best represents the variation of the current I in the solenoid with time t ?
A
B.I
C.I
D. I
E.I
I
0

t
0
16 V

20. (86) Out of syllabus

t0

2:
2:
2mH

2 PF

2:

When a steady state is reached, the current delivered by the battery is


A. zero.
B. 4.0 A.
C. 5.0 A.
D. 5.3 A.

21. (87)

E. 8.0 A.

N
C

A narrow coil C connected to a sensitive galvanometer G is placed between the poles of a powerful electromagnet,
with its plane normal to the magnetic field which is uniform. Initially the electromagnet is on, then it is switched
off at time ts . Which of the graphs below best represents the variation of the galvanometer current i with time t ?
A. i
B.
C.
D.
E.
i

0
ts

ts

t0

i
ts

0
ts

ts

22. (87)
The uniform magnetic field shown pointing into, and acting perpendicular to, the plane of the paper is confined to a
cylindrical volume of radius 8 cm. The magnetic flux density now decreases at a constant rate of 0.01 T s-1. A
circular loop of radius 10 cm is placed so that its plane is perpendicular to the magnetic field, as shown. What will
be the magnitude and direction of the induced e.m.f. in the loop?
loop
Direction
Magnitude
A. 2.0 u 10-4 V
from A to B via the loop
B
from B to A via the loop
B. 2.0 u 10-4 V
A
from A to B via the loop
C. 3.1 u 10-4 V
from B to A via the loop
D. 3.1 u 10-4 V
field
E.
here is no induced e.m.f.

MC Question
Chapter 15Electromagnetic Induction
23. (87)

primary
coil

(MC)15P5

secondary
coil

The above diagram shows step-down voltage transformer. Which of the following is/are correct?
(1) The dotted line indicates the path of the eddy current.
(2) If the terminals of the secondary coil are shorted, the primary current will increase.
(3) The number of turns in the primary is higher than that of the secondary.
A. (1) only
B. (3) only
C. (1) & (2) only
D. (2) & (3) only
E. (1), (2) & (3)
P

24. (88)

Two coaxial solenoids P and Q are arranged as shown. The cross-sectional areas of P and Q are A1 and A2
respectively. P contains N1 turns while Q contains N2 turns. Q is now connected to a power supply so that the
current through it rises at a uniform rate S. The maximum e.m.f. induced in P is
B. Po N1 N2 A2 S.
C. Po N1 N2 A1 S/L.
D. Po N1 N2 A2 S/L.
E. zero.
A. Po N1 N2 A1 S.
25. (88)
A rectangular wire frame surrounds a uniform magnetic field which is confined to a
square region as shown in the diagram. The magnetic field is situated at the centre of
the frame and is perpendicular to the plane of the paper. If the frame moves to the right
with a uniform velocity, which of the graphs below best represents the variation of the
induced current i with time t ? (The clockwise direction of the current is taken as
positive.)
A.
B.
C.
D.
i
i
i
i
0
t
0

wire frame

field
into
paper

E.

26. (89)
Two rectangular wire loops A and B are placed in the same plane. Loop A includes a battery E and switch K, which
is initially open. If K is suddenly closed, what is the direction of the induced current in loop B? Is the force between
the two loops attractive or repulsive?
Nature of force
Direction of current
A.
clockwise
attractive
A
B
B.
clockwise
repulsive
C.
anticlockwise
attractive
K
D.
anticlockwise
repulsive
E
E.
no current
no net force
27. (89)
A length of wire is bent to form a circular coil of one turn. The coil is placed in a uniform magnetic field with its
plane normal to the direction of the field. The flux linkage through the coil is I. The same length of wire is now
bent to forma double loop of smaller radius. The flux linkage is now
B. I/2.
C. I.
D. 2I.
E. 4I.
A. I/4.

MC Question
Chapter 15Electromagnetic Induction

(MC)15P6

28. (89) Out of syllabus


Which of the following statements about a d.c. motor is FALSE ?
A. The back e.m.f. developed dissipates the input energy in Joule heating.
B. If the motor is stopped by some obstruction, the current through the coil will increase.
C. The direction of rotation can be predicted by Fleming's left hand rule.
D. A single-coil motor will not start when it is switched on, if the plane of the coil is perpendicular to the
magnetic field.
E.
An ideal d.c. motor develops a back e.m.f. equal to the applied e.m.f
29. (90) Out of syllabus
The self-inductance of an air-cored solenoid can be increased by
(1)
increasing the current through it.
(2)
increasing the frequency of the supply.
(3)
placing a ferro-magnetic material inside the solenoid.
A. (1) only
B. (3) only
C. (1) & (2) only
D. (2) & (3) only

E. (1), (2) & (3)

30. (90) Out of syllabus


A cell, a switch, a resistor and an inductor are connected in series to form a circuit. At time t = 0 the switch is
closed. Which of the following best represents the variation of the current I in the circuit with time t ?
A.I
B. I
C.I
D.I
E.

t 0

t 0

31. (90) Out of syllabus


A d.c. motor is connected to a 12 V battery. Which of the following statements is correct?
(1) As the motor gathers speed, the current through the motor increases.
(2) As the motor gathers speed, the torque acting on the rotating coil increases.
(3) If the efficiency is 75% at steady state, the back e.m.f. is then 9 V.
A. (1) only
B. (3) only
C. (1) & (2) only
D. (2) & (3) only
E. (1), (2) & (3)
32. (90)
Electrical power is transmitted at high voltage over long distances because
(1) a larger amount of energy can be transmitted per unit charge.
(2) a smaller current is required for a fixed amount of power delivered.
(3) less power is lost through heating during transmission.
A. (1) only
B. (3) only
C. (1) & (2) only
D. (2) & (3) only
33. (91)

E. (1), (2) & (3)

uniform magnetic
field into paper

In the above figure, a copper disc rotates uniformly between the pole-pieces of a powerful magnet (not shown in
figure) in a clockwise direction. P and Q are metallic brushes making contact with the axle and the edge of the disc
respectively. Which of the following statements is correct?
A.
No current flows through R, because there is no flux change through the disc.
B.
No current flows through R because P and Q are at the same horizontal level.
C.
An alternating current flows through R.
D.
A steady current flows from P to Q through R.
E.
A steady current flows from Q to P through R.

MC Question
Chapter 15Electromagnetic Induction

(MC)15P7

34. (91) Out of syllabus


An additional mechanical load is applied to a rotating motor. Which of the following is/are true ?
(1) The motor will decrease in speed.
(2) The e.m.f. induced in the rotating coil will decrease.
(3) More power will be drawn from the power supply.
A. (1) only
B. (3) only
C. (1) & (2) only
D. (2) & (3) only
E. (1), (2) & (3)
12 V

35. (92)

Out of syllabus

8:

2H

When the switch in the circuit is closed,


(1) the current will rise initially at the rate of 6 A s-1.
(2) the final value of the current is 1.5 A.
(3) the final energy stored in the inductor is 2.25 J.
A. (1) only
B. (3) only
C. (1) & (2) only
D. (2) & (3) only
36. (92)

E. (1), (2) & (3)

In the above figure, when the metal rod PQ moves with constant velocity across a uniform magnetic field, a p.d. is
induced across the rod. Which of the following statements is/are correct?
(1) The magnitude of the p.d. depends on the length of the rod.
(2) Q is at a higher potential than P.
(3) A force is acting on the rod to oppose its motion.
A. (1) only
B. (3) only
C. (1) & (2) only
D. (2) & (3) only
E. (1), (2) & (3)
37. (92)
A metal ring is made to float above a coil carrying alternating current.
Which of the following will affect the height of the ring?
(1) the resistivity of the ring
(2) the density of the ring
(3) the frequency of the alternating current
A. (1) only
B. (3) only
C. (1) & (2) only
D. (2) & (3) only
E. (1).(2) & (3)
38. (93)

A
10 V
M

A 10 V battery of negligible internal resistance is applied to a d.c. motor M with armature resistance 5 :. If the
ammeter A of negligible internal resistance shows a reading of 0.4 A, the maximum useful output power delivered
by the motor is
A. 0.8 W.
B. 1.6 W.
C. 2.4 W.
D. 3.2 W.
E. 4.0 W.

MC Question
Chapter 15Electromagnetic Induction
39. (93)

primary coil

(MC)15P8
Ip

secondary coil
soft iron
bar

Ip
current
source

t1

t2

time

Two coils are linked by a soft iron bar as shown. A current source is connected to the primary coil. The primary
current IP varies with time as shown by the graph above.
Which of the following sketches represents the variation of the voltage across the secondary coil VAB with time ?
A.
B.
C. V
D. V
E. V AB
V AB

V AB

t1

t2

AB

AB

t1

time

t2

time

t1

t2

0
time

0
t1

t2

time

t1

t2

40. (94)

Out of syllabus
y

0
t
V2
V1
When the switch S in the above circuit is closed, the variation of quantity y with time t is plotted as shown. The
quantity y could be
(1) the current i in the circuit.
(2) the voltage V1 across the resistor.
(3) the voltage V2 across the inductor.
A. (1) only
B. (3) only
C. (1) & (2) only
D. (2) & (3) only
E. (1), (2) & (3)
41. (94) Out of syllabus
The current in a coil changes steadily from 3 A to 6 A in 75 ms so that a back e.m.f. of 4 V is induced in the coil.
The self-inductance of the coil is
A. 0.025 H.
B. 0.10 H.
C. 40 H.
D. 100 H.
E. 160 H.
42. (95)
A magnet is moved along a perpendicular direction towards a sheet of copper. Which of the following statements
is/are correct ?
(1) Eddy current flows in the sheet.
(2) Temperature of the sheet increases.
(3) Repulsive force is experienced by the magnet.
A. (1) only
B. (3) only
C. (1) & (2) only
D. (2) & (3) only
E. (1),(2) & (3)
N2 turns
area A 2

43. (95)

n 1 turns per metre


area A 1
I

A solenoid of n1 turns per metre and cross-sectional area A1 carries a current I. It is inserted into the core of another
larger solenoid of N2 turns and cross-sectional area A2. If the current in the smaller solenoid drops uniformly to zero
in time t, what is the e.m.f, induced in the larger solenoid ?
2 P o n1 IA1 N 2
2 P o n1 IA2 N 2
P n IA A
P n IA N
P n IA N
A.
B.
C. o 1 1 2
D. o 1 2 2
E. o 1 1 2
t
t
t
t
t

time

MC Question
Chapter 15Electromagnetic Induction

(MC)15P9

44. (95)
factory
machines

generator

The machines in a factory consume 10 kW of electrical power at a voltage of 500 V. If the generator is connected
to the factory through cables of total resistance 02 " the e.m.f produced by the generator should be
A. 500 V
B. 501 V
C. 502 V
D. 504 V
E. 508 V
45. (95)
The mechanical power output of a d.c. motor is always less than the electrical power input. Which of the following
can be an explanation of this?
(1) heat loss in the coil of the motor
(2) work done against friction
(3) work done against back e.m.f. generated by the rotating coil Statement 3 is out of syllabus. It is wrong.
A. (1) only
B. (3) only
C. (1) & (2) only
D. (2) & (3) only
E. (1), (2) & (3)
46. (96)
The diagram shows the circuit of an ideal transformer with
a load connected to the secondary. Which of the following
is/are correct at the moment when switch S is closed ?
(1) The magnetic flux in the soft iron core will be
reduced.
(2) The back e.m.f. in the primary will drop.
Statement 2 is correct but is out of syllabus
(3) The primary current will increase.
A. (1) only
B. (3) only
C. (1) & (2) only
D. (2) & (3) only
E. (1), (2) & (3)
47. (96) Out of syllabus
In the circuit shown, both the cell E and the inductor L have negligible
resistance. The e.m.f of E is 10 V, and the resistors R1 and R2 are of
resistances 2 : and 3 : respectively. What will the current drawn from
the cell be
(I) at the moment when switch S is closed ?
(II) when the steady state is reached after closing switch S?
(I)
(II)
A. 2 A
0A
B. 2 A
2A
C. 2A
5A
D. 5 A
0A
E.
0A
5A

S
E
R2

R1

48. (96)

Two uniform magnetic fields of equal magnitude but with one into paper and the other out of paper, are separated
on the two sides of the y-axis as shown. A wire is bent into a closed loop OABO, which is shaped as a quarter of a
circle. It is rotated uniformly in a clockwise direction about O on the plane of the paper. Which of the following
graphs best shows the variation of current, I, in the loop with time, t, in one revolution ?
A.
B.
C.
D.
E.
I

t
0

I
t

MC Question
Chapter 15Electromagnetic Induction

(MC)15P10

49. (97)

The above figure shows a small coil, connected to a light beam galvanometer, placed in a region of uniform
magnetic field between the poles of a magnet. The plane of the coil is parallel to the pole faces. Which of the
following actions would produce a deflection of the galvanometer ?
(1) Moving the coil to and fro between the poles.
(2) Moving the coil away from the region between the pole faces.
(3) Rotating the coil about a diameter through an angle of 180o.
A. (1) only
B. (3) only
C. (1) & (2) only
D. (2) & (3) only
E. (1), (2) & (3)
50. (97)
Out of syllabus
A d.c. motor is connected to a source of constant voltage V. When rotating at a steady speed, the motor draws a
current I and the back e.m.f developed in its armature coil is H. Which of the following statements is/are correct ?
(1) The resistance of the armature coil is V/I.
(2) The mechanical power developed is IH.
(3) The back e.m.f s increases with the speed of rotation.
A. (1) only
B. (3) only
C. (1) & (2) only
D. (2) & (3) only
E. (1), (2) & (3)
51. (97)

A light rectangular wire frame ABCD moves with a uniform speed to the left across a region of uniform magnetic
field acting into paper. Which of the following is/are true at the instant shown in the figure?
(1) A current is flowing in the clockwise direction.
(2) The electric potential at B is higher than that at C.
(3) The side AD experiences a magnetic force acting to the right.
A. (1) only
B. (3) only
C. (1) & (2) only
D. (2) & (3) only
E. (1), (2) & (3)
52. (98) Out of syllabus

A coil of inductance 0.8 H is connected in series with a switch S and a cell of e.m.f. 2.0 V. The total resistance of
the circuit is 4.0 :. What is the initial rate of growth of the current when the switch is closed ?
B.0.4 A s-1
C. 0.5 A s-1
D. 1.6 A s-1
E. 2.5 A s-1
A. 0 A s-1
53. (99)
Each of the following three students states a conclusion after learning the formulae P = I V, P = V2 / R and P = I2R
in electricity. Which of these conclusions is/are correct ?
(1) The power dissipated by any electric appliance can be calculated from P = I V
(2) The power dissipated by a kettle is inversely proportional to its resistance as P = V2 / R.
(3) The power consumed by a running motor cannot be calculated from P = I2R.
A. (1) only
B. (3) only
C. (1) &, (2) only
D. (2) & (3) only
E. (1), (2) & (3)

MC Question
Chapter 15Electromagnetic Induction
2V

54. (99)

(MC)15P11

Out of syllabus
L

Neon lamp

In the above circuit, a coil L of large inductance, a 2 V light bulb B, a switch S and a 2 V battery are connected in
series. A neon lamp is connected across L. Which of the following statements is/are correct upon the opening of
switch S?
(1) A large e.m.f. is induced across coil L.
(2) The neon lamp lights up momentarily.
(3) Light bulb continues to light up until the neon lamp dies out.
A. (1) only
B. (3) only
C. (1) & (2) only
D. (2) & (3) only
E. (1), (2) & (3)
55. (99)
A coil of metal wire is placed on a plane perpendicular to a uniform magnetic field. The coil is rotated through 180o
about a diameter as shown.
o

180

The total charge circulated in the coil due to the induced current is independent of
A.
the area of the coil.
B.
the flux density of the magnetic field.
C.
the number of turns of the coil.
D.
the resistance of the coil.
E.
the time taken for the flux change.
56. (99)

search
coil
long straight
wire carrying
an a.c. current

6 cm

6 cm

To CRO

The figure shows a long straight wire carrying an a.c. current which lies on the plane of the paper. When a small
search coil is placed at B such that the plane of the coil is on the paper, the length of the trace on the CRO is 2 cm
(with the time base of the CRO switched off). If the search coil is moved to A, the length of the trace would be
A. 0.5 cm.
B. 1 cm.
C. 2 cm.
D. 4 cm.
E. 8 cm.

I
57. (00)

Out of syllabus

A constant p.d. is applied to a solenoid. The variation of current I with time t is shown. Which of the following can
be deduced from the shape of the graph.
A.
The solenoid has resistance as well as inductance.
B.
The inductance of the solenoid decrease as time increase.
C.
The inductance of the solenoid increase as time increase.
D.
The e.m.f. induced in the solenoid is proportional to the current.
E.
The e.m.f. induced in the solenoid is proportional to the rate of change of current.

MC Question
Chapter 15Electromagnetic Induction

(MC)15P12

58. (00)
Out of syllabus
When a simple motor rotating at a steady speed is suddenly jammed and comes to a stop, which of the following
statements is/are correct?
(1) The rate of change of magnetic flux through the coil of the motor will be very large.
(2) A large e.m.f. will be induced in the coil of the motor.
(3) A large current will flow through the coil of the motor.
A. (1) only
B. (3) only
C. (1) and (2) only
D. (2) and (3) only
E. (1), (2) and (3)
59. (00)

L
B

Out of syllabus
A

In the above circuit, A and B are identical light bulbs and L is a pure iron-cored inductor. Which of the following
statement is/are correct?
(1) When the switch S is closed, bulb A will light up first.
(2) After switch is closed for some time, A and B are equally bright.
(3) When switch S is open, bulb A will go out first.
A. (1) only
B. (3) only
C. (1) and (2) only
D. (2) and (3) only
E. (1), (2) and (3)
60. (01)
Magnetic flux is measured in weber. Which of the following is/arc equivalent to weber ?
(1) henry-ampere
(2) volt-second
(3) tesla-metre
A. (1) only
B. (3) only
C. (1) and (2) only
D. (2) and (3) only
61. (01)

E. (I),(2) and (3)

The figure shows a light and flexible conducting loop C freely hung on a smooth horizontal rail. A bar magnet PQ
approaches the loop from the right. Which of the following descriptions about this process is correct?
A.
If P is an N-pole, the loop will be repelled to the left and its area will increase slightly.
B.
If P is a S-pole, the loop will be repelled to the left and its area will increase slightly.
C.
If P is an N-pole, the loop will be attracted to the right and its area will decrease slightly.
D.
No matter whether P is an N-pole or a S-pole, the loop will be repelled to the left and its area will increase
slightly.
E.
No matter whether P is an N-pole or a S-pole, the loop will be repelled to the left and its area will decrease
slightly.
P

62. (02)

Q
60o

"
R

A metal rod of length " is inclined at 60 to rail PQ as shown. It is moved across a uniform magnetic field along the
direction of the two horizontal rails PQ and RS. If the rod moves with a uniform velocity v and the flux density of
the field is B, the e.m.f induced in the rod is
A.

B"v
2

B. B"v

C.

2 B"v
3

D.

3 B"v
2

MC Question
Chapter 15Electromagnetic Induction

(MC)15P13

63. (02)

uniform
magnetic field

A circular frame and a square frame, made from the same type of insulated metal wires, are placed in a uniform
magnetic field as shown. When the flux density of the field is increased at a steady rate, the ratio of the induced
current in the circular frame to that in the square frame is
C. S : 4
D. 2 : S
A. 1 : 1
B. l : S
64. (03)
The efficiency of a transformer is always less than 100%. Which of the following is not a reason for this ?
A.
eddy current flowing in the iron core
B.
heat dissipation in the primary coil
C.
leakage of magnetic flux
D.
work done against the back e.m.f. induced in the primary coil
65. (03)

Left

Right

wire

The figure shows a rectangular coil PQRS moving from left to right with a uniform speed across an insulated metal
wire in the plane of the coil. The wire carries a steady current I. Which of the following gives the correct sequence
for the direction of the current induced in the coil PQRS?
A.
Clockwise and then anticlockwise
B.
Anticlockwise and then clockwise
C.
Clockwise, then anticlockwise and finally clockwise again
D.
Anticlockwise, then clockwise and finally anticlockwise again
66. (04)
For the relation
secondary voltage
primary voltage
no. of turns in the secondary
no. of turns in the primary

=
to hold for a transformer, which of the following conditions are necessary?
(1)
(2)
(3)

The flux leakage of the iron core is negligible.


The energy loss in the primary coil is negligible.
The secondary coil is on open circuit.

A.
B.
C.
D.

(1) and (3) only


(1) and (2) only
(2) and (3) only
(1), (2) and (3)

MC Question
Chapter 15Electromagnetic Induction

(MC)15P14

67. (04)
The figure shows a secondary coil placed at one end of a solenoid, which is connected to an a.c. source. The
changing magnetic flux in the solenoid induces an alternating voltage in the secondary coil, which is connected to a
CRO.

Which of the following changes will NOT affect the amplitude of the induced voltage in the secondary coil?
A.
B.
C.
D.

Increasing the frequency of the a.c. in the solenoid


Increasing the number of turns on the secondary coil
Placing the secondary coil in the middle of solenoid, without changing its orientation
Replacing the solenoid with one of greater cross-sectional area while keeping the same current

Out of syllabus
68. (04)
Which of the following relations about units is/are correct?
(1) 1 F = 1 s:-1
(2) 1 H = 1 :s
(3) 1 Wb = 1 Vs-1
A.
B.
C.
D.

(1) only
(1) and (2) only
(2) and (3) only
(1), (2) and (3)

69. (05)
A rectangular coil is placed adjacent to a straight wire MN carrying an alternating current I which varies with time
t as shown in the graph. The wire MN is in the plane of the coil and the current I is positive (+) when it is in the
direction from M to N.

T/2
T

N
Starting from t = 0, how does the direction of the current induced in the coil vary in one period ?
A. Clockwise first and then anticlockwise
B. Anticlockwise first and then clockwise
o clockwise o anticlockwise
C. Clockwise o anticlockwise
o clockwise o anticlockwise
o clockwise
D. Anticlockwise

MC Question
Chapter 15Electromagnetic Induction

(MC)15P15

70. (05)
R

uniform
magnetic
field

Conducting rods PQ and RS are placed on two smooth, parallel, horizontal conducting rails. A uniform magnetic
field is directed into the plane of the paper as shown above. PQ is given an initial velocity to the right. Which of the
following statements is INCORRECT ?
A. The induced current is in the direction PQRS.
B. The magnetic force acting on the rod PQ is towards the left.
C. Rod RS starts moving towards the right.
D. Rod PQ moves with a uniform speed.
71. (05)

2:
dynamo
In the circuit, the two 12 V, 6 W lamps are operating at their rated values. The internal resistance of the dynamo is
2 :. What percentage of the electrical power generated by the dynamo is consumed by the two lamps ?
A. 75%
B. 86%
C. 92%
D. 100%
72. (05)

Out of syllabus

R
L

A cell of e.m.f. E is connected to a resistor R and an inductor L as shown. When switch S is closed, the variation
with time t of a quantity x is represented by the graph. The quantity x is
A.
the induced e.m.f. in L.
B.
the current in L.
C.
the inductance of L.
D.
the energy stored in L.

MC Question
Chapter 15Electromagnetic Induction

73.

(MC)15P16

(06)

The above graph shows the time variation of the total flux ) linking a rectangular coil rotating in a uniform
magnetic field within one period To. If the angular velocity of the coil is doubled, the graph would become
A.

B.

C.

D.

74. (06)

An alternating current in a closely-packed long solenoid produces a changing magnetic flux through a secondary
coil of a few turns placed around the solenoid. Which of the following changes can increase the amplitude of the
induced voltage at the secondary coil ?
(1) Increasing the frequency of the alternating current in the solenoid, keeping its amplitude the same
(2) Increasing the cross-sectional area of the secondary coil around the solenoid, keeping its number of turns the
(3) Using a solenoid packed exactly in the same way but with more turns, keeping its current the same
A. (1) only
B. (3) only
C. (1) & (2) only
D. (2) & (3) only

MC Question
Chapter 15Electromagnetic Induction

(MC)15P17

75. (06)

In the figure, the ideal transformer has a primary coil of 500 turns and two secondary coils of 200 turns and 50
turns. If an a.c. voltage of 240 V is applied to the primary, which of the following voltages can be obtained from the
secondary using different connections of the secondary coils ?
(1) 120 V
(2) 96 V
(3) 72 V
A. (1) only
B. (1) & (2) only
C. (2) & (3) only
D. (1), (2) & (3)
76. (06)

y
P

In the figure, a metal rod PQ is placed along the x-direction and it is at right angles to a uniform magnetic field
pointing into the plane of the paper. In which of the following cases will there be an e.m.f. induced along the length
of the metal rod ?
(1) Rotating the rod about an axis through its centre along the y-direction
(2) Moving the rod in the x-direction
(3) Moving the rod in the y-direction
A. (1) only
B. (3) only
C. (1) & (2) only
D. (2) & (3) only

Answer
1.C
16.D
31.B
46.E
61.E
76.B

2.C
17.B
32.E
47.C
62.D

3.A
18.D
33.E
48.D
63.A

4.E
19.A
34.E
49.D
64.D

5.C
20.B
35.E
50.D
65.C

6.C
21.A
36.A
51.A
66.D

7.C
22.A
37.E
52.E
67.D

8.E
23.D
38.D
53.E
68.B

9.D
24.D
39.B
54.C
69.A

10.B
25.C
40.C
55.E
70.D

11.D
26.D
41.B
56.D
71.B

12.B
27.B
42.E
57.A
72.A

13.E
28.A
43.E
58.B
73.B

14.B
29.B
44.D
59.C
74.A

15.A
30.E
45.C
60.C
75.D

MC Question
Chapter 16 Alternating Current

(MC)16P1

Q 1 - 4 - Out of syllabus
1.

(80)
Which of the graphs below correctly represents the variation of the magnitude F of the reactance of a capacitor
with the frequency f of the applied voltage?
F
A. F
B. F
C. F
D. F
E.

0
2.

(81)

D
E
J
G
0

A series circuit consisting of a pure inductor L, a pure capacitor C and a pure resistor R is connected across an a.c.
supply. The variations with applied frequency f of the resistance R, the reactance Xc of the capacitor, the reactance
XL, of the inductor and the impedance Z of the circuit are represented in the diagram. Which of the following
sequences places the curves in an order so that they represent the magnitudes of R, Xc, XL and Z?
A. D G E J
B. E D G J
C. J D E G
D. J E G D
E.
JGED
3.

(81)
S is a source of alternating voltage. In which of the following circuits will the current flowing through component
X be rectified, i.e. flowing in one direction only ?
(1)
(2)
(3)

A. (1) only
4.

B. (3) only

C. (1) & (2) only

D. (2) & (3) only

E. (1), (2) & (3)

(82)

A sinusoidal potential difference of fixed frequency and variable amplitude V is applied across a capacitor of
capacitance C. Which of the graphs below best represents the variation of the amplitude I of the current in the
circuit with V?
I
I
I
A.
B.
C. I
D.
E. I

V
0

V
0

V
0

MC Question
Chapter 16 Alternating Current

5.

(82)

(MC)16P2

V/V
8
t / ms
0

10 11

The diagram shows the waveform of an alternating p.d. V applied across a resistor R. What is the value of the
steady p.d. which should be applied across R to give the same heating effect?
A. 0 V

Q6 - 8 Out of syllabus
6.

C. 4 2 V

B. 16/3 V

D. 8 2 / 3 V

E. 8 V

(82)
X

In the circuit above, V is an a.c. source, X is an air-cored solenoid and R is a heating coil used to boil some liquid
in a vessel. Which of the following adjustments would decrease the time required to boil the liquid?
(1) The frequency of the a.c. supply is increased.
(2) A soft iron cylinder is inserted into the solenoid.
(3) A suitable capacitor is added in series with the circuit.
A. (1) only
B. (3) only
C. (1) & (2) only
D. (2) & (3) only
E. (1), (2) & (3)
7.

(83)
An alternating voltage V is applied to an LCR series circuit, where the inductive reactance XL, the capacitive
reactance Xc and the resistance R satisfy the relation XL = 2Xc = 2R. What is the phase difference between V and
the potential difference Vc across the capacitor?
A. S / 4
B. S/ 2
C. 3S / 4
D. S
E.
5S/ 4

8.

(83)

dynamo

A dynamo is connected to an a.c. ammeter X and a large inductor L as shown. The ammeter reading is I. If the
resistance of the circuit is negligible, and the rate of rotation of the dynamo is doubled, the ammeter reads
A. I/ 2
B. I
C.
D.
E.

2I

2I
4I

MC Question
Chapter 16 Alternating Current

(MC)16P3

Q9-13 Out of syllabus


9.

(83)
The resonant frequency of a series LCR circuit is fo. When an alternating signal V of frequency f is applied to this
circuit, a current I flows in it. Which of the following is/are correct?
(1) There is no phase difference between I and V when f = fo.
(2) I leads V when f is very much less than fo.
(3) V leads I when f is very much greater than fo.
A. (1) only
B. (3) only
C. (1) & (2) only
D. (2) & (3) only
E. (1), (2) & (3)

10.

(83)
An alternating current I= Io sinZt flows in a coil of inductance L and resistance R. A back e.m.f. E is induced in
the coil, where E is equal to
A. IZL.
B. L dI/dt.
C. I (ZL -R).
D. L dI/dt -IR.
E.
L dI/dt + IR.

11.

(83)

3V
6V
a.c.

For the circuit shown above, which of the graphs below best represents the variation of current I with time t ?
I
A
B.
C.
I
I
0

D. I

t
I

E.
0

12.

(84)
A capacitor (of negligible resistance) and a solenoid (whose resistance is not negligible) are connected in series
with an a.c. supply. The resonant frequency of the circuit can be increased by
(1) replacing the solenoid with one of lower resistance, but the same inductance.
(2) replacing the capacitor with one of greater capacitance.
(3) replacing the solenoid with one of lower inductance, but the same resistance.
A. (1) only
B. (3) only
C. (1) & (2) only
D. (2) & (3) only
E. (1), (2) & (3)

13.

(84)
input

load
resistance
R

The above figure shows a half-wave rectifier with a smoothing circuit. The time constant of C and R should be
A. small compared with the time of one cycle.
B. equal to half the time of one cycle.
C. equal to the time of one cycle.
D. equal to twice the time of one cycle.
E. large compared with the time of one cycle.

MC Question
Chapter 16 Alternating Current

(MC)16P4

Q14-16 Out of syllabus


14.

(84)

(3)

V
VL

VR

(2)
(4)

(1)
(5)

(6)

In Figure (a), an alternating voltage V is applied across an inductor L and a resistor R connected in series. VL
represents the instantaneous voltage across the inductor, and VR represents the voltage across the resistor. In the
phasor diagram in Figure (b), the phasor representing VR points in the direction (1). In which of the directions
shown in Figure (b) will the phasors representing VL and V point?
VL V
A. (1) (1)
B. (2) (3)
C. (3) (2)
D. (5) (6)
E.
(6) (5)

15.

(85)
A black box, known to contain an electrical device, is connected in series with a resistor R. The following circuit
is set up:
Y1
Y1
50 Hz
a.c.

black
box
Y2
R

Y2

To
double
beam
CRO

ground

The pattern observed on the double beam C.R.O. is also shown above.
The electrical device inside the box is
A. a capacitor. B. a diode.
C. an inductor.
D. a resistor.
16.

E. a transistor.

(86)

1 cm

An alternating current passes through a resistor R of resistance 10:. The voltage across the resistor is measured
by a C.R.O. The figure above shows the waveform on the screen. The y-sensitivity of the C.R.O. is 2V cm-1. The
average power dissipated in the resistor is
A. zero.
B. 0.1 W.
C. 0.4 W.
D. 0.8 W.
E. 1.6 W.

MC Question
Chapter 16 Alternating Current

Q17 - 20 Out of syllabus


17.

(86)

(MC)16P5

In an LCR -series circuit carrying a sinusoidal alternating current, the voltages across the resistor and the capacitor
are 4.0 V r.m.s. and 2.0 V r.m.s. respectively. If the applied voltage is 10.0 V r.m.s., then the r.m.s. voltage across
the inductor will be
A. 4.0 V.
B. 8.9 V.
C. 9.8 V.
D. 11.2 V
E. 13.8 V.
18.

(86)
Which of the following circuits can give a half wave rectification waveform on a C.R.O. ?
A
B.
C.
D.
To
C.R.O.

To
C.R.O.

To
C.R.O.

E
To
C.R.O.

19.

(87)
A 120 V, 60 W lamp is run from a 240 V, 50 Hz mains supply using a capacitor connected in series with the lamp
and supply. What is the theoretical value of the capacitor required to operate the lamp at its normal rating?
A. 3.8 PF
B. 6.6 PF
C. 7.7 PF
D. 13.3 PF
E.
83.3 PF

20.

(87)

In the above a.c. smoothing circuit, the ripple on the current passing through the load R can be reduced by
increasing
(1) the load resistor R.
(2) the capacitance C.
(3) the a.c. supply frequency.
A. (1) only
B. (3) only
C. (1) & (2) only
D. (2) & (3) only
E. (1), (2) & (3)
21.

(87)

voltage
V
0

The voltage of an a.c. source varies with time t as shown. The r.m.s. value of the applied voltage is
A. zero.

B. V / 2.

C. V / 2 .

D. V.

E.

2 V.

To
C.R.O.

MC Question
Chapter 16 Alternating Current
22.

23.

(MC)16P6

(88)
Q22- 25 Out of syllabus
A resistor R and a capacitor C are connected in series with an a.c. supply. The r.m.s. applied voltage and the r.m.s.
current are V and I, respectively. If the resistance of R is one half of the total impedance of the circuit, the power
consumed in the circuit will be
A. IV / 4.
B. IV / 2.
C.
D.

IV / 3 .
IV.

E.

3 IV / 2.

(89)
6 Vr.m.s.

An ideal inductor of reactance 1 : and an ideal resistor of resistance 2 : are joined in series across the terminals
of a 6 V r.m.s. a.c. source as shown. The power dissipated in the circuit is
A. 8 W.
B. 12 W.
C. 14.4 W.
D. 16.1 W.
E. 18 W.
24.

(89)

V
t
p

The circuit diagram shows an alternating potential difference connected across an inductor L, and the graphs
shows the variation of the source voltage V with time t. Which of the following is/are correct?
(1) At time p, both the current through L and the p.d. across it are zero.
(2) The current through L and the p.d. across it are 180o out of phase.
(3) At time q, the current through L is at its maximum value.
A. (1) only
B. (3) only
C. (1) & (2) only
D. (2) & (3) only
E. (1), (2) & (3)
25.

200 :

(89)
1 k:

Which of the following graphs best represents the variation with time t of the current I through the segment AB in
the above circuit ?
I
A. I
B. I
C.

t
D. I

E.

MC Question
Chapter 16 Alternating Current

(MC)16P7

Q26, 27 and 29 are out of syllabus


26.

27.

(89)
In the circuit shown, D is an ideal diode, R is a resistor of resistance 5 k:
and the alternating e.m.f. is represented by the equation E = 10 cos (5S t) ,
where E is in volts and t is in seconds. The power dissipated in R is
A. zero.
B. 2.5 mW.
C. 5 mW.
D. 10 mW.
E. 20 mW.

D
R

B1

(90)

B2

12 V
r.m.s.
a.c.

Bulbs B1 and B2 are connected to an a.c. supply and both bulbs are lit. If the a.c. supply is replaced by a 12 V d.c.
supply with negligible internal resistance, what will happen to the brightness of each bulb?
B2
B1
A.
unchanged
increases
B.
unchanged
decreases
C.
decreases
increases
D.
decreases
decreases
E.
increases
unchanged
28.

voltage

voltage

(91)

VP

VP

time

time

(a)
(b)
A signal generator produces either (a) a sinusoidal wave or (b) a square wave with the same peak value of voltage
as shown in the diagram. For a pure resistive load, the ratio of the mean power for (a) compared with (b) is
B. 2

A. I/ 2

29.

(91)
L

C. 1 / 2

D. 2

E. 2 2

impedance Z

1 PF

10 :
10 kHz

A coil and a capacitor are joined in series to a signal generator as shown in the diagram. As the frequency f of the
signal generator is increased, a graph of the total impedance Z of the circuit is plotted against f. If the capacitance
of C is 1 PF, what is the inductance of the coil?
A. 0.25 mH
B. 1.6 mH
C. 2.5 H
D. 10 H
E. 16 H

MC Question
Chapter 16 Alternating Current

(MC)16P8

Q30 - 34 Out of syllabus


30.

6V r.m.s.

(91)

XL = 1 :

R=2:

An inductor of reactance 1 : and a resistor of resistance 2 : are joined in series to the terminals of a 6 V (r.m.s.)
a.c. source as shown. What is the power dissipated in the circuit?
A. 8 W
B. 12 W
C. 14.4 W
D. 16.1 W
E. 18 W
31.

(92)
50 :

In the above circuit, if the sinusoidal a.c. source has a peak-to-peak voltage of 20 V, the r.m.s. current through the
50: resistor is
A. 0.07 A
B. 0.10 A
C. 0.14 A
D. 0.20 A
E. 0.28 A

32.

(92)

E = A sin Zt
D1

I/mA

D2
R

V/V
0.6

In the circuit shown above, D1, D2 are two diodes used to rectify a sinusoidal a.c. supply. Each of the diodes has
the I-V characteristic as shown. For a current to flow through R, the minimum value of A is
A. 0.6 V

33.

(92)

B. 1.2 V

C. 0.3 2 V

D. 0.6 2 V

E. 1.2 2 V

0.40 PF
0.20 PF

In the above circuit, the a.c. source has a r.m.s. voltage of 5.0 V alternating at 50 Hz. What is the r.m.s. current?
A. 0.033 PA
B. 0.12 PA
C. 0.21 mA
D. 120 mA
E. 210 mA
34.

(93)
Which of the following physical quantities take(s) the unit ohm (:)?
(1) resistance
(2) reactance
(3) impedance
A. (1) only
B. (3) only
C. (1) & (2) only
D. (2) & (3) only

E. (1), (2) & (3)

MC Question
Chapter 16 Alternating Current

Q35-37 Out of syllabus


35.

(MC)16P9

a.c. supply

(93)

The above figure shows an a.c. circuit with a filament lamp B connected in series with a variable inductor L and a
capacitor C. The frequency of the a.c. supply can be varied. Initially the applied voltage leads the current in the
circuit. Which of the following methods will make the filament lamp B brighter?
(1) Connect a capacitor in parallel with C.
(2) Increase the inductance of L.
(3) Decrease the frequency of the a.c. supply.
A. (1) only
B. (3) only
C. (1) & (2) only
D. (2) & (3) only
E. (1), (2) & (3)

36.

100 k :

(94)
a.c.
source

to CRO

In the above circuit, the waveform across the diode shown on the screen of the CRO should be

37.

A.

B.

D.

E.

C.

Vr.m.s.

(94)
R

VR

VC

VL

In the LCR series circuit shown above, the frequency of the a.c. source is varied while its r.m.s, output voltage
Vr.m.s. is kept constant. The voltmeter connected across R, C and L gives r.m.s. readings VR, VC and VL respectively.
Which of the following statements is/are correct?
(1) At any instant, the current through R, C and L is the same.
(2) At resonance VL = Vc.
(3) At any instant, Vr.m.s. = VR + VC + VL
A. (1) only
B. (3) only
C. (1) & (2) only
D. (2) & (3) only
E. (1), (2) & (3)

MC Question
Chapter 16 Alternating Current

(MC)16P10

Q38 - 40 Out of syllabus


38.

(94)

x
0
A plot of two physical quantities x and y yields the above graph. Which of the following could be those two
physical quantities?

Quantity x
A.
B.
C.
D.
E.

39.

40.

Quantity y

radius of a satellites orbit


distance from a point charge
extension of a spring
time
frequency of an a.c. supply

period of the satellite


electric potential due to the point charge
energy stored in the spring
charge on a charging capacitor
inductive reactance of an inductor connected to the a.c. supply

(95)
A black box containing two unknown components is connected
to a cell, a resistor and an ammeter as shown. A current flows
steadily no matter which way the box's terminals are connected to
the cell and the same ammeter reading is obtained. The two
components in the black box could be
A. two diodes in series.
B. two diodes in parallel.
C. two capacitors in parallel.
D. a diode and a resistor in parallel.
E. a diode and a capacitor in parallel.
A

(95)

black
box

B
S

to CRO

A capacitor C and an inductor L with some resistance are connected to a battery as shown in the figure. When
switch S is moved from A to B, what is the trace observed on the screen of the CRO with the time base on?
A.
B.
C.

D.

E.

MC Question
Chapter 16 Alternating Current

(MC)16P11

41.

(95)
Q41-45 Out of syllabus
In an LCR series circuit, the total impedance across the three components is the same at both frequencies 25 Hz
and 225 Hz. What is the resonant frequency of the circuit?
A. 50 Hz
B. 75 Hz
C. 100 Hz
D. 125 Hz
E.
150 Hz

42.

(95)
reactive
component

An ac. power source of negligible impedance is connected in series with a reactive component and a resistor of
resistance R. At the source frequency, the source voltage has r.m.s. value V, the component's reactance is X and
the circuit's total impedance is Z. The power consumption in the circuit is
B. V2/Z
C. V2R / Z2
D. V2R / X2
E. V2X / Z2
A. V2/ R
43.

44.

(96)
The reactance of a solenoid depends on
(1) the frequency of the applied a.c. voltage.
(2) the inductance of the solenoid.
(3) the resistance of the solenoid.
A. (1) only
B. (3) only
C. (1) & (2) only

D. (2) & (3) only

E. (1), (2) & (3)

(96)

When a CRO is connected to a circuit, the trace obtained is as shown above. To which of the following circuits
has the CRO been connected ?
A.
B.
C.
D.
E.
To CRO

45.

To CRO

(95)
0

To CRO

To CRO

V
time

t1 t2

A sinusoidal voltage V is applied across a capacitor C. Which of the following statements is/are correct ?
(1) At time t1, the charge in the capacitor is at a maximum.
(2) At time t2, the current in the circuit is at a maximum.
(3) The current in the circuit leads the voltage V.
A. (1) only
B. (3) only
C. (1) & (2) only
D. (2) & (3) only
E. (1), (2) & (3)

To CRO

MC Question
Chapter 16 Alternating Current

(MC)16P12

Q46-48 &Q50 are out of syllabus


46.

(97)
The instantaneous value of an alternating current I (in A) at time t (in s) is given by the equation
I = 2 2 sin 50St
Which of the following description is/are correct ?
(1) The a.c. frequency is 50 Hz.

(2) The value of the current varies between 2 2 A and +2 2 A.


(3) The r.ms. value of the current is 2 A.
A. (1) only
B. (3) only
C. (1) & (2) only
D. (2) & (3) only
47.

48.

49.

(97)
In the circuit shown, a signal generator of fixed output voltage is
connected in series with a resistor R and a capacitor C. When the
frequency of the signal generator is gradually increased, which of the
following is the correct variation of the ammeter and voltmeter readings?
Ammeter reading
Voltmeter reading
A. increases throughout
increases throughout
B. increases throughout
decreases throughout
C. increases throughout
increases and then decreases
D. increases and then decreases
decreases throughout
E.
increases and then decreases
decreases and than increases
(97)
A variable capacitor can be used
(1) as the volume control of an amplifier
(2) to tune a radio to receive a certain channel.
(3) to change the power factor of an a.c. circuit.
A. (1) only
B. (3) only
C. (1) & (2) only

D. (2) & (3) only

E. (1), (2) & (3)

signal generator

E. (1), (2) & (3)

(98)
current/A
5.0
0
-3.0

time /ms
1

An alternating current flowing through a resistor varies with time as shown above. What is the value of the direct
current that gives the same heating effect as the alternating current?
A. 3.9 A
B. 4.0 A
C. 4.1 A
D. 4.2 A
E. 4.3 A

50.

(98)

L
C
The root-mean-square voltages across the resistor R, capacitor C and inductor L of an RCL series circuit are
respectively 4 V, 1 V and 4 V. What is the phase difference between the applied voltage and the current flowing in
the circuit?
A. 30o
B. 37o
C. 45o
D. 53o
E. 60o

MC Question
Chapter 16 Alternating Current

(MC)16P13

Q51-53 Out of syllabus


51.

(99)

The above figure shows an ac. circuit in which a variable resistor R is connected in series with an air-cored
inductor L of negligible resistance. The a.c. supply has constant r.m.s. voltage and its frequency can be varied.
Which of the following methods, on its own can increase the r.m.s voltage across the variable resistor R?
(1) Inserting an iron core in the inductor L.
(2) Increasing the frequency of the a.c. supply.
(3) Increasing the resistance of R.
A. (1) only
B. (3) only
C. (1) & (2) only
D. (2) & (3) only
E. (1), (2) & (3)

52.

(00)
D
R

The figure shows a bridge rectifier circuit in which all the diodes are assumed ideal. The source is a sinusoidal a.c.
supply. Which of the following traces ( I , II or III ) would be displayed on a CRO connected across the load
resistor R if
(1) the diode D were reversed in the circuit,
(2) the diode D were removed leaving a break in the circuit ?
I

A.
B.
C.
D.
E.
53.

II

(1)
I
II
II
III
III

III

(2)
II
II
III
II
III

(00)
C

The figure shows an LC oscillatory circuit in which C is a capacitor and L is an air-cored inductor. At a certain
instant the magnetic flux inside L is pointing upward as shown. Which of the following descriptions of the
capacitor is/are possible at this instant ?
(1) The capacitor is charging with the upper plate positively charged.
(2) The capacitor is charging with the lower plate positively charged.
(3) The capacitor is discharging with the upper plate positively charged.
(4) The capacitor is discharging with the lower plate positively charged.
A. (1) only
B. (1) or (3) only
C. (1) or (4) only
D. (2) or (3) only
E. (2) or (4) only

MC Question
Chapter 16 Alternating Current

(MC)16P14

Q54, 56, 57 Out of syllabus


54.

(00)

C
R
A capacitor C and a resistor R are connected in series to an a.c. source of constant output voltage. Which of the
following statements is/are correct ?
(1) The current leads the applied voltage.
(2) The impedance of the circuit decreases when the frequency increases.
(3) The power factor of the circuit decreases when the frequency increases.
A. (1) only
B. (3) only
C. (1) & (2) only
D. (2) & (3) only
E. (1), (2) & (3)
55.

(00)
The following diagrams show the currents of sinusoidal waveform and square waveform passing in turn through a
resistor. The average power dissipated is the same in both cases.

I /A

1
0

2T

Io
0

time

time
T

2T

Given that the peak value of the sinusoidal current is 1 A. What is the value of Io of the square waveform current ?
1
1
A
B.
A
C. 1 A
D. 2 A
E. 2 2 A
A.
2
2
56.

(01)
In an a.c. circuit, two components, one of resistance 5 : and the other of inductive reactance 10 :, are connected
in series to the source. The r.m.s. current is 2 A. What is the average power dissipated as heat in the circuit ?
A. 10 W
B. 20 W
C. 40 W
D. 45 W
E. 60 W

57.

(02)

Y1

Y2

In the circuit shown, the voltages across the resistor and the capacitor are respectively fed to the input terminals
Y1and Y2 of a dual trace CRO where E is the common earth terminal. Which of the following pairs of traces could
be obtained ? (The trace of Y1is represented by a solid curve and the trace of Y2 is represented by a dotted one.).
A.
B.
C.
D.

MC Question
Chapter 16 Alternating Current

(MC)16P15

Q58-61 Out of syllabus


58. (02)
I

V
Which of the following combinations of a cell, an ideal diode and a resistor will give the above I-V relationship,
where I and V are the applied current and voltage respectively?

59.

A.

B.

C.

D.

(03)

In the above circuit the a.c. supply has a variable frequency but has a constant peak voltage and zero impedance. L
is a coil of many turns. Which of the following ways can increase the brightness of the bulb ?
(1) Increasing the frequency of the a.c. supply
(2) Inserting a soft iron core into the coil L
(3) Adding a suitable capacitor in series with L
A. (1) only
B. (3) only
C. (1) and (2) only
D. (2) and (3) only
60.

(03)
An a.c. circuit has a total impedance of 13 : and a total reactance of 12 :. What is the average power dissipated
in the circuit when the r.m.s. current passing through it is 2 A ?
A. 4 W
B. 20 W
C. 48 W
D. 52 W

61.

(04)

In the above LCR series circuit, L is a pure inductor. Which of the following statements is INCORRECT?
A.
B.
C.
D.

Energy is dissipated in resistor R but not in C or L.


Electrical energy is transferred between C and L.
The impedance of the circuit depends on the frequency of the a.c. supply.
The current through C is 180q out of phase with that through L.

MC Question
Chapter 16 Alternating Current

(MC)16P16

Out of syllabus
62. (04)
The resistance of a reverse-biased diode is to be determined by the ammeter-voltmeter method using moving-coil
meters. Which of the following circuits is correctly connected?
A.

B.

C.

D.

63.

(04)
I/A
2

0.01

0.02

0.03

0.04

0.05

t/s

The figure shows how a half-wave rectified sinusoidal a.c. flowing through a resistor varies with time. What is the
value of a steady d.c. that gives the same heating effect as the rectified a.c.?
A.
B.
C.
D.
64.

1
A
2
1

2
1A

2A

Out of syllabus
(05)
A coil of inductance 3.2 u 10-2 H and resistance 20 : is connected to a 50 Hz a.c. source. What is the impedance
of the coil at 50 Hz ?
A. 10 :
B. 20 :
C. 22 :
D. 30 :

MC Question
Chapter 16 Alternating Current

(MC)16P17

Out of syllabus
65.

(05)

The above figure shows a rectifier circuit with a smoothing capacitor C which has been installed to provide
current to a load resistor R. Which of the following would reduce the ripple on the current through R?
(1) Increasing the capacitance of C
(2) Increasing the resistance of R
(3) Increasing the a.c. supply frequency
A. (1) only
B. (1) & (3) only
C. (2) & (3) only
D. (1), (2) & (3)
66.

(06)

Out of syllabus
1V
4V

4V

The r.m.s. voltages across the three components of the above RCL series circuit are indicated as shown. What is
the r.m.s. value of the applied voltage ?
A. 9V
B. 7 V
C. 5 V
D. 3 V

Answer
1. B

2. E

3. E

4. A

5. D

6. B

7. C

8. B

9. E

10. B

11. D
21. D
31. B
41. B

12. B
22. B
32. B
42. C

13. E
23. C
33. C
43. C

14. C
24. B
34. E
44. E

15. A
25. D
35. B
45. E

16. D
26. C
36. E
46. D

17. D
27. A
37. C
47. B

18. E
28. C
38. E
48. D

19. C
29. A
39. B
49. C

20. E
30. C
40. C
50. B

51. B
61. D

52. D
62. D

53. D
63. C

54. C
64. C

55. C
65. D

56. B
66. C

57. C

58. C

59. B

60. B

MC
Chapter 23 Radioactivity
1.

(MC)23P1

(80)
Which of the graphs below correctly represents the variation of the activity X of a radioactive sample with the
number N of undecayed nuclei in the sample?
A.
B.
C.
D.
E.
X

2.

(81)
A stationary radioactive nucleus of mass N units emits an alpha particle of mass 4 units, leaving a residual nucleus
of mass (N 4) units. The ratio of the kinetic energy of the alpha particle to the kinetic energy of the residual
nucleus is
A. (N 4)/4
B. N2/ (N 4)2
C. (N 4)2/N
D. (N 4)2/42
E.
(N 4)3/43

3.

(82)
A radioactive source with a half-life t1/2 initially contains N atoms of the radioactive element. The energy released
in each disintegration is E. What is the total energy released in time 2t1/2 ?
1
NE
A.
4
1
NE
B.
2
3
C.
NE
4
1
NE t1/2
D.
2
E.
NE t1/2

4.

(83)
A radioactive source of gamma rays has a half-life of 2 days. A Geiger counter placed 3 cm from the source
initially has a count-rate of 1440 per minute. After 6 days the counter is moved back to a distance of 9 cm from
the source, and its count-rate, in counts per minute, is then
A. 20.
B. 60.
C. 180.
D. 320.
E.
360.

5.

(85)

A dish containing a strong D-source is placed inside a gold leaf electroscope containing dry air. If the gold-leaf is
originally positively charged, what will happen to it after a few minutes ?
A. It will increase in divergence.
B. It will increase in divergence and then decrease.
C. It will collapse.
D. It will collapse and then re-diverge.
E.
There will be no change in divergence.

MC
Chapter 23 Radioactivity

(MC)23P2

6.

(85)
Proactinium extracted from a solution of uranyl nitrate decays with a half-life of 72 s. The value of the decay
constant is
A. 9.6 u 10-3 s.
B. 9.6 u 10-3 s-1.
C. 0.014 s-1.
D. 49.9 s.
E.
49.9 s-1.

7.

(88)
An D-source originally consisted entirely of the element polonium. After the emission of a single D-particle, each
polonium atom becomes an atom of lead. At the end of two years, the source was found to contain 98% lead and
2% polonium. At the end of one year, the sample would have had the approximate composition :
A. 14% lead, 86% polonium.
B. 25% lead, 75% polonium.
C. 50% lead, 50% polonium.
D. 75% lead, 25% polonium.
E.
86% lead, 14% polonium.

8.

(89)
Which of the following statements is/are true for beta particles originating from nuclear disintegration ?
(1) Beta particles travel at the speed of light.
(2) Emitted beta particles from a nuclide have a continuous energy distribution.
(3) Beta decay is accompanied by the emission of neutrinos.
A. (1) only
B. (3) only
C. (1) & (2) only
D. (2) & (3) only
E. (1), (2) & (3)

9.

(89)
A gold-leaf electroscope is positively charged. Which of the following objects, when placed near the cap, will
cause the leaf of the electroscope to fall ?
(1) a lighted match
(2) a strong D-source
(3) an earthed metal plate
A. (1) only
B. (3) only
C. (1) & (2) only
D. (2) & (3) only
E. (1), (2) & (3)

10.

(89)

R
S

A neutral particle decays in a short time into a proton and a negative particle. The initial and subsequent paths of
the particles in a magnetic field acting into the plane of the paper are shown above. Which of the following gives
the respective paths for the neutral particle, the proton and the negative particle ?
The proton
Negative particle
Neutral particle
A.
R
S
T
B.
R
T
S
C.
S
R
T
D.
S
T
R
E.
T
R
S

MC
Chapter 23 Radioactivity

(MC)23P3

11.

(90)
A radioactive source is placed in front of a GM tube connected to a counter. Various absorbers are placed between
the source and the GM tube and the count-rate recorded. The following results were obtained:
Counts per minute
Absorber
no absorber
711
a sheet of paper
508
5 mm thick aluminium sheet
493
25 mm thick lead block
218
It can be deduced from these results that the radiation(s) emitted by the source is/are
A. Dand J rays only
B. E and J rays only
C. D rays only
D. E rays only
E.
J rays only

12.

(91)
The graph below shows how the count rate A of a radioactive source as measured by a Geiger-counter varies with
time t : (Assume in 12 = 2.5)
ln A

3
2
1
10 20 30
The relationship between A and t is
A. A = 2.5e-10t
B. A = 12e10t
C. A = 2.5e-0.1t
D. A = 12 e-0.1t
E.
A = 12 e0.1t

time t /s

13.

(92)
A radioactive source consists of a mixture of two radioisotopes P and Q. P has a half-life of 1 hour and Q has a
half-life of 2 hours. Both P and Q have stable daughter nuclei. The initial activity recorded by a counter is
600/min. After 4 hours the counter registers an activity of 60 counts per min. What was the contribution of P to
the initial count rate (in counts per min.) ?
A. 120
B. 200
C. 360
D. 400
E.
480

14.

(93)
The number of radioactive nuclides in two different samples P and Q are initially 4N and N respectively. If the
half-life of P is t and that of Q is 2t, the number of radioactive nuclides in P will be the same as the number of
radioactive nuclides in Q after a time of
A. t/2
B. t
C. 2t
D. 4t
E.
8t

MC
Chapter 23 Radioactivity

(MC)23P4

15.

(94)
When a radioactive atom emits J-rays, which of the following statements about the atom is/are correct?
(1) Its mass number remains unchanged.
(2) The energy of the atomic nucleus decreases.
(3) An electron falls from a higher energy level to a lower one.
A. (1) only
B. (3) only
C. (1) & (2) only
D. (2) & (3) only
E. (1), (2) & (3)

16.

(94)
A detector is used for monitoring an D-source and a reading of 120 units is observed. After a time equal to the
half-life of the D-source, the reading has fallen to 64 units. If a 5 mm thick lead sheet is inserted between the
D-source and the detector, the reading would probably be
A. 0 unit
B. 4 units
C. 8 units
D. 16 units
E.
32 units

17.

(94)
A stationary uranium-238 nucleus undergoes D-decay. What is the ratio of the kinetic energy of the daughter
nucleus to that of the D-particle?
A. 238 : 4
B. 4 : 238
C. 234 : 4
D. 4 : 234
E.
1:1

18.

(95)
226
88 Ra decays to
(1)
(2)

222
86 Rn with

a half-life of 1 600 years. Which of the following statements is/are correct?

D particle is produced in the decay.


All 226
88 Ra has decayed after 3 200 years.

(3) The half-life of 226


88 Ra can be shortened by heating.
A. (1) only
B. (3) only
C. (1) & (2) only
19.

(96)
A radioactive source is tested as follows :
Absorber placed between
source and GM counter
(1) Thin aluminium foil (0.2 mm)
(2) Thin lead sheet (2 mm)
(3) Thick lead sheet (20 mm)

D. (2) & (3) only

E. (1), (2) & (3)

Effect on count rate


Falls appreciably.
No significant difference with (1).
Falls below that in (1).

What type(s) of radiation does the source emit ?


A. D only
B. E only
C. J only
D. D and J only
E.
E and J only
20.

(96)
A counter is placed near a very weak radioactive source which has a half-life of 1 hour. The counter registers 100
counts/min at noon and 80 counts/min at 1 p.m. The expected count rate in counts/min at 3 p.m. on the same day
is
A. 40
B. 50
C. 55
D. 60
E.
65

MC
Chapter 23 Radioactivity
21.

(MC)23P5

(97)
226
206
88 Ra is one of the nuclides in the uranium decay series. If the stable end-product of this series is 82 Pb , the
number of E-particles emitted between the 226
88 Ra stage and the end of the series is
A. 4
B. 6
C. 10
D. 14
E.
20

22.

(98)
The activity of a sample of radioactive isotopes decreases to 1/3 of its initial value in 12 s. How much more time
would be required for the activity to decrease to 1/9 of its initial value ?
A. 4 s
B. 8s
C. 12s
D. 16s
E.
24s

23.

(98)
A radioactive sample, initially consists of only nuclide X, decays by the emission of an alpha particle to form a
stable daughter nuclide Y. Which of the following quantities will decrease with time?
(1) The rate of decay of nuclide X.
(2) The rate of growth of nuclide Y.
(3) The rate of emission of alpha particles.
A. (1) only
B. (3) only
C. (1) & (2) only
D. (2) & (3) only
E. (1), (2) & (3)

24.

(99)

Out of syllabus

For stable nuclides, the number of neutrons N when plotted against the number of protons Z would give points
lying within the above shaded region. Which of the following statements is/are correct ?
(1) The heavy stable nuclides have more neutrons than protons.
(2) Unstable nuclides decay to produce new nuclides closer to the shaded region.
(3) Unstable nuclides above the shaded region usually decay through D-emission.
A. (1) only
B. (3) only
C. (1) & (2) only
D. (2) & (3) only
E. (1), (2) & (3)
25.

(99)
Carbon-14 is radioactive and undergoes E-decay with a half-life of about 5600 years. A GM-tube together with a
scaler are used in an experiment to determine the concentration of carbon-14 in a piece of excavated wood. To
find an accurate value for the activity of the excavated wood, the experimenter should
(1) prepare a sample of pure carbon-14 from the piece of excavated wood.
(2) take a count over as long a period of time as possible.
(3) take a second count without the presence of the excavated wood.
A. (1) only
B. (3) only
C. (1) & (2) only
D. (2) & (3) only
E. (1), (2) & (3)

MC
Chapter 23 Radioactivity

26.

(MC)23P6

(00)
A GM counter is placed close to and in front of a radioactive source which emits both D and J radiation. The
count rate recorded is 500 counts per minute while the background count rate is 50 counts per minute. Three
different materials are placed in turn between the source and the counter. The following results are obtained.
Material
(Nil)
Cardboard
1 mm of aluminium
5 mm of lead

Recorded count rate / counts per minute


500
x
y
z

Which of the following is a suitable set of values for x, y and z ?


x
y
z
A. 350 350 150
B. 350 150 50
C. 350 150 0
D. 150 150 50
E.
150 50 50
27.

(01)
The activity of a radioactive sample was 70 Bq at time t = 5 minutes and 49 Bq at t = 10 minutes. Deduce its
activity at time t = 0.
A. 112 Bq
B. 100 Bq
C. 95 Bq
D. 91 Bq
E.
80 Bq

28.

(01)
The table below gives the corrected count rate (in counts per minute) from three samples of radioactive isotopes at
three different times.
Isotope
0 min
20 min
40 min
X
480
243
119
Y
135
32
9
Z
168
118
93
The above data show that
(1) X produces the most penetrating radiation.
(2) Y has the largest decay constant.
(3) Z has the longest half-life.
A. (1) only
B. (3) only
C. (1) & (2) only
D. (2) & (3) only
E. (1), (2) & (3)

29.

(02)

ln(A/Bq)
5

t/s
400

The graph above shows how the logarithm of the activity A of a radioactive isotope varies with time t. What is the
half-life of the isotope?
A. 200 s
B. 55 s
C. 24 s
D. 0.42 s

MC
Chapter 23 Radioactivity

30.

(MC)23P7

(03)
A nuclide in a radioactive sample has a constant chance of

1
to decay in one second. What is the approximate
10 6

half-life of the sample?


A. 1 day
B. 1 week
C. 1 month
D. 1 year
31.

(03)
The average background count rate, in s-1, in Hong Kong is of the order of
A. 10-2.
B. 100.
C. 102.
D. 104.

32.

(04)
The activity of a freshly prepared sample of 60Co is 1.0 u106 Bq. The half-life of 60Co is 5.3 years. Estimate the
number of 60Co nuclei in the sample that decay in the first day.
A. 5.2 u 102
B. 3.2 u 108
C. 8.6 u 1010
D. It cannot be estimated as the initial number of nuclei in the sample is not given.

33.

(04)
A E-source is placed in front of a GM tube connected to a counter. The corrected count rate recorded is NOT
equal to the activity of the source because
(1) most E-particles cannot pass through the mica end-window of the GM tube.
(2) not all the E-particles are emitted in the direction of the GM tube.
(3) E-particles arriving within the dead time of the GM tube cannot be resolved.
A. (1) and (3) only
B. (1) and (2) only
C. (2) and (3) only
D. (1), (2) and (3)

34.

(05)
A detector placed near a source of gamma radiation records a count rate of 960 counts per second. The half-life of
the source is 12 hours. A slab of material of thickness 6 cm is then placed between the source and the detector.
The half-value thickness of the material is 2 cm (i.e. the intensity of the radiation would reduce by half after
passing through 2 cm of the material). Estimate the count rate recorded by the detector after one day. (Neglect any
background radiation.)
A.
l60 counts per second
B.
80 counts per second
C.
60 counts per second
D.
30 counts per second

35.

(05)
On which of the following does the activity of a radioactive source depend ?
(1) the number of active nuclides in the source
(2) the half-life of the source
(3) the nature of the nuclear radiation emitted by the source
A. (1) only
B. (3) only
C. (1) & (2) only
D. (2) & (3) only

MC
Chapter 23 Radioactivity

(MC)23P8

36.

(06)
Which of the following gives the correct meaning of the decay constant of a radioactive substance ?
A. It is the rate of disintegrations of the substance.
B. It is the number of disintegrations of the substance occurring on one half-life of the substance.
C. It is the fraction of the active nuclei present that decay in one second.
D. It is equal to the reciprocal of the half-life of the substance.

37.

(06)
Some typical radiation doses are tabulated as follows :
Radiation dose
Watching television
0.005 mSv/hr for watching every day in a year
Flying in an aircraft
0.001 mSv/hr
X-ray check
0.020 mSv each time
Arrange the following in ascending order of total radiation dose in one year.
(1) Watching television for 4 hours every day
(2) Travelling on an aircraft for 10 hours every month
(3) Taking X-ray check every 6 months
A.
(1), (2), (3)
B.
(2), (1), (3)
C.
(1), (3), (2)
D.
(3), (1), (2)

Answer
1. C
11. A
21. A
31. B

2. A
12. D
22. C
32. C

3. C
13. E
23. E
33. C

4. A
14. D
24. C
34. D

5. C
15. C
25. D
35. C

6. B
16. C
26. A
36. C

7. E
17. D
27. B
37. C

8. D
18. A
28. D

9. E
19. D
29. B

10. D
20. E
30. B

MC
Chapter 24 The Nucleus

1.

Energy and Use of Energy


(80)
Which of the following equations represent possible reactions ?
(1)

10
5B 

(2)

210
o 210
83 Bi 
84 Po

(3)
A. (1) only
2.

(MC)24P1

(81)
Given :

neutron 
o 63 Li  42 He
 beta particle

14
4
o178 O 
7 N  2 He 

proton
B. (3) only

C. (1) & (2) only

D. (2) & (3) only

E. (1), (2) & (3)

Energy and Use of Energy

the mass of a proton is 1.6733 u 10-27 kg


the mass of a neutron is 1.6744 u 10-27 kg
the mass of an alpha particle is 6.6443 u 10-27 kg
the electronic charge e is -1.6 u 10-19 C
the speed of light in vacuum is 3.00 u 108 m s-1
The binding energy per nucleon in a helium nucleus is
A. 1.15 u 10-12 J
B. 2.30 u 10-12 J
C. 4.60 u 10-12 J
D. 1.44 u 107 eV
E.
2.87 u 107 eV

3.

(83)
Energy and Use of Energy
The binding energy per nucleon varies with mass number as shown below:
binding
energy /
nucleon

R
Q
P

mass
number

If nuclear energy were to be generated by the fusion of the nuclei of an element X, which of the points P, Q and R
would represent possible positions of X on the graph ?
(1) P
(2) Q
(3) R
A. (1) only
B. (3) only
C. (1) & (2) only
D. (2) & (3) only
E. (1), (2) & (3)
4.

5.

Atomic World
(83)
When alpha particles are fired in a beam at a thin sheet of gold, they are scattered through a wide range of angles,
from zero to 180 from the incident beam. This experiment shows that the gold atom has
(1) electrons in different orbits.
(2) a volume consisting mainly of empty space.
(3) a small, dense, charged nucleus.
A. (1) only
B. (3) only
C. (1) & (2) only
D. (2) & (3) only
E. (1), (2) & (3)
Energy and Use of Energy
(84)
3
He is an isotope of helium. Given that the masses of a proton, a neutron and a 3He nucleus are 938.3 MeV/c2 ,
939.6 MeV/c2 and 2808.5 MeV/c2 respectively, the binding energy of 3He is
A. zero.
B. 7.7 MeV.
C. 9.0 MeV.
D. 930.6 MeV.
E.
947.3 MeV.

MC
Chapter 24 The Nucleus

6.

7.

8.

(MC)24P2

Energy and Use of Energy


(84)
In a controlled thermal fission reactor, the function of the moderator is to reduce
A. the speed of the neutrons released on fission.
B. the rate of production of the neutrons.
C. the energy generated in the nuclear reactor.
D. the amount of radioactive radiation produced in the nuclear reactor.
E.
the rate of disintegration of the 235U nucleus.
Energy and Use of Energy
(86)
When fission occurs in a heavy nucleus, the two nuclei produced
(1) are stable.
(2) contain more protons than neutrons.
(3) have more binding energy per nucleon than the original nucleus.
A. (1) only
B. (3) only
C. (1) & (2) only
D. (2) & (3) only
(87)
P

E. (1), (2) & (3)

gold
foil
Q

Atomic World

In an experiment on D-particle scattering, D-particles are directed onto a gold foil, and detectors are placed at
positions P, Q and R as shown. What is the distribution of D-particles as recorded at P, Q and R respectively ?
P
Q
R
A.
all
none
none
B.
none
none
all
C.
most
some
some
D.
most
some
few
E.
few
some
most
9.

(88)
Energy and Use of Energy
In a controlled thermal fission reactor, the use of the control rods will NOT affect
A. the speed of the neutrons released on fission.
B. the rate of production of the neutrons.
C. the energy generated in the nuclear reactor.
D. the amount of radioactive radiation produced in the nuclear reactor.
E.
the rate of disintegration of the 235U nucleus.

10.

(89)
A helium atom, a hydrogen atom and a neutron have masses of 4.003u, 1.008u and 1.009u (unified atomic mass
units) respectively. Assuming that hydrogen atom and neutrons can fuse to form helium, the binding energy of a
helium nucleus is
A. 0.0031u.
B. 0.031u.
C. 1.017u.
D. 2.014u.
E.
2.017u.

11.

Energy and Use of Energy


(91)
Which of the following statements concerning a moderator in a nuclear fission reactor is/are correct ?
(1) It is used to slow down the neutrons released during fission.
(2) It is used to absorb excess neutrons.
(3) Boron steel is a suitable material.
A. (1) only
B. (3) only
C. (1) & (2) only
D. (2) & (3) only
E. (1), (2) & (3)

MC
Chapter 24 The Nucleus
12.

13.

(MC)24P3

Atomic World
(92)
Which of the following conclusions could NOT be deduced from Rutherford's scattering experiment ?
(1) Alpha particles are helium nuclei.
(2) There are discrete energy levels in an atom.
(3) The positive charge in an atom is confined to a very small region.
A. (1) only
B. (3) only
C. (1) & (2) only
D. (2) & (3) only
E. (1), (2) & (3)
(92)
 alpha particle 
o X  proton
In the above nuclear reaction, X is
A. 178 O
B. 179 F
14
7N

14.

15.

16.

C.

17
7N

D.

17
6C

E.

18
9F

Energy and Use of Energy


(92)
The main reason why a chain reaction can occur in a nuclear reactor using uranium is that
A. a large quantity of energy is evolved in each fission.
B. the products of nuclear fission are highly radioactive. .
C. plutonium is produced and it undergoes further fission.
D. neutrons are produced when a nucleus undergoes fission.
E.
uranium is a highly radioactive element.
Energy and Use of Energy
(94)
Given:
mass of proton = 1.0073 u
mass of neutron = 1.0087 u
mass of 206
82 Pb = 205.969 u
1 u corresponds to 931 MeV
Find the binding energy per nucleon for a 206
82 Pb nucleus.
A. 7.46 MeV
B. 7.72 MeV
C. 12.39 MeV
D. 12.83 MeV
E.
19.40 MeV
(96)
Given :

Energy and Use of Energy

mass of proton = 1.0073 u


mass of neutron = 1.0087 u
binding energy of an alpha particle = 28.396 MeV
1 u is equivalent to 931 MeV
The mass of an alpha particle is
A. 3.9100 u.
B. 4.0015 u.
C. 4.0320 u.
D. 4.0625 u.
E.
4.1540 u.

17.

Energy and Use of Energy

(97)
Which of the following statements about a typical nuclear reactor in a power plant is/are correct ?
(1) The percentage of 235U isotope in fuel rods is higher than that in natural uranium.
(2) Boron-coated steel rods are used to control the rate of neutron production.
(3) Pressurised water is used as coolant inside the reactor core.
A. (1) only
B. (3) only
C. (1) & (2) only
D. (2) & (3) only
E. (1), (2) & (3)

MC
Chapter 24 The Nucleus

(MC)24P4

18.

(98)
Energy and Use of Energy
When several neutrons and protons come together to form a stable nucleus, which of the following statements
is/are correct ?
(1) Energy is released.
(2) The mass of the nucleus is smaller than the sum of the mass of the individual nucleons.
(3) In the nucleus, the electrostatic repulsion between two protons is overcome by the nuclear force between
them.
A. (1) only
B. (3) only
C. (1) & (2) only
D. (2) & (3) only
E. (1), (2) & (3)

19.

(98)
Energy and Use of Energy
In a nuclear reactor, the 235
92 U nucleus is fissioned by a neutron into two nuclei and a few neutrons are produced.
Which of the following statements about this process is/are correct ?
(1) High energy neutrons are most likely to produce nuclear fission than low energy neutrons.
(2) Graphite can be used to absorb excess neutrons.
(3) The two nuclei produced have smaller neutron-to-proton ratios than that of the uranium nucleus.
A. (1) only

20.

B. (3) only

C. (1) & (2) only

E. (1), (2) & (3)

D. (2) & (3) only

E. (1), (2) & (3)

Energy and Use of Energy


(00)
The following equations represent some typical nuclear reactions
(I)

9
1
o 63 Li  42 He
4 Be1 H 

(II)

2
3
o 42 He  01n
1 H 1H 

85
1
1
(III) 235
o148
92 U  0 n 
57 La  35 Br  3 0 n
Which of the following descriptions of these reactions is/are correct ?
(1) Reaction (I) represents a spontaneous D-decay.
(2) Reaction (II) represents a nuclear fusion.
(3) Reaction (III) represents a chain reaction.
A. (1) only
B. (3) only
C. (1) & (2) only

21.

D. (2) & (3) only

(00)

N
P
The diagram shows the path of an D-particle as it approaches a massive nucleus at N. At point P the D-particle is
nearest to the nucleus. Which of the following statements is correct ?
A. At P the electric potential energy of the D-particles is at a minimum.
B. At P the total energy of the D-particle is at a minimum.
C. At P the angular momentum of the D-particle about N is at a minimum. C is out of syllabus. Wrong.
D. If the initial kinetic energy of the D-particle was greater, the distance between P and N would be larger.
E.
If the atomic number of the nucleus was greater, the distance between P and N would be larger,
22.

23.

(01) Energy and Use of Energy


Isotopes must have the same
(1) nuclear binding energy.
(2) number of nucleons.
(3) number of protons.
A. (1) only
B. (3) only

C. (1) & (2) only

D. (2) & (3) only

E. (1), (2) & (3)

(01) Atomic World


The concept that energy change occurs in discrete and not continuous amounts is NOT REQUIRED to explain
A. absorption line spectrum.
B. nuclear fission.
C. emission of J-rays in radioactive decay.
D. photoelectric effect.
E.
X-ray line spectrum emitted in X-ray tubes.

MC
Chapter 24 The Nucleus

(MC)24P5

24. (01) Energy and Use of Energy


Given: mass of proton = 1.0078 u
mass of neutron = 1.0087 u
mass of deuteron 21 H = 2.0146 u
1 u is equivalent to 931 MeV
Calculate the binding energy per nucleon, in MeV, of a deuteron.
A. 9.5 u 10-4
B. 1.9 u 10-3
C. 8.8 u 10-1
D. 9.4 u 102
E. 1.9 u 103
25.

(03)
The sun and stars generate their energy mainly by
(1) radioactive decay.
(2) nuclear fission.
(3) nuclear fusion.
A. (1) only
B. (3) only
C. (1) and (2) only

D. (2) and (3) only

26.

(04)
The following nuclear reaction represents the two deuterons, 21 H , which combine to form a helium isotope, 23 He ,
with the release of energy.
2 21 Ho 23 He  X  energy
Which of the following statements are correct?
(1) This is an example of nuclear fusion.
(2) The total mass of 23 He and X is greater than that of the two 21 H .
(3) X is a neutron.
A. (1) and (3) only
B. (1) and (2) only
C. (2) and (3) only
D. (1), (2) and (3)

27.

(05) Atomic World


Which of the following phenomena supports the fact that a nucleus is made up of other fundamental constituents ?
A. emission of electrons when a metal is illuminated by light
B. the emission of line spectrum by atoms
C. the scattering of a-particles from a gold foil
D. the decay of radioactive substances

Answers
1. D
11. A
21. E

2. A
12. C
22. B

3. C
13. A
23. B

4. D
14. D
24. C

5. B
15. B
25. B

6. A
16. B
26. A

7. B
17. E
27. D

8. E
18. E

9. A
19. B

10. B
20. D

MC
Chapter 22 Extra-nuclear structure of atom

(MC)22P1

This section is about Atomic World.


1.

(80)

incident light

VS

variable
d.c
source

In an experiment with a photocell, readings were taken of the stopping potential VS for a series of frequencies Q of
the incident light, and the results plotted as a graph of VS against Q are shown in the diagram above. If the emitting
electrode is now changed to one of a different metal, which also gives photoelectrons, the new graph
(1) cuts the VS axis at the same point
(2) cuts the Q axis at the same point
(3) has the same slope
A. (1) only
B. (3) only
C. (1) & (2) only
D. (2) & (3) only
E. (1), (2) & (3)
2.

(81)(84)
For the photoelectric effect, which of the following is the correct relationship between the energy E of a photon,
the work function W of the surface which it strikes, and the maximum kinetic energy K of the emitted
photoelectrons ?
A. E = W + K
B. E = W - K
C. E = K - W
D. K = 2 (W+E)
E.
W = (K + E)/2

3.

(81)
An electron of mass m and charge e, is accelerated by a potential V, then strikes an atom, exciting it from its
ground state to a higher energy state. The electron is scattered with speed u, and the excited atom subsequently
decays back to the ground state with the emission of a photon of frequency f. If h the Planck constant, the value of
u is
A. eV - hf

4.

C.

2(eV  hf )
m

eV  hf
m

D.

2(eV  hf )
m

E.

(82)
Which of the following graphs best represents the variation of the maximum velocity v of the photoelectrons
emitted from a target with the frequency f of the incident light ?
v
A.
B. v
C. v
D. v
E. v

5.

B. 2(eV + hf )

(82)
In an X-ray tube, an accelerating potential V produces X-rays of minimum wavelength 1.2 u 10-10 m. In order to
produce X-rays of minimum wavelength 0.6 u 10-10 m, the accelerating potential should be
1
1
B. V .
C. 2 V.
D. 2V.
E. 4V.
A. V .
4
2

MC
Chapter 22 Extra-nuclear structure of atom

(MC)22P2

6.

(83)
The allowed energy levels for an atom are En = - W / n2, where W is a constant and n is any positive integer. The
frequency of a photon corresponding to the energy transition from n = m + 1 to n = m
(1) is directly proportional W.
(2) decreases as m increases.
(3) increases as the temperature increases.
A. (1) only
B. (3) only
C. (1) & (2) only
D. (2) & (3) only
E. (1), (2) & (3)

7.

(83)
C
E

The above figure shows a photoelectric cell. E is the emitter and C is the collector. The stopping potential for
photoelectrons depends on
(1) the nature of E.
(2) the distance between E and C.
(3) the surface area of E.
A. (1) only
B. (3) only
C. (1) & (2) only
D. (2) & (3) only
E. (1), (2) & (3)
8.

(84)
The ionisation energy of an atom in its ground state is
A. the energy required to separate all the electrons from the remainder of the atom.
B. the maximum energy required to separate one electron from the remainder of the atom.
C. the minimum energy required to separate one electron from the remainder of the atom.
D. the minimum energy required to add one electron to the atom.
E.
the minimum energy required for an electron to change its state while at the same time remaining attached
to the nucleus.

9.

(84)
The minimum wavelength of the X-rays emitted from a hot cathode X-ray tube is controlled by
A. the cathode temperature.
B. the nature of the target.
C. the anode-cathode voltage.
D. the size of the target.
E.
the length of the X-ray tube.

10.

(85)
Red light shines on the photoelectric cell C as shown. If the reading of the microammeter is zero, this may be
explained by the fact that
red
(1) the e.m.f. of the battery is too small.
light
(2) the intensity of the light is too low.
(3) electrode X is made of a material with too great a work
microammeter
C
function.
A
A. (1) only
X
B. (3) only
C. (1) & (2) only
battery
D. (2) & (3) only
E.
(1), (2) & (3)

MC
Chapter 22 Extra-nuclear structure of atom

11.

Out of syllabus
(85)
The photons emitted by a laser
(1) are of the same frequency.
(2) travel in the same direction.
(3) are in phase.
A. (1) only
B. (3) only

C. (1) & (2) only

(MC)22P3

D. (2) & (3) only

E. (1), (2) & (3)

12.

(85)
The ionisation potential of a hydrogen atom is 13.6 V. Which of the following energy levels is/are possible for the
atom ?
(1) - 1.51 eV
(2) - 3.40 eV
(3) - 6.80 eV
A. (1) only
B. (3) only
C. (1) & (2) only
D. (2) & (3) only
E. (1), (2) & (3)

13.

(85)

The figure above shows two adjacent lines in the spectrum of a hot gas. Line X is brighter than line Y because
A. line X has a higher frequency.
B. line X has a greater wavelength.
C. line X originates in the hotter part of the gas.
D. for line X, more electrons undergo transitions between the two states involved in the emission.
E.
for line X, electrons undergo transitions between two states of greater energy difference.
14.

(86)
When a beam of light of intensity I and frequency f is shone on the surface of a metal connected to earth, 200
electrons are ejected from the surface per second. If alight beam of intensity 2I and frequency 2f is used, the
number of electrons ejected from the metal per second will be
A. 50.
B. 100.
C. 200.
D. 400.
E.
800.

15.

(86)
In the diagram above, E1 and Ef represent (to scale) the energy levels of a hydrogen
atom in its ground state and the ionised state respectively. Which of the drawn lines
represents the energy level of the atom in its first excited state?
A. I
B. II
C. III
D. IV
E. V

Ef
V
IV

III

II
I
E1

MC
Chapter 22 Extra-nuclear structure of atom

16.

(87)

(MC)22P4

n= 4
n= 3
n= 2

n =1

The figure shows the four lowest energy levels of a hydrogen atom. The hydrogen atom is excited from ground
state to the energy level n = 3 when an electron collides inelastically with it. What is the minimum energy
required for the electron to do this ? (ionisation potential of hydrogen = 13.6 V)
A. 4.9 eV
B. 12.1 eV
C. 12.8 eV
D. 15.1 eV
E. 20.0 eV
17.

18.

(87)
The figure below shows the currents observed in a photocell
circuit as a function of the p.d. between the plates of the
photocell when light beams I, II, III and IV were each directed in
turn at the cathode. Which of the beams has the highest
frequency ?
A. I
_
B. II
C. III
D. IV
E.
They all have the same frequency

current
I
II
III
IV

+ p.d.

(88)

Light falls on the photo-sensitive metal surface of a photocell. A battery and a sensitive meter are connected to the
photocell as shown. Which of the following statements is correct ?
A. The number of electrons emitted from the metal surface per second is proportional to the potential
difference between the metal surface and the anode.
B. No current is observed in the meter until after a considerable time, when the metal surface has heated up.
C. The maximum energy of the electrons emitted is proportional to the intensity of light.
D. The maximum kinetic energy of the electrons emitted is independent of the particular metal used.
E.
No current is observed in the meter unless the frequency of light is above a minimum value.
19.

(88)

intensity

Out of syllabus

O min

wavelength

The graph above shows the spectrum of X-rays from an X-ray tube. When the accelerating potential is increased,
what will happen to Omin and the wavelengths of the characteristic lines?
Omin
wavelengths of characteristic lines
A. decreased
unchanged
B. decreased
decreased
C. decreased
increased
D. increased
unchanged
E.
increased
decreased

MC
Chapter 22 Extra-nuclear structure of atom

20.

(MC)22P5

E5

(88)

E4
E3
E2

E1

The diagram shows the first five energy levels of an atom. Which of the spectra below best corresponds to the
transitions indicated ?
low

frequency

high

A.
B.
C.
D.
E.

21.

(89)
When light of wavelength 4.0 u 10-7 m is incident on the surface of a metal, the kinetic energy of the electrons
emitted has a maximum value of 3.0 u 10-19 J. What is the longest wavelength of light which would cause
electrons to be emitted from the metal ?
(Planck constant = 6.6 u 10-34 J s, speed of light in air = 3.0 u 108 m s-1)
A. 6.6 u 10-7 m
B. 1.0 u 10-6 m
C. 2.5 u 10-6 m
D. 9.8 u 105 m
E.
1.5 u 106 m

22.

(89)
An atom emits light of wavelengths 122 nm and 103 nm when one of its electrons returns to its ground state from
its first and second excited states, respectively. The wavelength of light emitted when the electron passes from the
second excited state to the first excited state is
A. 19 nm.
B. 12 nm.
C. 113 nm.
D. 225 nm.
E.
661 nm.

23.

(89)
When a beam of white light passes through iodine vapour, the spectrum of the emergent light shows dark lines.
Which of the following statements is/are correct ?
(1) The iodine vapour absorbs from the light all frequencies except those which it emits.
(2) The iodine vapour emits less energy than it absorbs.
(3) The iodine vapour absorbs the same frequencies as it emits.
A. (1) only
B. (3) only
C. (1) & (2) only
D. (2) & (3) only
E. (1), (2) & (3)

MC
Chapter 22 Extra-nuclear structure of atom

(MC)22P6

24.

(90)
In a collision between an electron and an atom which leads to excitation of the atom without ionization, which of
the following is/are correct?
(1) An orbital electron escapes from the nucleus.
(2) An orbital electron acquires energy.
(3) The energy transferred appears later as electromagnetic radiation.
A. (1) only
B. (3) only
C. (1) & (2) only
D. (2) & (3) only
E. (1), (2) & (3)

25.

(90)

4E
3E

E
The diagram shows the energy levels of a certain atom. When an electron changes energy from 4E to E, a photon
of wavelength O, is emitted. Which of the following wavelengths of photons could be produced by other
transitions between the energy levels shown ?
A. O / 3 and 2O / 3
B. O / 3 and 3O
C. 2O / 3 and 3O/2
D. 2O / 3 and 3O
E.
3O/2 and 3O

26.

(90)
The photoelectric effect occurs when monochromatic light falls upon a metal surface in a photocell. What
happens when the light intensity increases?
A. More electrons are emitted with unchanged speed.
B. More electrons are emitted with increased speed.
C. The same number of electrons is emitted with increased speed.
D. More photons are emitted from the surface.
E.
Photons of greater energy are emitted from the surface.

27.

(91)
Which of the following illustrates atomic resonance?
(1) Photoelectric emission
(2) Absorption spectrum
(3) Fraunhofer dark lines in the sun's spectrum
A. (1) only
B. (3) only
C. (1) & (2) only

28.

D. (2) & (3) only

E. (1), (2) & (3)

(91)
Electron transitions occur in an atom resulting in the emission of the following light wavelengths:
from level C to level A : 600 nm
from level B to level A : 500 nm
Which of the following statements is/are correct ?
(1) Level A has a lower energy than both levels B and C.
(2) Level C has a higher energy level B.
(3) The wavelength of light emitted for the transition between C and B is 100 nm.
A. (1) only
B. (3) only
C. (1) & (2) only
D. (2) & (3) only

E. (1), (2) & (3)

MC
Chapter 22 Extra-nuclear structure of atom

(MC)22P7

29.

(91)
A beam of monochromatic light falls on a surface. If the frequency of the light is doubled but the intensity
remains unchanged, which of the following statements is/are correct ?
(1) The photon energy is doubled.
(2) The number of photons falling on the surface per second is halved.
(3) The maximum kinetic energy of photoelectrons ejected is doubled.
A. (1) only
B. (3) only
C. (1) & (2) only
D. (2) & (3) only
E. (1), (2) & (3)

30.

(91)
The minimum wavelength of the X-rays emitted from a hot cathode X-ray tube is controlled by
(1) the cathode temperature.
(2) the nature of the target.
(3) the anode-cathode voltage.
A. (1) only
B. (3) only
C. (1) & (2) only
D. (2) & (3) only
E. (1), (2) & (3)

31.

(91)
In an experiment on the photoelectric effect, a beam of monochromatic light is directed onto a metal plate to
liberate electrons. Which of the following statements is true ?
A. The velocity of the fastest electrons is directly proportional to the frequency of the incident light.
B. The velocity of the fastest electrons is directly proportional to the intensity of the incident light.
C. The kinetic energy of the fastest electrons is directly proportional to the frequency of the incident light.
D. The velocity of the fastest electrons is independent of the intensity of the incident light.
E.
The velocity of the fastest electrons is independent of the type of metal.

32.

(92)
The work function of a metal is the least energy required to
A. release one mole of electrons from the surface of the metal.
B. bring one mole of electrons from the interior of the metal to the surface.
C. release one electron from the surface of the metal.
D. bring one electron from the interior of the metal to the surface.
E.
bring one electron from the interior of the metal to the surface and release it

33.

(93)
An X-ray tube emits X-rays with a minimum wavelength of 3.55 u 10-11 m. Estimate the potential difference
between the cathode and the anode (target) in the X-ray tube.
Given: Planck constant = 6.63 u 10-34 J s, Electronic charge = 1.6 u 10-19 C.
A. 20000 V
B. 25000 V
C. 30000 V
D. 35000 V
E.
40000 V

34.

(93)
The spectrum of sunlight has dark lines in it. Which of the following statements concerning these dark lines is/are
correct ?
(1) They are due to the absorption of certain wavelengths of light in the sun's atmosphere.
(2) Light absorbed in the sun's atmosphere is re-emitted but in all directions.
(3) They are due to the absorption of certain wavelengths of light in the earth's atmosphere.
A. (1) only
B. (3) only
C. (1) & (2) only
D. (2) & (3) only
E. (1), (2) & (3)

MC
Chapter 22 Extra-nuclear structure of atom

(MC)22P8

35.

(93)
The ionisation potential of a hydrogen atom is 13.6 V. What is the minimum excitation potential of a ground state
hydrogen atom?
A. 1.9 V
B. 3.4 V
C. 6.8 V
D. 10.2 V
E.
12.8 V

36.

(94)
V
A student measures the p.d. V to stop photoelectrons emitted in a photocell
illuminated by monochromatic light of various frequencies f. The resulting points
when plotted on a V - f graph (as shown) do not lie on the solid line drawn from
standard results obtained with a similar photocell. The reason could be
A. the standard results are obtained with light of higher intensity.
0
B. he has used a voltmeter which has a fixed zero error.
C. he has read the wrong scale on his voltmeter so that his readings always double the actual readings.
D. he has connected the variable d.c. supply with the wrong polarities to the photocell.
E.
he has plotted the wavelength of light in place of the frequency on the horizontal axis.

37.

(94)
ln a hydrogen atom, electron transitions from the first excited state to the ground state give photons of frequency f.
If an electron falls from the second excited state to the first one, the frequency of the photon emitted would be
A. 0.19 f
B. 0.44 f
C. 0.84 f
D. 1.19 f
E.
2.25 f

38.

(95)
When an electron in an atom undergoes a transition from a higher energy level to a lower one, the time taken is
about 10-9 s. Which of the following statements about the transition is/are correct?
(1) Electromagnetic radiation is emitted during the transition.
(2) The length of the wave train from such a transition is about 0.3 m.
(3) The energy of the photon emitted depends on the energy difference between the two levels.
A. (1) only
B. (3) only
C. (1) & (2) only
D. (2) & (3) only
E. (1), (2) & (3)

39.

(95)
When light of frequency f1 is shone on to a metal surface, the maximum energy of the electrons emitted is E1. If
the same surface is illuminated with light of frequency f2, the maximum energy of the electrons emitted is E2. The
Planck constant is given by
f 2 E1  f1 E2
A.
f1 f 2
B.

f 2 E1  f1 E2
f1 f 2

C.

E1  E2
f1  f 2

D.

E1  E2
f1  f 2

E.

E1  E2
f1  f 2

MC
Chapter 22 Extra-nuclear structure of atom

40.

(96)

(MC)22P9

energy
0 eV
-1 eV

-4 eV

-12 eV

A hypothetical atom has only four energy levels as represented above. It can change from any one level to any
other. Which of the following statements about this hypothetical atom is INCORRECT ?
A. The most stable state of the atom corresponds to the level -12 eV.
B. If the atom is at the level -4 eV, it can absorb a photon of energy 4 eV.
C. If the atom is at the level --4 eV, it can emit a photon of energy 8 eV.
D. If the atom is at the level -12 eV and collides with an electron of kinetic energy 10 eV, it can change to the
level -4 eV.
E.
If the atom is at the level -12 eV and collides with two photons, each of energy 4 eV, it can change to the
level -4 eV.
41.

42.

(96)
A metal surface is illuminated with monochromatic light so that it emits photoelectrons. The maximum kinetic
energy of the emitted photoelectrons depends on
(1) the distance of the metal surface from the light source.
(2) the work function of the metal surface.
(3) the wavelength of the incident monochromatic light.
A. (1) only
B. (3) only
C. (1) & (2) only
D. (2) & (3) only
E. (1), (2) & (3)
(97)

incident
monochromatic
radiation

C
A

d.c. source

A d.c. source is applied to a photocell as shown. Monochromatic radiation is incident on cathode C so that
photoelectrons are emitted from the cathode surface. The maximum kinetic energy of the photoelectrons reaching
anode A can be increased by using
(1) a d.c. source of higher voltage.
(2) monochromatic radiation of longer wavelength.
(3) the same monochromatic radiation but of higher intensity.
A. (1) only
B. (3) only
C. (1) & (2) only
D. (2) & (3) only
E. (1), (2) & (3)
43.

(97)
The transition of electrons between three energy levels in a particular atom gives rise to three spectral lines. The
shortest and longest wavelengths of those spectral lines are O1 and O2 respectively. The wavelength of the other
spectral line is
O  O2
O1O2
1
1
B. O2 - O1
C.
D. (  ) 1
E. O1O2
A. `1
2
O1  O2
O1 O2

MC
Chapter 22 Extra-nuclear structure of atom

44.

(MC)22P10

(98)

The diagram shows some energy levels (drawn to scale) of a certain atom. Transition X results in the emission of a
photon of wavelength 600 nm. Which transition (P to T) would result in the emission of a photon of wavelength
300 nm ?
A. P
B. Q
C. R
D. S
E. T
45.

(99)
The following are energy values for hydrogen.
X
mean kinetic energy of hydrogen gas molecules at room temperature.
Y
ionization energy of a hydrogen.
Z
photon energy of a particular line in the visible emission spectrum of
hydrogen
Which of the following shows the correct ascending order (the least first) of these energy values ?
A. X<Z<Y
B. X< Y<Z
C. Y<Z<X
D. Z<Y<X
E.
Z<X<Y

46.

(99)
When an electron in an atom falls from an excited state to the ground state, which of the following forms of
radiation is not emitted ?
A. infra-red
B. gamma-ray
C. ultra-violet
D. visible light
E.
X-rays

47.

(99)

f
The above graph shows the variation in maximum kinetic energy K of photoelectrons with the frequency f of the
incident radiation on a metallic surface. If radiation of twice the intensity is used, which of the following graphs
(dotted line) shows the variation of K with f ?
A. K
B. K
C. K
D. K
E. K

MC
Chapter 22 Extra-nuclear structure of atom

48.

(MC)22P11

(00)
In a series of photoelectric emission experiments on a certain metal surface, relationships between the following
physical quantities were investigated.
f = frequency of incident light
I = intensity of incident light
i = photoelectric current
K = maximum kinetic energy of photoelectrons.
Two of these quantities, when plotted on a graph of y against x, would give a straight line through the origin.

Which of the following correctly identifies x and y ?


(Assume the frequencies used are greater than the threshold frequency.)
y
x
A. K
i
B. f
K
C. f
i
D. I
K
E.
I
i
49.

(00)
Energ
0

-4E
-6E

The energy levels of a certain atom are as shown. Which of the following may undergo an inelastic collision with
the atom ?
(1) an electron with kinetic energy 3E
(2) a photon with energy 2E
(3) a photon with energy 3E
A. (1) only
B. (3) only
C. (1) & (2) only
D. (2) & (3) only
E (1), (2) & (3)
50.

(01)
A certain photocell emits electrons when illuminated with yellow light. This photocell will PROBABLY NOT
emit electrons when illuminated with
A. blue light.
B. green light.
C. red light.
D. ultra-violet radiation.
E.
X-rays.

51.

(01)
Which of the following electron transitions between energy levels in an atom will emit electromagnetic radiation
of the highest frequency?
A. n = 2 to n = 1
B. n = 3 to n = 2
C. n = 4 to n = 3
D. n = 4 to n = 2
E.
n = 5 to n = 2

MC
Chapter 22 Extra-nuclear structure of atom

(MC)22P12

52.

(02)
To produce an X-ray photon of energy 3.6 u 10-15 J, the bombarding electrons in the X-ray tube must be accelerated
through a potential difference of at least
(Given: electronic charge = 1.6 u 10-19 C)
A. 11.3 kV.
B. 22.5 kV.
C. 28.8 kV.
D. 57.6 kV.

53.

(02)
A beam of red light falls on one electrode of a photocell and electrons are emitted. The red beam is then replaced by
a blue one that has the same intensity. Which of the following physical quantities would decrease as a result of this
change?
A. The maximum kinetic energy of the photoelectrons emitted
B. The energy of each photon striking the electrode
C. The number of photons striking the electrode per second
D. The magnitude of the potential difference across the photocell required to reduce the photo-electric current to
zero

54.

(02)
In which of the following calculations is it necessary to use the Planck constant?
(1) Finding the momentum of an electron from its kinetic energy
(2) Finding the energy of a photon emitted as a result of the transition of an electron between two energy
levels in an atom
(3) Finding the maximum kinetic energy of photoelectrons from the wavelength of incident radiation
A. (1) only
B. (3) only
C. (1) and (2) only
D. (2) and (3) only

55. (02)
A hydrogen atom absorbs a photon of wavelength O such that the electron in the ground state (energy level
corresponding to n = 1) is brought to an excited state (energy level corresponding to n = 3). What is the maximum
wavelength of a photon that can cause ionization of a hydrogen atom in the ground state?
8O
9O
3O
2O
B.
C.
D.
A.
9
8
2
3
56.

(03)
The second line in the Lyman series (corresponding to the K-shell) of the hydrogen spectrum has a wavelength of
102 nm. What is the wavelength of the first line in the series?
A. 93.5 nm
B. 121 nm
C. 136 nm
D. 153 nm

57. (03)
In an experiment on the photoelectric effect, a student measured the potential Vs required to prevent photoemission
when a metal was illuminated with radiations of varying wavelength O. His observations led him to plot the graph
as shown, but he omitted the axes labels. The correct labels for the axes are

B.
C.

x
1
O
Vs
O

D.

Vs

A.

y
Vs
O
Vs
1
O

MC
Chapter 22 Extra-nuclear structure of atom

(MC)22P13

58. (04)
A beam of blue light fal1s on the cathode of a photocel1 so that electrons are emitted. The blue beam is then
replaced by a yel1ow one with the same intensity and electrons are also emitted. What would happen to each of the
fol1owing physical quantities when the blue beam is replaced by the yel1ow beam?
K : the maximum kinetic energy of the electrons emitted
I : the magnitude of the photoelectric current
I
K
A.
increase remain unchanged
B.
decrease remain unchanged
C.
decrease increase
D.
decrease decrease
59. (04)
The ionization energy for a hydrogen atom in ground state is 13.6 eV. If the atom is in the first excited state, the
energy for ionizing it should be
A. 3.4 eV.
B. 4.5 eV.
C. 6.8 eV.
D. 10.2 eV.
60. (04)
Which of the following descriptions about spectra is/are correct?
(1) The emission spectrum from a very hot liquid is continuous.
(2) The dark lines in the solar spectrum illustrate that the elements corresponding to these dark lines are absent
from the sun.
(3) The dark lines in the solar spectrum are due to the solar radiation being absorbed by the elements in the earth's
atmosphere corresponding to these dark lines.
A. (1) only
B. (3) only
C. (1) and (2) only
D. (2) and (3) only

61.

(05)

n=4
n=3

-1.6 eV
-2.7 eV

n=2

-5.5 eV

n=1

-10.4 eV

The figure shows the possible energy levels of a mercury atom. A free electron with kinetic energy 8.4 eV collides
with a mercury atom, which is in its ground state. The change in kinetic energy of the mercury atom in the
collision may be neglected. What is/are the possible value(s) for the kinetic energy of the electron after collision ?
(1) 0.7 eV
(2) 2.9 eV
(3) 3.5 eV
A. (1) only
B. (2) only
C. (2) and (3) only
D. (1) and (3) only
62.

(05)
Which of the following descriptions about spectra is/are correct ?
(1) A tungsten-filament lamp emits a continuous spectrum of light.
(2) An electrical discharge through a gas at low pressure produces a line spectrum.
(3) A gas produces an emission spectrum at high temperatures.
A.
(1) and (2) only
B.
(1) and (3) only
C.
(2) and (3) only
D.
(1), (2) and (3)

MC
Chapter 22 Extra-nuclear structure of atom

63.

(MC)22P14

(06)
It is known that the energy of a photon of yellow light is about 2 eV. The energies of some particles are stated
below:
(1) the energy of a photon of red light from a ruby laser
(2) the average kinetic energy of an air molecule near room temperature
(3) the kinetic energy of an electron in a cathode-ray tube arriving at the screen
Given : Boltzmann constant = 1.38 u 10-23 J K-1
1 eV = 1.60 u l0-19 J
Use the information to arrange the above in the order of increasing energy.
A.
B.
C.
D.

(2), (1), (3)


(1), (2), (3)
(3), (2), (1)
(3), (1), (2)

Answer
1.
11.
21.
31.
41.
51.
61.

B
E
B
D
D
A
D

2.
12.
22.
32.
42.
52.
62.

A
C
E
C
A
B
A

3.
13.
23.
33.
43.
53.
63.

E
D
B
D
D
C
A

4.
14.
24.
34.
44.
54.

C
C
D
C
B
B

5.
15.
25.
35.
45.
55.

D
D
E
D
A
A

6.
16.
26.
36.
46.
56.

C
B
A
B
B
B

7.
17.
27.
37.
47.
57.

A
B
D
A
A
A

8.
18.
28.
38.
48.
58.

C
E
A
E
E
C

9.
19.
29.
39.
49.
59.

C
A
C
E
C
A

10.
20.
30.
40.
50.
60.

B
E
B
E
C
A

You might also like